117
Universidad La Salle. Facultad Mexicana de Medicina. Curso de Extensión Universitaria para la Preparación del Examen Nacional para Aspirantes a Residencias Médicas. Examen Final 2016 1.- Se trata de masculino de 9 meses, ingresa al servicio de urgencias por presentar en 24 hrs., 7 evacuaciones diarreicas, sin sangre, fiebre no cuantificada y vómito 4 veces. La madre refiere que en las últimas horas el menor está hiporéxico pero bebe con avidez. E.F. : Inquieto, fontanela anterior deprimida, ojos hundidos, llanto sin lágrimas, lienzo húmedo negativo, pulsos normales, llenado capilar de 3 segundos. El agente causal más probable en éste paciente es: a) Parasito. b) Virus. c) Bacteria. d) Toxina. Causada por un agente infeccioso Es el tipo más frecuente. La etiología del 70% de los casos de diarrea infecciosa es la viral. 4 Todos los años, las causas principales de gastroenteritis infantil son los Rotavirus, causantes de 600.000-800.000 muertes en todo el mundo. El virus infecta los enterocitos del intestino, disminuye la actividad de las enzimas que digieren los azúcares, y disminuye la reabsorción del ión Na+ y del agua en el intestino. Además producen activación del sistema nervioso entérico y la secreción de iones Cl-. Todo ello produce un exceso de fluidos en la luz intestinal, que tiene como consecuencia una diarrea acuosa. Otros agentes etiológicos virales son los Norovirus, que ejercen una acción directa sobre la actividad de las enzimas de los enterocitos. Gastroenteritis Virales Agentes etiológicos Reovirus (Rotavirus) Calicivirus (Agente Norwalk) Adenovirus (entéricos) Astrovirus Otros: Torovirus, Picornavirus, Picornabirnavirus Causas de Diarrea Infantil Aguda 9 Acuosa Con sangre (Disentérica) Secretora Osmótica Invasiva No invasiva

Examen final 20460

Embed Size (px)

Citation preview

Page 1: Examen final 20460

Universidad La Salle. Facultad Mexicana de Medicina.

Curso de Extensión Universitaria para la Preparación del Examen Nacional para Aspirantes a Residencias Médicas.

Examen Final 2016 1.- Se trata de masculino de 9 meses, ingresa al servicio de urgencias por presentar en 24 hrs., 7 evacuaciones diarreicas, sin sangre, fiebre no cuantificada y vómito 4 veces. La madre refiere que en las últimas horas el menor está hiporéxico pero bebe con avidez. E.F. : Inquieto, fontanela anterior deprimida, ojos hundidos, llanto sin lágrimas, lienzo húmedo negativo, pulsos normales, llenado capilar de 3 segundos. El agente causal más probable en éste paciente es:

a) Parasito. b) Virus. c) Bacteria. d) Toxina.

Causada por un agente infeccioso Es el tipo más frecuente. La etiología del 70% de los casos de diarrea infecciosa es la viral.4 Todos los años, las causas principales de gastroenteritis infantil son los Rotavirus, causantes de 600.000-800.000 muertes en todo el mundo. El virus infecta los enterocitos del intestino, disminuye la actividad de las enzimas que digieren los azúcares, y disminuye la reabsorción del ión Na+ y del agua en el intestino. Además producen activación del sistema nervioso entérico y la secreción de iones Cl-. Todo ello produce un exceso de fluidos en la luz intestinal, que tiene como consecuencia una diarrea acuosa. Otros agentes etiológicos virales son los Norovirus, que ejercen una acción directa sobre la actividad de las enzimas de los enterocitos. Gastroenteritis Virales Agentes etiológicos Reovirus (Rotavirus) Calicivirus (Agente Norwalk) Adenovirus (entéricos) Astrovirus Otros: Torovirus, Picornavirus, Picornabirnavirus Causas de Diarrea Infantil Aguda9

Acuosa Con sangre

(Disentérica)

Secretora

Osmótica Invasiva No invasiva

Page 2: Examen final 20460

Rotavirus Vibrio choleraeECET Vibrios no cóleraShigella

Virus G. lambliaCryptosporidiumLaxantes Desnutrición

Shigella ECEI Salmonella no tifoideaCampylobacter jejunYersinia

E. coli O157:H7ECEP Clostridium difficile

2.- Se trata de femenino de 6 años ingresa al servicio de urgencias, refiere la madre que la menor ha presentado tos, ardor retroesternal y silbido del pecho. . Tiene tos irritativa, no productiva, hipertermia no cuantificada. Antecedentes: Originaria de Veracruz, nivel socioeconómico bajo, habita en un rancho. Malos hábitos de higiene. Exploración física: febril 39 °c, con dificultad para respirar, aleteo nasal, tiraje intercostal y subcostal. Laboratorio: eosinofilia marcada. Radiografía de tórax muestra infiltrados redondos u ovalados de unos milímetros hasta varios cm. de diámetro, bilateral. El tratamiento de elección en éste síndrome es:

a) TMP/SMZ 800mg / 160MG c/12 hrs b) Amikacina 30 mg. por kg. de peso para 24 hrs. c) Albendazol 400 mg dosis unica. d) Clindamicina 300 mg v.0. cada 8 hrs

Page 3: Examen final 20460

Tratamiento. Mebendazol (100 mg/12 h/3 dias o 500 mg a dosis unica). Alternativas: pamoato de pyrantel, 10 mg/kg (maximo 1 g), dosis unica; albendazol, 400 mg, dosis unica (ninos entre 2-5 anos: 200 mg); piperazina, 75 mg/kg (maximo 3,5 g en adultos o 2,5 g en Niños entre 2-12 años). La ivermectina tambien es eficaz. (200 μg/kg dosis unica). Parasitosis intestinales J. Gascón Brustenga y J. Muñoz Gutiérrez 3.- Se trata de recién nacido pretermino de 30 semanas de gestación, peso de 1200 gr. Cursó con enfermedad de membranas hialinas por lo que se manejo con una dosis de surfactante y manejo con ventilación convencional por 3 días. Posteriormente inició con alimentación enteral por sonda orogástrica con leche humana, sin tolerancia a la misma (residuo gástrico y distensión abdominal), sangre oculta en heces e ictericia. El diagnóstico más probable es:

a) Alergia a las proteínas de la leche b) Enterocolitis necrotizante c) Atresia intestinal d) Gastroenteritis infecciosa

Page 4: Examen final 20460

La enterocolitis necrotizante (ECN) es una enfermedad grave que afecta a recién nacidos, en especial prematuros, con una incidencia y morbimortalidad elevados. Constituye la urgencia gastrointestinal más frecuente en las UCI neonatales. Se presenta como un síndrome gastrointestinal y sistémico que comprende síntomas variados y variables, como distensión e hipersensibilidad abdominal, sangre en heces, intolerancia a la alimentación, apnea, letargia, y en casos avanzados acidosis, sepsis, CID y shock.

El síndrome clínico ha sido clasificado en estadios por Bell y col. (1978) y modificado por Walsh y Klegman (1986) para incluir hallazgos sistémicos, intestinales y radiológicos.

A. Estadio I : sospecha de enterocolitis necrotizante

Los hallazgos sistémicos son inespecíficos.

Los hallazgos intestinales incluye el residuo gástrico y heces guayaco –positivas.

Los hallazgos radiológicos son normales e inespecíficos.

B. Estadio II A: enterocolitis necrotizante leve

Los hallazgos sistémicos son similares al estadio I.

Los hallazgos intestinales incluyen distensión abdominal prominente con hipersensibilidad a la palpación o sin ella, ruidos hidroaéreos ausentes, sangre macroscópica en materia fecal.

Los hallazgos radiológicos, íleo con asas dilatadas con áreas focales de neumatosis intestinal.

C. Estadio II B: enterocolitis necrotizante moderada

Los hallazgos sistémicos incluyen acidosis leve y trombocitopenia

Los hallazgos intestinales incluyen edema de la pared abdominal e hipersensibilidad a la palpación con una masa palpable o sin ella.

Los hallazgos radiológicos incluyen neumatosis extensa y ascitis temprana.

Puede haber gas en la vena porta intrahepática.

D. Estadio IIIA: enterocolitis necrotizante avanzada:

Los hallazgos sistémicos incluyen acidosis respiratoria y metabólica, ventilación asistida por apnea, hipotensión arterial, oliguria, neutropenia y coagulación intravascular diseminada.

Los hallazgos intestinales incluyen edema que disemina, eritema e induración del abdomen.

Los hallazgos radiológicos incluyen ascitis prominente y asa centinela persistente sin perforación.

E. Estadio IIIB: enterocolitis necrotizante avanzada:

Los hallazgos sistémicos revelan signos vitales e índices de laboratorio en deterioro, síndrome de shock y desequilibrio electrolítico.

Los hallazgos intestinales y radiológicos muestran evidencias de perforación.

Hallazgos radiológicos en la radiografía de abdomen

Distensión abdominal generalizada

Íleo paralítico

Neumatosis intestinal quistoide (patognomónico)

La neumatosis intestinal quistoide se la puede visualizar de varias maneras, cúmulos de gas lineales, curvilíneos, esponjosos y espumosos.

Page 5: Examen final 20460

Este último debe distinguirse de materia fecal o meconio mezclado con aire.

Cualquier lactante con sospecha de enterocolitis necrotizante en el que se encuentren radiográficamente colecciones de aire lineales, curvilíneas esponjosas o espumosas debe considerarse que tiene neumatosis intestinal quistoide hasta que se demuestre lo contrario.

La neumatosis intestinal quistoide suele verse con mayor frecuencia en el colon, pero puede ocurrir desde estomago hasta recto.

1. Hartmann G. E., Drugas G. T., Shochat S. J. Post-necrotizing enterocolitis strictures presenting with sepsis of perforation: risk of clinical observation. J. Pediatr. Surg. 1988; 23: 562-6.

2. Kosloske A. M., Burstein J., Bartow S. A. Intestinal obstruction due to colonic stricture following neonatal necrotizing enterocolitis. Ann Surg. 1980 Aug;192 (2): 202-7.

3. Schwartz M. Z., Hayden C. K., Richardson C. J., Tyson K. R., Lobe T. E. A prospective evaluation of intestinal stenosis following necrotizing enterocolitis. J. Pediatr. Surg. 1982 Dec; 17 (6): 764-70.

4. Bell M. J., Ternberg J. L., Askin F. B. Intestinal stricture in necroting enterocolitis. J. Pediatr. Surg. 1976; 11: 319-27.

5. Pokorny W. J., Harr V. L., McGill, C. W., et al; Intestinal stenosis resulting from necrotizing enterocolitis. Am J. Surg 1981 42: 721-724.

6. Schimpl G., Hollwarth M. E., Fotter R., Becker H. Late intestinal strictures following successful treatment of necrotizing enterocolitis. Acta Paediatr. Suppl. 1994; 396: 80-3.

7. Bütter A., Flageole H., Laberge J. M. The Changing face of Surgical Indication for Necrotizing Enterocolitis J. Pediatr. Surg. 2002; 37: 469- 499.

Gobet R. , Sacher P. , Schwobel M. G. Surgical procedures in colonic strictures after necrotizing enterocolitis. Acta Paediatr. Suppl. 1994;396:77-9. 4.- Ingresa al servicio de urgencias paciente masculino de 2 años que presenta letargo después de haber estado jugando sin vigilancia. El niño hasta entonces estaba sano, con un desarrollo normal. En la historia familiar aparece como dato importante que uno de sus hermanos tiene epilepsia y toma fenobarbital. La exploración revela un niño letárgico sin fiebre que abre brevemente los ojos ante estímulos dolorosos. No hay signos de traumatismo ni anomalías focales. Las pupilas son pequeñas pero reactivas. La etiología más probable relacionada al letargo del niño es: a) Convulsiones no observadas. b) Intoxicación. c) Hemorragia intracraneal. d) Traumatismo craneoencefálico no observado. . La causa más común de inicio agudo de somnolencia en un niño que empieza a caminar, es la intoxicación. La presencia de agentes tóxicos potenciales en la casa es importante en la historia clínica. La miosis también sugiere intoxicación y la falta de signos de traumatismo o anomalías focales hace que una hemorragia sea poco probable. LECTURA RECOMENDADA PROGRAMA DE ACTUALIZACION CONTINUA EN PEDIATRIA INTOXICACIONES EN PEDIATRÍA Dr. Miguel Angel Montoya Cabrera

Page 6: Examen final 20460

Jefe del Departamento de Admisión Continua y Toxicología, Hospital de Pediatría, Centro Médico Nacional Siglo XXI, Instituto Mexicano del Seguro Social. Miembro de la Academia Nacional de Medicina, de la Academia Mexicana de Cirugía y de la Academia Mexicana de Pediatría 5.- Femenino de 8 meses refiere la madre que ha presentado cuadros frecuentes de vómitos, movimientos incordinados y crisis convulsivas de difícil control, se le ha puesto el pelo más claro que al resto de sus hermanos, piel seborreica, microcefalia, maxilar prominente, dientes espaciados y retraso en el crecimiento. El diagnóstico más probable de éste paciente es: a) Citrulinemia b) Homocistinuria c) Fenilcetonuria d) Enfermedad de Jarabe de Maple.

La fenilcetonuria, es un tipo de hiperfenilalaninemia, también conocida como PKU, es una alteración del metabolismo en el que el organismo no puede metabolizar el aminoácido tirosina a partir de fenilalanina en el hígado. Esta enfermedad es genética y es provocada por la carencia de enzima fenilalanina hidroxilasa .

La fenilcetonuria tiene como rasgo principal la herencia genética autosómica recesiva, es decir, los padres son portadores de los genes defectuosos y al ser traspasados de ambos progenitores, la enfermedad se expresa en los descendientes.

La causa de la enfermedad es la carencia de la enzima fenilalanina hidroxilasa (FAOH) o de la dihidropterina reductasa (DHPR) (también llamada tirosina hidroxilasa). Ambas enzimas son responsables de la hidroxilación del aminoácido fenilalanina en la reacción que produce tirosina. Por ello, el defecto o falta de alguna de ellas determina un incremento de la concentración sanguínea de fenilalanina al impedirse su transformación en tirosina. También se aumenta la transaminación de la fenilalanina como vía metabólica alternativa, y asimismo se acumulan los metabolitos fenilpiruvato, fenilactato y fenilacetato. El defecto en la síntesis de FAOH se debe a una anomalía génica localizada en el cromosoma 12, y el de la DHPR en el cromosoma 4. Existen también formas de la enfermedad con déficits parciales.

El fenilpiruvato es un neurotori que afecta gravemente al cerebro durante el crecimiento y el desarrollo. Los efectos de su acumulación causan oligofrenia fenilpirúvica, caracterizada por un cociente intelectual inferior a 20. Los primeros meses de vida, los niños que padecen esta enfermedad parecen estar sanos. Entre los tres y los seis meses pierden el interés por el entorno, y al año se evidencia un retraso importante en su desarrollo. Los síntomas suelen ser retraso psicomotor, cuadros psicóticos de tipo autista, convulsiones, síndrome de West, convulsiones generalizadas y un eczema facial muy rebelde. Por lo general su desarrollo físico es bueno, tienden a tener el cabello más claro que sus hermanos, piel clara, y presentan un olor característico a paja mojada.

Cuadro clínico

La enfermedad se manifiesta, por primera vez, algunas semanas después del nacimiento, iniciándose con una elevación en el plasma de la fenilalanina hasta un nivel 30 veces superior al normal y por la excreción de ácido fenilpirúvico por la orina. Transcurridos 6 meses se hace patente el retraso del desarrollo mental. La mayor parte de los pacientes son deficientes graves o profundos y en ocasiones se alcanza la deficiencia media.

Page 7: Examen final 20460

El portador de esta anomalía, que nace tras un embarazo normal y sin complicaciones, se desarrolla durante los primeros meses casi siempre sin mostrar anormalidad ninguna. Sin embargo Partington encontró, casi en la mitad de los lactantes, la existencia de vómitos en los primeros meses de vida y en un tercio de ellos una irritabilidad inacostumbrada. En una proporción similar de casos, a los padres ya les había llamado la atención un desagradable olor del cuerpo del niño. Una parte de ellos mostró dermatosis eczematiformes durante el primer trimestre, 7 de 36 ya había tenido en el primer año de vida ataques convulsivos. A los 9 meses llama la atención el retraso en el desarrollo psicomotor.

Datos físicos

El desarrollo corporal cursa casi con normalidad. No obstante puede comprobarse cierta tendencia al enanismo, aunque también se han descrito casos con tallas superior a la frecuente. La dentición suele retrasarse hasta después del undécimo mes.

La gran mayoría de los enfermos muestran una piel clara, ojos azules, y color claro del pelo. Alrededor del 10% poseen cabellos oscuros. La pobreza de pigmentos llama más la atención en los pueblos de cabellos oscuros. La piel de los portadores además de ser clara es muy suave aterciopelada y muy sensible. En algunos enfermos se han observado eflorescencias papulosas en las caras de extensión de las extremidades y en la faz. En ciertos pacientes se puede encontrar también una tendencia a la acrocianosis.

Datos conductuales

Características clínicas raras

Cifosis. Pies planos. Espina bifida. Sindactilia en los dedos de los pies. Bloqueo cardiaco intraventricular. Hipogenitalismo. Dermografismo. Sensibilidad a la luz. Hipersegmentación de las células neutrófilas de la sangre. Disminución de la tolerancia a la galactosa. Metabolismo basal ligeramente elevado.

Las etapas del desarrollo habitual, la edad en la que el niño se sienta y habla, a veces, se alcanzan a la edad normal, pero, de ordinario, se retrasa. En la edad límite en que debe esperarse que el niño normalmente realice estos actos, el 35% no puede andar y el 63% no puede hablar.

Estos niños, en general, tienen un peso y talla promedio por debajo del correspondiente a su edad. En la mitad de los casos tiene microcefalia y prominencia del maxilar.

Sus movimientos son lentos y patosos y a menudo suelen adoptar la posición de sastre. Las anomalías del tono muscular que contribuyen a estos cambios son de origen neurológico.2 de cada 3 pacientes tienen hiperreflexia tediciosa e hipercinesia sobreañadida estos últimos son voluntarios y muy variados

Referencias 1. Scriver CR, Kaufman S. 2001. Hyperphenylalaninemia: phenylalanine hydroxylase deficiency. In Scriver CR, Beaudet AL, Valle D, Sly WS (eds). The Metabolic and Molecular bases of Inherited Disease. McGraw-Hill., pp

Page 8: Examen final 20460

1667-1724. 2. Martínez-Pardo M, Marchante C. et al: 1998. ”Protocolo de diagnóstico, tratamiento y seguimiento de las hiperfenilalaninemias”. An. Esp. Pediatría suplemento 114: 3-8. 10 3. Belanger-Quintana A, Morais A, Mena MA, Martínez- Pardo M. 2004. Niveles de neurotransmisores dopaminérgicos y serotoninérgicos en líquido cefalorraquídeo en niños. An Esp Pediatr 60:82. 6.- Femenino de 39 años de edad la cual inicia con un cuadro de exoftalmos axial de varias semanas de evolución, con predominio en su ojo derecho. En la exploración se aprecia una conjuntiva con síntomas discretos de hiperemia y edema, una queratitis de carácter punteado en tercio inferior corneal y se sospecha una retracciónpalpebral al observar cómo el borde del párpado superior se encuentra por encima del limbo, permitiéndonos visualizar la esclerótica. La paciente no refiere diminución de visión ni alteraciones tipo visión doble y toma presión intraocular con parámetros dentro de los normales. De los siguientes el diagnóstico más probable es: a) Tumor intraorbitario. b) Enfermedad de Graves-Basedow. c) Queratoconjuntivitis epidémica. d) Tumor intraocular. OFTALMOPATÍA TIROIDEA. Es la causa más frecuente de exoftalmos tanto bilateral como unilateral en adultos. La forma típica aparece en pacientes con enfermedad de Graves-Basedow, en los que podemos encontrar exoftalmos y síndrome palpebro-retráctil. Este síndrome ocular puede aparecer en pacientes eutiroideos o hipotiroideos, pudiendo constituir el signo más precoz de una tirotoxicosis incipiente. CLÍNICA. Se distinguen dos formas clínicas: a) Tirotóxica (hipersensibilidad a las catecolaminas y habitualmente hay hipertiroidismo): exoftalmos moderado depresible. Hay edema del contenido orbitario, pero no fibrosis ni oftalmoparesia. b) Maligna (inflamación orbitaria autoinmune y puede haber normo o hipotirodismo): exoftalmos irreductible severo con oftalmopejia progresiva y queratitis por exposición. Puede afectar al nervio óptico por compresión y producir pérdida visual. Hay fibrosis de la grasa y vientres musculares. Manifestaciones oculares asociadas: retracción palpebral bilateral que permite ver la esclera por encima del limbo, disminución de la motilidad palpebral, alteración de la pigmentación palpebral, hiperemia conjuntival. DIAGNÓSTICO. Por los signos clínicos descritos y exploraciones complementarias, como la exoftalmometría (medida de la protrusión ocular), la radiología (aumento de densidad de los tejidos blandos), el engrosamiento del vientre de algunos músculos extraoculares (apreciados en la TC, la RM y la ecografía orbitaria) y la analítica sistémica

Page 9: Examen final 20460

AACE Thyroid Task Force. American Association of Clinical Endocrinologists medical guidelines for clinical practice fo rthe evaluation and treatment of hyperthyroidism and hypothyroidism. Endocr Pract. 2002;8(6).

Davies TF, Larsen PR. Thyrotoxicosis. In: Kronenberg HM, Melmed S, Polonsky KS, Larsen PR, eds. Williams Textbook of Endocrinology. 11th ed. Philadelphia, Pa: Saunders Elsevier; 2008:chap 11.

7.- Masculino de 5 años es llevado al servicio de urgencias por dolor súbito hiperemia, y aumento de volumen en región interna de órbita derecha. Exploración Física: Se palpa masa bien delimitada dolorosa en el borde interno de la órbita. La medida terapéutica inicial en este paciente es:

a) Sólo drenaje. b) Dicloxacilina y drenaje. c) Tobramicina tópico y drenaje. d) Extirpación de saco lagrimal

El tratamiento debe iniciarse antes de que se identifique el microorganismo causal. Tan pronto se obtienen cultivos de nariz, conjuntivas y sangre deben administrarse antibióticos. La terapéutica antibiótica inicial debe cubrir estafilococos, H influenzae y anaerobios. La mayor parte de los casos responde a los antibióticos. Aquellos en los cuales esto no es así, pueden requerir drenaje. Riordan P,Witcher j, Oftalmología general de Vaughan y Asbury, 17ª Ediciión, Pág. 262

8.- Paciente diagnosticado con esclerosis múltiple (considerada como un proceso inflamatorio y desmielinizante del SNC). En el diagnóstico de esta enfermedad es muy útil la presencia de: a) Más de 100 liníocitos por microlitro en el líquido cefalorraquídeo. b) Más de 50 polimorfonucleares por microlitro en el líquido cefalorraquídeo. c) Elevaciones del ácido úrico en plasma. d) Bandas oligoclonales en el líquido cefalorraquídeo.

La esclerosis múltiple (EM) es una enfermedad desmielinizante del sistema tema nervioso central, que habitualmente se caracteriza por ataques recurrentes de disfunción neurológica focal y multifocal.

Los síntomas y signos clásicos de EM dependen de la localización del foco de desmielinización y pueden incluir una variedad de disfunciones que no son específicas para la EM, tales como alteración de la visión, ataxia y temblor intencional, debilidad o parálisis de una o más extremidades, espasticidad y problemas vesicales. Hay,sin embargo, tres anormalidades, altamente sugestivas de EM:

Neuritis óptica, la cual es el síntoma inicial en cerca del 25% de los pacientes

Page 10: Examen final 20460

Oftalmoplejia internuclear que se asocia con nistagmus monoocular Signo de Lhermitte, que es una sensación "eléctrica" por detrás del cuello y de la

espalda que llega hasta las piernas (al flexionar el cuello).

En fibras nerviosas normales mielinizadas, la conducción ocurre de manera saltatoria; las corrientes de acción se confinan a las secciones no mielinizadas del axón saltando de un nódulo de Ranvier al siguiente. Este tipo de conducción es mucho más eficiente desde el punto de vista energético que la transmisión a través de la superficie del axón entero, aumentando así la velocidad de conducción con una pérdida mínima de energía.

La desmielinización de una porción de la fibra nerviosa normalmente mielinizada, puede conducir a: bloqueo de la conducción en el sitio de la pérdida de mielina, disminución de la velocidad de conducción nerviosa a través de la fibra alterada y fatiga subjetiva aumentada o un mayor consumo de energía.

La disfunción neurológica observada en la EM es una reflexión de la alteración de la conducción a través de segmentos parcial o completamente desmielinizados o de una fibra nerviosa mielínica. Además, el hecho de que el tiempo de conducción a través de los segmentos desmielinizados disminuye con aumento de la temperatura puede explicar por qué los síntomas clínicos de EM se empeoran al aumentar la temperatura corporal.

Diagnóstico

El diagnóstico de EM se basa fundamentalmente en dos parámetros. Por un lado los hallazgos en los estudios de imagen por Resonancia Magnética en los que se presentan (en las imágenes dependientes de T2) áreas hiperintensas de localización fundamentalmente periventricular en la substancia blanca subcortical en múltiples localizaciones así como también en la médula espinal; este estudio permite visualizar no solamente la localización de las áreas desmielinizantes sino también en forma seriada pueden realizarse evaluaciones para valorar la evolución clínica y subclínica de la enfermedad. Los otros estudios básicos para el diagnóstico son la determinación de bandas oligoclonales, la determinación de la proteína básica de la mielina y la medición de las inmunoglobulinas en el líquido cefalorraquídeo. Estos dos factores, en conjunto con la información clínica, permiten establecer con ciertas bases de seguridad el diagnóstico de esta enfermedad.

Referencias bibliográficas:

1. Noseworthy JH, Lucchinetti C, Rodríguez M, Weinshenker BJ. Multiple sclerosis. N. England J Med. 2000;343:938-52.

2. Omari KM, John GR, Sealfon SC, Raine CS. CXC chemokine receptors on human oligodendrocytes: implications for multiple sclerosis. Brain 2005;128:1003-15.

3. Mi S, Miller RH, Lee X, et al. LINGO-1 negatively regulates myelination by oligodendrocytes. Nat Neurosci. 2005;8:745-51.

4. John GR, Shankar SL, Shafit-Zagardo B, et al . Multiple sclerosis: re-expression of a developmental pathway that restricts remyelination. Nat Med. 2002;8:1115-21.

5. Lucchinetti C. The spectrum of idiopathic inflammatory demyelinanting disease. In: American Academy of Neurology. Syllabi on CD ROM. 2000.

6. Hartung HP, Grossman RI. ADEM: distinct disease or part of the MS spectrum? Neurology 2001; 56:1257-60.

7. Capello E, Voskuhl RR, and McFarland HF, Raine CS. Multiple sclerosis: re-expression of a developmental gene in chronic lesions correlates with remyelination. Ann Neurol. 1997; 41:797-805.

8. Atlanta. Georgia. AAN. 115 Anual Meeting. Natural Course of Multiple Sclerosis Redefined: National Inst of Neurological Disorder and Stroke, 16, 1990.

9. Hemmer B, Archelos JJ, Hartung HP. New concepts in the inmunopathogenesis of MS. Nat Rev Neurosci. 2002;3:291-301.

Page 11: Examen final 20460

Kurtzke JF. Geography in multiple sclerosis. J Neurol. 1977;215:1-26. 9.- Una mujer de 66 años presenta deterioro al caminar. En la exploración se encuentra marcha ligeramente espástica, mala posición y sensación de vibración en los dedos de los pies, reflejos de estiramiento muscular en las rodillas +++ y reflejos aquíleos ausentes. De los siguientes diagnósticos el más probable ES: a) Hipovitaminosis B12 b) Esclerosis múltiple c) Hidrocefalia con presión normal (NPH) d) Infección con virus linfotrópico humano de células T tipo I (HTLV-I) La hipovitaminosis B12 causa degeneración combinada de sistemas. Los pacientes presentan alteración de la marcha caracterizada por espasticidad y disminución de la sensación vibratoria y la posición. Puede haber neuropatía leve que produce depresión de los reflejos aquíleos. Como es una causa tratable de marcha anormal, es importante identificar la deficiencia de vitan B12. La esclerosis múltiple (MS) no se manifiesta en este grupo de edad. La ausencia de reflejos aquíleos es un dato común de MS; más bien, la hiperactividad de los reflejos concuerda con los datos de neurona motora superior encontrados a menudo. La hidrocefalia de presión normal (NPH) es una causa de deterioro de la marcha. Sin embargo, hay donación cognitiva y los pacientes pueden tener incontinencia urinaria. La alteración de la vibración y la sensación de posición, así como la ausencia de reflejos aquíleos, no son característicos de NPH. La infección virus linfotrópico de células T humano tipo I (HTLV-I) es causa de mielopatía y debe sospecharse en pacientes que han recibido transfusiones de sangre, usuarios de drogas intravenosas o personas que hayan habitado en áreas endémicas. La adrenomieloneuropatía es un trastorno recesivo ligado a X que guarda relación con adrenoleucodistrofia, la cual típicamente se presenta en varones jóvenes. El trastorno origina la acumulación de ácidos grasos de cadena muy larga debido a una imposibilidad para catabolizar estos lípidos. Las mujeres portadoras manifiestan una paraparesia espástica leve, pero no la gama de datos que se encuentran en esta paciente.

BIBLIOGRAFíA:

Braunwald, Isselbachre, Petersdorf, Wilson, Martin, Fauci

“HARRISON: PRINCIPIOS DE MEDICINA INTERNA”

Undécima edición D.S.Mc Laren, M.Frigg 10.- Paciente de 79 años que consulta por una mácula pigmentada heterocroma, de contorno irregular, de unos 2 x 3 cm de diámetro, localizada en mejilla derecha, que ha experimentado un crecimiento muy lento en los últimos años. El diagnóstico más probable de este paciente es: a) Melanoma lentiginoso acral. b) Carcinoma Basocelular pigmentado. c) Eritema fijo pigmentario. d) Lentigo maligno.

Page 12: Examen final 20460

El lentigo maligno puede definirse como una mácula pigmentada extensa localizada en una zona de la piel expuesta al daño actínico, habitualmente desarrollada en ancianos y que consiste en una proliferación de melanocitos atípicos situados sobre una epidermis atrófica. La importancia del lentigo maligno reside en que puede ser el sustrato de un melanoma maligno. El lentigo maligno también ha sido denominado, lentigo de Hutchinson, lentigo senil, melanosis precancerosa circunscrita, lentigo “malin des viellards ”, melanosis premaligna, melanosis circunscrita preblastomatosa y melanocitoma no nevoide precanceroso. El lentigo maligno puede ser considerado como la única variedad de displasia melanocítica intraepidérmica capaz de evolucionar a un melanoma invasivo. El porcentaje de melanomas que se suponen derivados de un lentigo maligno es pequeño, 2.5 a 5%. El lentigo maligno es fácilmente reconocible y habitualmente tratable en las fases más precoces de su desarrollo. El lentigo maligno fue descrito en 1892 por Hutchinson. Dubreuilh remarcó su naturaleza precancerosa en 1894. Mishima determinó que el precursor histogénico del lentigo maligno es diferente al precursor del nevus juncional descrito por Allen y Spitz. La histopatología macroscópica y microscópica del lentigo maligno fue documentada por Whyte y Helwig (1) y por Clark y Mihm.

Lentigo maligno 01. Dermatoscopia. Pigmentación asimétrica de las salidas foliculares (flechas pequeñas), estructura romboidal (flecha grande), oclusión de las salidas foliculares (círculo) y puntos azul-gris (asterisco) Lentigo Maligno Cox NH, Aitchison TC, Sirel JM, Mackie RM. Comparison between lentigo maligna melanoma and other hystogenetic types of malignant melanoma of the head and neck. Br J Cancer 1996; 73:940-4. Cohen LM, McCall MW, Zax RH. Mohs micrographic surgery for lentigo maligna and lentigo maligna melanoma. Dermatol Surg 1998; 24:673-77 11.- Femenino de 26 años su padecimiento actual inició hace 4 años, cuando sus ciclos menstruales empezaron a ser irregulares. Su ritmo actual es de 40-90 x 3-4. FUR: hace 3 meses. Inició vida sexual a los 23 años y no ha podido embarazarse. En la exploración física encontramos la piel ligeramente seca, hay salida de líquido blanquecino escaso a la expresión del pezón izquierdo con presencia de giordano positivo derecho.

El estudio que es de mayor utilidad para aclarar el diagnóstico es:

a) prueba de embarazo b) LH, FSH y estrógenos c) tomografía de cráneo d) prolactina

Page 13: Examen final 20460

En toda paciente con galactorrea, trastornos menstruales, hirsutismo, disminución de la libido o infertilidad, deben determinarse los niveles de PRL plasmática, entre el tercer y quinto día del ciclo menstrual, si este es regular o en cualquier momento, si presenta amenorrea u oligomenorrea.2 Si en la primera determinación se obtienen cifras elevadas debe repetirse y si se confirma nuevamente, se establece el diagnóstico de hiperprolactinemia. Una historia clínica com-pleta, con un interrogatorio y examen físico detallados, permitirá orientarnos hacia la causa de la hiperprolactinemia. La primera causa que se debe descartar antes de emprender otros estudios, es el embarazo. El uso de estrógenos, anticonceptivos orales y drogas que aumentan la secreción de PRL debe precisarse en el interrogatorio, así como la presencia de quemaduras u otras lesiones en tórax que se buscan en el examen físico.

Pocas mediciones hormonales tienen el significado clínico que se observa con la prolactina. La técnica está bien estandarizada y presenta bastante confiabilidad; la muestra de sangre se puede obtener a cualquier hora del día y debido a las variaciones fisiológicas (efecto del ejercicio, alimentos, irritación de la pared costal) cuando se obtiene una cifra de prolactina por arriba de lo normal, es necesario repetir el análisis. Varios fármacos pueden producir una moderada elevación en los niveles de prolactina, sin alcanzar los valores que se encuentran en prolactinoma; las drogas más comunes son metoclopramida, fenotiazinas, risperidona, verapamil,metildopa, reserpina y los inhibidores de MAO. Exámenes: prolactina plasmática (prolactinemia), tirotropina (hormona estimulante de la glándula tiroides o TSH), test de embarazo. Prolactina menor de 100 ng/ml: baja sospecha de prolactinoma. Prolactina entre 100 y 300 ng/ml: mayor posibilidad de prolactinoma. Prolactina superior a 300 ng/ml: alta sospecha de prolactinoma. Es precisa resonancia magnética nuclear (RNM) de hipófisis (detecta prolactinoma mayor de 3mm). . En el 50% de los casos de hiperprolactinemia se detecta tumor: microadenomas o macroadenomas; la prolactina suele encontrarse entre 100 y 200ng/ml; los macroadenomas tienen efecto de masa y causan hipopituitarismo; los microadenomas no presentan efecto de masa y sólo producen hipogonadismo. Referencias 1. Schlechte JA. Prolactinoma. N Engl J Med 2003;349:2035-2041. 2. Zárate A, Canales ES, Jacobs LS, Soria J, Daughaday WH. Restoration of ovarian function in patients with the amenorrhea-galactorrhea syndrome after long-term therapy with L-Dopa. Fertil Steril 1973;24:340. 3. Tyson JE, Carter JN, Andreassen B, Huth J, Smith B. Nursing mediated prolactin and luteinizing hormone secretion during puerperal lactation. Fertil Steril 1978;30:154. 4. Schlechte JA, Sherman BM, Chapler FK, VanGilder J. Long-term followup of women with surgically treated prolactin-secreting pituitary tumors. J Clin Endocrinol Metab 1986;62:1296-301. 5. Losa M, Mortini P, Barzaghi R, Gioia L, Giovanelli M. Surgical treatment of prolactin-secreting pituitary adenomas: early results and long-term outcome. J Clin Endocrinol Metab 2002;87:3180-3186.

Page 14: Examen final 20460

12.- Es una de las enfermedades tiroideas autoinmunes organo-específica más frecuentes. Es más común en la mujer, posee una asociación directa con otras enfermedades autoinmunes y, por lo general, se presenta con hipertiroidismo, bocio difuso, oftalmopatía, además, en algunos casos coincide con mixedema pretibial:

a) Carcinoma basocelular b) Enfermedad de Graves c) Adenoma tóxico d) Síndrome del eutiroideo enferm

Introducción

La enfermedad de Graves-Basedow (EGB) constituye la entidad más importante, por su frecuencia, entre las enfermedades que producen hipertiroidismo. Se caracteriza por la presentación de la tríada sintomática constituida por hipertiroidismo, bocio difuso y oftalmopatía (50%). Es más frecuente en la 3ª y 4ª década de la vida y afecta con más frecuencia a mujeres (7/1 en zonas no bociógenas y 3/1 a partir de los 45 años en zonas de bocio endémico) 1.

Etiopatogenia

Aunque no se conocen exactamente los factores que inician y mantienen la enfermedad, hay elementos que permiten encuadrarla en el marco de la patología autoinmunitaria, como la presencia de autoanticuerpos antitiroglobulina y antitiroperoxidasas (TPO), así como la presencia de inmunoglobulinas antireceptor de la TSH (TRAb) que son estimuladoras de la función y del crecimiento del tiroides.

Para el desarrollo de la enfermedad parece que es necesaria la intervención de factores genéticos y ambientales (exceso de Yodo) 2.

Clínica

La enfermedad se caracteriza por la presencia de síntomas de hipertiroidismo (nerviosismo 99%, sudoración 91%, intolerancia al calor y palpitaciones 89%, pérdida de peso 85%, aumento de apetito 69% y síntomas oculares 55%) y signos clínicos (aumento uniforme del tamaño de la glándula 97%, oftalmopatía 60%, taquicardia en reposo 90%, temblor 70% y retracción palpebral 38%) relacionados con el aumento de receptores adrenérgicos que determinan las hormonas tiroideas 1,2.

La historia natural de la enfermedad se caracteriza por fases cíclicas de exacerbación y remisión, de duración y presentación imprevistas; aunque, hoy en día, está artefactada por los tratamientos utilizados. En aproximadamente el 25% de los pacientes, especialmente en aquellos con una forma leve de enfermedad, el proceso se autolimita al año o más, regresando espontáneamente a un estado eutiroideo 3.

En cuanto a la oftalmopatía, hay una forma no infiltrativa o pálpebro-retráctil (afecta más frecuentemente al sexo femenino, afecta simétricamente a ambos ojos y que evoluciona paralelamente al hipertiroidismo) y una forma infiltrativa (infiltración del tejido retroorbitario por mucopolisacáridos y linfocitaria con protusión ocular asimétrica)1.

Otras manifestaciones menos frecuentes son la aparición de mixedema pretibial o dermopatía infiltrativa y, la acropaquia tiroidea 1.

Referencias

1.- Foz M. Enfermedades del tiroides. En Rozman C (ed): Farreras-Rozman Medicina interna. Editorial Doyma. Barcelona, 1992:1997.

Page 15: Examen final 20460

2.- Wartofsky L. Diseases of the thyroid. En Braunwald E, Isselbacher KJ, Wilson JD, Martin JB, Fauci AS, Kasper DL (eds): Harrison’s principles of internal medicine. Editorial McGraw-Hill. EEUU, 1994: 1930.

3.- De Groot. Graves’ diseases and the manifestations of thyrotoxicosis. En De Groot LJ, Reed Larsen P, Hennemann G.(eds): The thyroid and its diseases. 1996: 371.

4.- Haynes RC, Murad F. Drogas tiroideas y antitiroideas. En Goodman LS, Gilman A (eds): Las Bases farmacológicas de la terapéutica. Editorial Panamericana. México, 1982: 1376.

5. - Feliciano DV. Everything you wanted to know about Graves’ disease. Am J Surg 1992, (164): 404. 13.- Masculino de 42 años con antecedentes de litiasis renal cálcica bilateral recidivante. Acude Con resultados de laboratorio los cuales demuestran una calcemia de 11.1 mg%, fosfatemia de 2 mg, calciuria de 280 mg/24h y fosfaturia de 1600 mg/24h, con una reabsorción tubular de fósforo (RTP) del 70%. La sospecha más probable del origen de ésta litiasis es:

a) Acidosis tubular b) Intoxicación vitamina D c) Hiperparatiroidismo primario d) Sarcoidosis

Exploración y pruebas complementarias.

El examen clínico es muy pobre. Excepcionalmente se puede palpar una tumoración cervical en el 4% de los casos. Son los síntomas lo que hacer evocar el diagnóstico y aconsejan determinar la calcemia. Calcemia: su determinación es el elemento fundamental del diagnóstico y se aconseja repetirla varias veces hasta comprobar su existencia, ya que puede ser constante o intermitente por circunstancias como la deshidratación o determinados fármacos pueden causar un aumento transitorio de la misma.

HIPERCALCEMIA

Es el aumento del calcio sérico por encima de 10,5 mg/dl. Cerca de la mitad de las hipercalcemias son “falsas hipercalcemias”, por extracción sanguínea en condiciones no ideales (postpandriales, torniquetes prolongados...) o por hiperalbuminemia que condiciona “pseudohipercalcemia”, por lo que ante todo paciente con hipercalcemia debemos en primer lugar calcular la calcemia según las proteínas totales (calciocorregido Ca2+) en base a la siguiente fórmula:

Page 16: Examen final 20460

Junto al calcio se ha determinar la albúmina pues si esta es menor de cuatro habrá que corregir convenientemente la cifra del calcio. Como se ha expuesto, la etiología más frecuente de las hipercalcemias es el hiperparatiroidismo primario. Los valores sumamente altos suelen asociarse a los infrecuentes carcinomas paratiroideos, ello es debido a que el tumor produce una proteína parecida a la hormona paratiroidea que ha sido detectada en el suero y que se denomina proteína relacionada con la hormona paratiroidea. Se aconseja investigar esta proteína en los pacientes con una aparición reciente de hipercalcemia y valores normales de PTH, no obstante el hiperparatiroidismo normocalcémico es una posibilidad rara, si no existe insuficiencia renal, ni deficiencia de vitamina D. A pesar de ello pueden darse situaciones confusas de hiperparatiroidismo con calcemia en ayunas normal, en estos casos puede ser útil comprobar la hipercalcemia postabsortiva, o realizar una prueba de provocación con benzotiacidas.

La fosforemía generalmente esta disminuida. La hipofosfatemia es un dato diagnóstico menos específico que la hipercalcemia.

En el hiperparatiroidismo es frecuente la hipercalciuria, pero menos que en otros estados de hipercalcemia. La calciuria está elevada siendo superior a 200 mg/24 h, si bien sus valores oscilan en función del aporte de calcio, así cuando es bajo, la calciuria se sitúa entre 50 y 150 mh/24 h; al aumentar el aporte, los valores de calciuria oscilan entre 100 y 300 mg/24h..La determinación de la calciuria se realiza cuando las cifras de calcio y PTH en suero están elevadas. Esta determinación es necesaria para descartar una HHF antes de la exploración quirúrgica, así como para decidir cuando intervenir a un paciente con hiperparatiroidismo primario asintomático

La fosfaturia está elevada, siendo superior a 800 mg/24 h. El aclaramiento de fósforo unido a la creatinina está elevado, siendo superior a 12 ml/mn

Las fosfatasas alcalinas esta elevadas en las formas con signos óseos así como la citremia (por encima de 26 mg/dl) y la hidroxipolinuria. Puede aparecer con frecuencia una acidosis metabólica con hipercloremia.

Determinación de la PTH: es determinante. La tríada analítica clásica diagnóstica del hiperparatiroidismo primario incluye: hipercalcemia, hiper-PTH e hipofosforemia.

También su pueden realizar pruebas de frenaje por perfusión cálcica y una prueba de estimulación por hipocalcemia provocada por medio de EDTA sódico, esta prueba aporta pocos datos útiles debido a su dificultad de interpretación.

Page 17: Examen final 20460

ALTERACIONES DEL METABOLISMO DEL CALCIO

AUTORES:

Carlos López Vargas

Coral Suero Méndez

José Mª Benavente Ramos

14.- Se trata de paciente masculino de 14 años de edad higiénica dietética, convivencia con aves, gatos e ingestión de berros. Inicia su padecimiento actual hace dos semanas con fiebre de 39ºC sin predominio de horario, náusea y dolor en hipocondrio derecho. A la exploración física con palidez generalizada y hepatomegalia 3-3-5. Resto de la exploración sin datos patológicos. Laboratorio con BH, con Hb de 11.5, Hto de 40, leucocitos; 16,500, eosinófilos; 38%, linfocitos; 30%, formas inmaduras; 4, plaquetas; 270,000, Fosfatasa alcalina; 280 UI/L. El diagnóstico más probable ES:

a) Larva migrans visceral

b) Toxoplasmosis generalizada

c) Fasciolosis

d) Gnatostomiasis

Definición

La Fasciolosis es una zoonosis parasitaria causada por la Fasciola hepática que ocasiona patología y sintomatología hepato-biliar.

Clínica Se considera las siguientes formas de presentación clínica:

a) Sintomática: Aguda o invasiva, crónica o de localización y extrahepática. 1. Aguda o invasiva: Hay tres elementos esenciales a identificar: hepatomegalia dolorosa, fiebre y eosinofilia con cifras que superan frecuentemente el 30-40%. 2. Crónica o de localización: La sintomatología y signología corresponden a padecimiento

crónico hepato biliar incluyendo cólicos biliares y litiasis biliar. 3. Extrahepática: Incluye nódulos subcutáneos en el hipocondrio derecho, seno derecho, escápula derecha con poco dolor local y signos inflamatorios. Eosinofilia alta. b) Asintomática: En algunas personas los síntomas o signos suelen pasar desapercibidos.

Diagnóstico de Laboratorio

Page 18: Examen final 20460

En la forma aguda, la búsqueda de huevos en las deposiciones es inútil, ya que las formas juveniles están en el tejido hepático, por lo tanto las pruebas inmunobiológicas son importantes. Son útiles la inmunoelectroforesis o inmunodifusión buscando el arco 2 de Caprón (4). El inmunoblot o westernblot tiene buena sensibilidad y especificidad (5,6). Se han identificado fracciones antigénicas en las cistenilproteasas de F. hepatica (7). En las formas crónicas, la búsqueda de huevos en heces es lo indicado. Son útiles la sedimentación rápida de Lumbreras (8). Recientemente se han preparado anticuerpos monoclonales contra el parásito y ello ha permitido elaborar la técnica de ELISA para detectar los coproantígenos (E/S) del parásito en heces. La ecografía de vías biliares (v.b.) puede detectar al parásito moviéndose en las v.b. o vesícula. En formas extrahepáticas, la eosinofilia alta es orientadora; F. hepatica en las biopsias confirma el diagnóstico.

15.- Ante un paciente con diagnóstico de trastorno obsesivo-compulsivo, usted decide indicar el siguiente fármaco por ser el principal tratamiento de elección: a) Antipsicóticos. b) Antidepresivos inhibidores de la recaptación de noradrenalina. c) Antidepresivos inhibidores de la recaptación de serotonina. d) Benzodiacepinas. El abordaje farmacológico del TOC se basa en los inhibidores selectivos de la recaptura de serotonina (ISRS), medicamentos que han resultado ser efectivos y seguros. Como grupo, los ISRS son igual de eficaces que la clorimipramina, pero producen menos efectos secundarios y, por lo tanto, mejor tolerancia y mejor apego al tratamiento. La efectividad antiobsesiva parece ser independiente de su actividad antidepresiva. Entre los factores predictores de respuesta al tratameinto, se ha propuesto que los altos puntajes en las compulsiones predicen una mala respuesta al tratamiento. Para los casos refractarios y resistentes o cuando hay síntomas de comorbilidad, se han utilizado, con relativo éxito, combinaciones con diferentes IRS, con benzodiacepinas, o bien, con potenciadores como el litio, o antipsicóticos, como la risperidona y el haloperidol. El uso del carbonato de litio es controvertido, aunque parece ser útil como potenciador a largo plazo entre 15 y 30% de los pacientes. La combinación de ISRS con antipsicóticos comenzó a utilizarse en los pacientes con síntomas psicóticos, aunque ahora se combinan también en los pacientes resistentes. Tratamiento farmacológico del TOC Cristina Lóyzaga* Humberto Nicolini* *División de Investigaciones Clínicas. Instituto Nacional de Psiquiatría. Ramón de la Fuente. Calzada México-Xochimilco, 101, San Lorenzo Huipulco, 14370, México D.F. Primera versión: 14 de septiembre de 2000. Aceptado: 26 de septiembre de 2000. 16.- Masculino de 44 años, con antecedentes de mialgias, artralgias de grandes articulaciones, fatiga y pérdida ponderal desde 5 meses. Hace 2 semanas presentó febrícula y 10 días después orina oscura y tos seca. Dentro de sus antecedentes menciona infecciones recurrentes de los oídos. A la exploración física cuenta con 37.2 °C, frecuencia cardiaca de 110 por minuto, frecuencia respiratoria de 18 por minuto, tensión arterial de 130/80 mm Hg, oídos con cicatrices en ambos tímpanos, tórax con estertores en ambas bases, dolor costovertebral y en extremidades hemorragia periungueal en los dedos. Laboratorio: leucocitosis de 13,200. Hepatocitos: 35 %, VSG 55 mm/h, BUN 49 mg/dl, creatinina 4.0 mg/dl, serología con factor reumatoide 1/40, VDRL negativo, anticuerpos citoplasmáticos antineutrófilos positivos, sedimento urinario con cilindros hemáticos-gabinete: Rx con infiltrados alveolares bilaterales.

Page 19: Examen final 20460

El diagnóstico más probable de este paciente es. a) Poliarteritis. b) Polimialgia reumática. c) Vasculitis probablemente granulomatosa de Wegener. d) Púrpura de Henoch-Schonlein. La Púrpura de Schönlein-Henoch es un síndrome vasculítico sistémico, de pequeños vasos, con afectación primaria de la piel, articulaciones, tuvo digestivo y riñones (1-11). Es la causa más frecuente de púrpura no trombocitopénica y vasculitis en niños. (1) (9) Se postula que la etiopatogenia puede ser debida a una alteración inmunológica, pues en esta enfermedad se describen una gran variedad de anormalidades de la IgA, así como niveles elevados de citosina, FNT alfa e IL-6. Se plantea una asociación entre esta respuesta inmunitaria y una causa infecciosa, en la literatura se describen casos desencadenados por Estreptococo sp, virus de la hepatitis A y C, Salmonella sp, Mycoplasma, virus de Epstein Barr, varicela, parvovirus B19, adenovirus, parainfluenza, campylobacter y yersinia entre otros, también se invocan determinados fármacos, alimentos, vacunas, exposición al frío y picaduras de insectos. (1, 3, 9) Parece ser que la afinidad de los gérmenes por el endotelio y los hematíes se debe a un esfingofosfolípido neutro en la pared de estas células y su efecto citotóxico está mediado por una proteína no estructural del germen, que induce la apoptosis mediante la ruta del TNF. Este efecto puede explicar el daño selectivo hacia los capilares característicos de la PSH, y hacer que las células sean más susceptibles al depósito de IgA que sufren. (9) Los hallazgos histopatológicos son los típicos de la vasculitis leucocitoclástica ligados patogénicamente al depósito de inmunocomplejos, activación del complemento, quimiotaxis de neutrófilos, con daño secundario de la pared vascular debido a la liberación de enzimas por parte de los neutrófilos activados. (1, 9, 12, 13) El diagnóstico se basa fundamentalmente en las manifestaciones clínicas. (1, 2) 17.- Femenino de 18 meses, originário de zona rural del estado de Guerrero, es traído a consulta por cojera desde que empezó a caminar (a los 16 meses). A la exploración presenta marcha en Trendelenburg. La primera sospecha diagnóstica es:

a) Sinovitis transitoria de cadera. b) Enfermedad de Perthes. c) Luxación congénita de cadera. d) Artritis séptica de cadera. .

La falta de relación normal en las estructuras que forman una articulación se conoce como luxación. En el caso de la cadera, la cabeza del fémur no encaja debidamente en su cavidad (acetábulo). El desarrollo del acetábulo será normal siempre que la cabeza del fémur permanezca en posición correcta y el esfuerzo muscular sea adecuado. Siempre que esto no se cumpla, se producirá una alteración entre el cótilo y la cabeza femoral, dando lugar a una alteración con el resultado de luxación de la cadera. FACTORES DE RIESGO El factor principal en cuanto a la estructura es la falta de estabilidad en la articulación debido a la laxitud de la cápsula y del ligamento redondo. Entre los factores genéticos más destacados estaría pertenecer a una familia con antecedentes, factor que se observa en ciertas razas, (en relación con una laxitud ligamentosa familiar).

Page 20: Examen final 20460

La frecuencia es mayor en las niñas que en los niños, factor que se incrementa más cuando la presentación del feto es de nalgas completas. La posición del niño cuando ha nacido también es determinante, siempre que la posición sea estable se reduce el riesgo de luxación. DIAGNÓSTICO: EXPLORACIÓN FÍSICA a) Maniobras de Ortolani y Barlow - click audible-. Ortolani: el click se produce cuando la fóvea de la cabeza del fémur encuentra la cresta cotiloidea caminando sobre ella. La limitación a la abducción desaparece, esto es signo de reducción. Barlow: se realiza la aducción de la cadera, que tras una ligera presión longitudinal sobre el fémur, produce una luxación. La prueba de Ortolani traduce una luxación y la de Barlow una cadera luxable. b) Asimetría de pliegues -no valorable en luxaciones bilaterales-. c) Limitación a la abducción. d) Dismetría La marcha de Trendelenburg también es llamada marcha en sacudida del glúteo medio o de los abductores. Observamos la marcha de Trendelenburg en pacientes con mala función de los abductores. Si estos músculos se acortan, pierden su ventaja mecánica. El glúteo mediano, que es el principal abductor de la cadera, se origina en la superficie lateral del ilíaco y se inserta en la superficie superior del trocánter mayor. Si una enfermedad de la cadera provoca que el trocánter mayor se localice más proximalmente de lo normal (se mueve más cerca de la masa muscular del glúteo mediano), la contracción del músculo generará una fuerza abductora menor. Son causas de dicho acortamiento: - Luxación congénita de cadera - Coxa vara debida a: * Causa idiopática * Fractura del cuello femoral consolidada con mala alineación * Enfermedad de Perthes * Deslizamiento epifisario * Raquitismo García Portabella, M. Luxación congénita de cadera antes de los tres meses de edad. 2001. Disponible en: http://www.vhebron.es/htr/ortopediatria. • Garcia-Siso Pardo, J. M. Displasia del desarrollo de la cadera. (Parte I). Peditría Rural y Extrahospitalaria. Vol. 32. Nº 304 Págs. 481-491. 2002. • Graf, R. Classification of hip joint dysplasia by means of sonography. Arch Orthop Trauma Surg 102:248-255, 1984. • Edeiken, J. Luxación congénita de cadera. Diagnóstico Radiológico de las Enfermedades de los Huesos. Ed. Médica Panamericana. Págs. 388-392. 18.-The following conditon is commonly seen in the magnesium-amonium-phosphate (struvite) stones: a) Recurrent P. mirabilis infection b) Double J stent placement c) Resorptive hypercalciuria d) Renal azotemia

Page 21: Examen final 20460

Esta bacteria de colonias redondeadas tiene la habilidad de producir grandes niveles de ureasa. La ureasa hidroliza urea a amoníaco, (NH3) y eso hace a la orina más alcalina. Y al subir la alcalinidad puede liderar la formación de cristales de estruvita, carbonato de calcio, y/o apatita. Esta bacteria puede encontrarse en cálculos, y esas bacterias escondidas allí, pueden reiniciar una infección post tratamientos antibióticos. Al desarrollarse los cálculos, después de un tiempo pueden seguir creciendo más y causar obstrucción dando fallas renales.

Proteus también puede producir infecciones de heridas, septicemia y neumonías, sobre todo en pacientes hospitalizados.

Esipov, Sergei E. and J. A. Shapiro (1998). «Kinetic model of Proteus mirabilis swarm colony development». Journal of Mathematical Biology 36 (3). doi 10.1007/s002850050100.

Frénod, Emmanuel (2006). «Existence result for a model of Proteus mirabilis swarm». Differential and integral equations 19 (6): pp. 697-720.

http://arxiv.org/abs/math.FA/0702761.

19.- Femenino de 34 años, es atendida en sala de partos secundario a eutocia, durante la reparación de la episiotomía media hay un marcado incremento en el sangrado transvaginal. La causa más probable de la sintomatología de esta paciente es:

a) Retención de restos placentarios b) Atonía uterina c) Laceración cervical d) Laceración vaginal

Atonía uterina La atonía uterina o inercia, consiste en la falta de contracción del útero gestante luego del alumbramiento. Este fenómeno ocurre en el 2 al 5 % de los partos por vía baja. La atonía uterina es favorecida por numerosas situaciones como: la multiparidad, la sobre distensión uterina, la macrosomía, el hidramnios, los intentos de versión, los antece3 0 M. Pesaresi, J. Palacios Jaraquemada Patología de Urgencia, Año 9, Nro. 3, Septiembre de 2001 dentes de hemorragia pre y posparto, la corioammnionitis y la interrupción en la administración de oxitócicos, entre otros. No ha sido demostrada una relación directa entre las atonías y el uso de los anestésicos administrados por vía peridural. Por el contrario, hay una asociación demostrada con el uso de Halotano como anestésico general. El tratamiento clásico de la atonía uterina está constituído por la reposición volumétrica (sangre, coloides y cristaloides), la utilización de masajes y por la administración de drogas que promueven la contracción del músculo uterino. Cuando estas medidas no son eficientes, se procede casi invariablemente a la histerectomía de hemostasia. Es de notar, que la atonía uterina puede aparecer en el primer embarazo y sin ningún antecedente previo. La atonía uterina constituye, en casi todas las series mundiales, más del 50% de las hemorragias graves del posparto. Patología de Urgencia, Año 9, Nro. 3, Septiembre de 2001 20.- Lactante de 10 meses. Acude al servicio de urgencias por presentar, vómito en 5 ocasiones, fiebre 38.7 ºc, Antecedentes: Acude a guardería. Desde hace 2 días presentó aumento en el número de evacuaciones en promedio de 4 a 5 veces al día, de consistencia líquidas, abundantes, sin moco ni sangre, actualmente ha pesnetado 6 en las últimas 6 hrs. . Previamente sano. Exploración física: Temp. 38. 5, Oc, alerta, sed aumentada bebe con avidez, ojos hundidos, llanto sin lágrimas, mucosa oral seca, fontanela anterior hundida.

Page 22: Examen final 20460

El tratamiento de primera elección para este paciente es:

a) Plan a de hidratacion oral. b) Plan b de hidratacion oral. c) Plan c de hidratacion oral. d) Antibiotico y ayuno.

Las enfermedades diarreicas, de acuerdo con la evaluación del estado de hidratación, se clasifican en: casos sin deshidratación, con deshidratación, con choque hipovolémico por deshidratación. 7.2.3 Caso sin deshidratación, es aquel que presenta generalmente menos de cuatro evacuaciones líquidas en 24 horas, ausencia de vómito, sin signos clínicos de deshidratación. 7.2.4 Caso con deshidratación, es aquel que presenta dos o más de las manifestaciones clínicas siguientes: 7.2.4.1 Inquieto o irritable; 7.2.4.2 Ojos hundidos, llanto sin lágrimas; 7.2.4.3 Boca y lengua secas, saliva espesa; 7.2.4.4 Respiración rápida; 7.2.4.5 Sed aumentada, bebe con avidez; 7.2.4.6 Elasticidad de la piel, mayor o igual a dos segundos; 7.2.4.7 Pulso rápido; 7.2.4.8 Llenado capilar de tres a cinco segundos; 7.2.4.9 Fontanela anterior hundida (lactantes); 7.2.5 Caso con choque hipovolémico, es aquel que presenta dos o más de las manifestaciones clínicas siguientes: 7.2.5.1 Inconsciente o hipotónico; 7.2.5.2 No puede beber; 7.2.5.3 Pulso débil o ausente; 7.2.5.4 Llenado capilar mayor de cinco segundos; 7.2.6 El manejo de los casos de enfermedades diarreicas se basa en tres planes generales de tratamiento: 7.2.6.1 Plan A: Para pacientes con enfermedad diarreica sin deshidratación con atención en el hogar: 7.2.6.1.1 Continuar con la alimentación habitual; 7.2.6.1.2 Aumentar la ingesta de los líquidos de uso regular en el hogar así como Vida Suero Oral: de este último, en los niños menores de un año de edad, ofrecer media taza (75 ml) y en los mayores de un año, una taza (150 ml) y administrarlo a cucharadas o mediante sorbos pequeños, después de cada evacuación. 7.2.6.1.3 Capacitar a la madre para reconocer los signos de deshidratación y otros de alarma por enfermedades diarreicas: (sed intensa, poca ingesta de líquidos y alimentos, numerosas heces líquidas, fiebre, vómito y sangre en las evacuaciones), con el propósito de que acuda nuevamente a solicitar atención médica en forma oportuna. 7.2.6.2 Plan B: Para pacientes con diarrea y deshidratación con atención en la unidad de salud: 7.2.6.2.1 Administrar Vida Suero Oral 100 ml por kilogramo de peso, en dosis fraccionadas cada 30 minutos durante cuatro horas; 7.2.6.2.2 Si el paciente presenta vómito, esperar 10 minutos e intentar otra vez la hidratación oral, más lentamente; 7.2.6.2.3 Al mejorar el estado de hidratación, pasar al Plan A. En caso contrario, repetir el Plan B por otras cuatro horas, de no existir mejoría pasar al Plan C; 7.2.6.2.4 Si los vómitos persisten, existe rechazo al Vida Suero Oral, o gasto fecal elevado (más de 10 g/kg/hora o más de tres evacuaciones por hora) se hidratará con sonda nasogástrica, a razón de 20 a 30 ml de Vida Suero Oral por kilogramo de peso, por hora. 7.2.6.3 Plan C: Para pacientes con choque hipovolémico por deshidratación: 7.2.6.3.1 Inicie inmediatamente administración de líquidos por vía intravenosa, con solución Hartmann; si no se encuentra disponible, use solución salina isotónica al 0.9%, de acuerdo con el siguiente esquema:

Page 23: Examen final 20460

PRIMERA HORA50 ml/kg

SEGUNDA HORA25 ml/kg

TERCERA HORA25 ml/kg

� Evalúe al paciente continuamente. Si no mejora, aumente la velocidad de infusión. � Cuando pueda beber (usualmente en dos a tres horas), administre VSO, a dosis de 25ml/kg/hora; mientras sigue líquidos IV. � Al completar la dosis IV, evalúe al paciente para seleccionar Plan A o B, y retirar venoclisis, orepetir Plan C. � Si selecciona el Plan A, observe durante dos horas para asegurarse de que el responsableencargado del paciente puede mantenerlo hidratado con VSO y además, alimentarlo en sudomicilio.

NORMA Oficial Mexicana NOM-031-SSA2-1999, Para la atención a la salud del niño.

Evaluación del estado de hidratación de un paciente con diarrea17

Signos Plan A Plan B Plan C

Definición Leve o inaparente Moderada o clínica Grave

Pérdida de aguacorporal

Menos de 50 ml/kg peso omenos de 5% del peso

50 a 100 ml/kg peso ó6 a 9% del peso

100 ml/kg peso o más(10% o más del peso)

Condición general

Bien, alerta Irritable Letárgico o inconsciente

Globo ocular Normales Algo hundidos Muy hundidos y secos

Lágrimas Presentes Ausentes Ausentes

Mucosa oral Húmeda Seca Muy seca

Sed Paciente bebe normalmentePaciente bebe conavidez, sediento

Paciente bebe mal o noes capaz de hacerlo

Pliegue cutáneo Vuelve a lo normarápidamente

Se retrae lentamente< 2 s

Se retrae muy lentamente> 2 s

Tratamiento Plan A Plan B Plan C

21.- Masculino de 17 años de edad, con antecedentes de cuadro gripal de tres días de evolución, inicia hace 12 hrs. con dolor en la bolsa escrotal y aumento de volumen, a la exploración física se encuentra edema. tensión y enrojecimiento del escroto y no se puede palpar el testículo derecho. El diagnóstico más probable de este paciente es:

a) Torsión testicular b) Orquiepididimitis c) Hidrocele agudo d) Hernia inguinal estrangulada.

El síndrome escrotal agudo es una urgencia urológica cuyo principal síntoma es el dolor intenso del contenido escrotal, su importancia radica en el diagnóstico precoz de la torsión testicular, cuadro de emergencia quirúrgica, por lo que se ha establecido un síndrome en el que se engloban diversas afecciones intraescrotales cuyo denominador común es el dolor testicular de aparición brusca que puede ir acompañado de otros síntomas dependiendo de su etiología, como tumefacción escrotal, náuseas, vómitos, síndrome miccional... que nos orientarán sobre el diagnóstico.

RECUERDO ANATOMICO

Page 24: Examen final 20460

El escroto es la bolsa que aloja al testículo, éste es una glándula de forma tubular cuya capa externa es la túnica albugínea. Del polo superior del testículo sale el epidídimo que tras descender por la cara posterior de éste, se continúa con el cordón espermático. El cordón espermático está formado por el conducto deferente, los vasos sanguíneos (arterias espermáticas y deferencial y venas espermáticas), linfáticos y nervios que llegan a la glándula. El cordón espermático está envuelto por el músculo cremaster.

La irrigación del testículo procede de la arteria espermática interna, rama de la aorta abdominal y el drenaje venoso se realiza a través de la vena espermática interna que desemboca en la vena cava inferior en el lado derecho y en la vena renal en el lado izquierdo.

Las funciones del testículo son endocrina, las células de Leydig sintetizan y liberan la testosterona y exocrina que es la espermatogénesis.

ORQUITIS Y EPIDIDIMITIS

La inflamación del contenido escrotal plantea un problema de diagnóstico en el escroto agudo principalmente con la torsión funicular. La infección aislada del testículo es rara, siendo más frecuente la presentación en forma de orquiepididimitis. Predomina en hombres jóvenes sexualmente activos y en ancianos con infección urinaria pero se puede producir a cualquier edad. La causa más frecuente de la inflamación del contenido escrotal es la infección. Así en los jóvenes prepuberales los gérmenes más frecuentes son las enterobacterias aunque sobre factores predisponentes como son las malformaciones congénitas (válvulas uretrales, abombamiento ectópico de un uréter a vesícula seminal). En el adolescente es causa frecuente la orquitis en el transcurso de una parotiditis. En los varones jóvenes menores de 40 años es la causa infecciosa con mucho lo más frecuente, sobre todo enterobacterias (E. coli, Proteus Mirabilis, Klebsiella pneumoniae) y gérmenes de transmisión sexual como Neisseria gonorrheae, Mycoplasma, pero sobre todo la Chlamydia tracomatis, y cocos gram positivos como estafilococos aureus. Por encima de los 40 años a la causa infecciosa se suele añadir una obstrucción urinaria distal, tuberculosis, reflujo urinario posterior a la prostatectomía transuretral, y la reacción granulomatosa por el tratamiento del cáncer vesical superficial con bacilo de Calmette Guerin (BCG).

También puede aparecer orquiepididimitis en el curso de la varicela, fiebre tifoidea, mononucleosis infecciosa, rickettsiosis, brucelosis filariasis, actinomicosis, sinusitis, osteomielitis, endocarditis y ser secundaria a cuadros septicémicos por E. coli, Klebsiella, pseudomona, estreptococo y estafilococo. Una causa no infecciosa es el tratamiento con amioradona. Clínicamente aparece dolor escrotal intenso, irradiado a lo largo del cordón espermático por lo que irradia a ingle e incluso a abdomen con tumefacción escrotal no distinguiéndose el epidídimo del testículo, con induración de la piel escrotal que puede producir hidrocele reactivo con síntomas de infección urinaria concomitantes y de secreción uretral, además de afectación del estado general con fiebre que puede llegar a ser de hasta 40 °C, con escalofríos, acompañado de náuseas y vómitos.

En la exploración física, el escroto se observa edematoso, tenso y enrojecido, el cordón se palpa engrosado y doloroso, siendo muy dolorosa la palpación, y difícil de distinguir el límite entre escroto y epidídimo en casos evolucionados. El signo de Prehn, al elevar el testículo afecto produce mejoría del dolor, es positivo. La transiluminación del escroto es negativa a no ser que exista un hidrocele reactivo, en tal caso será positiva. En cuanto a los estudios complementarios habrá que realizar analítica de orina, cultivo de orina, y de semen y de secreción uretral si la hubiere, cultivo de Lowenstein y tinción de Ziehl-Nielsen tanto en orina como en semen e inmunofluorescencia para Chlamydia, además de antibiograma. También es necesario realizar hemograma, hemocultivos seriados, una bioquímica sanguínea que incluya urea, creatinina e iones.

Page 25: Examen final 20460

En el sedimento de orina nos mostrará piuria en la mayoría de las ocasiones, discreta piuria sin bacteriemia en las ETS y piuria franca con urocultivo positivo en caso de gérmenes gram negativos, aunque también podremos encontrar a pacientes con sedimento urinario negativo. En el hemograma es frecuente leucocitosis con desviación a la izquierda. La ecografía nos mostrará engrosamiento y edema de las túnicas escrotales, pudiendo haber o no hidrocele, con epidídimo engrosado (patrón heterogénico), y un testículo aumentado de tamaño (patrón hipoecogénico global) o como una zona hipoecogénica perihiliar. La eco-dopopler está indicada cuando hay dudas sobre el diagnóstico diferencial con la torsión funicular, siendo en estos casos muy útil, y en la orquiepididimitis encontraremos una hipervascularización.

La gammagrafía está indicada para el diagnóstico diferencial con procesos isquémicos y neoplásicos, es un método de gran sensibilidad y especificidad, pero que no se tiene siempre disponible en el área de urgencias. Se observará un aumento de la captación a nivel testicular debido a la hiperemia. Si después de todas las pruebas complementarias existen dudas sobre el diagnóstico, será necesaria la cirugía para llegar al diagnóstico definitivo.

Referencias 1. Young Y., Miller R Incidence of malignant tumours in U.S. children. J Pediatr 1975; 86: 254-258. 2. Reiter A., Schrappe M. Chemotherapy in 998 unselected childhood acute lymphoblastic leukemia patients. Results and conclusions of the multicenter trial ALL-BFM 86. Blood 1994; 84: 3122-3133. 3. Schorin M., Blattner S., Gelber R., et al. Treatment of childhood acute lymphoblastic leukemia: Results of Dana Farber Cancer Institute/Children s Hospital acute lymphoblastic leukemia consortium protocol 85-01. J Clin Oncol 1994; 12: 740-747. 22.-Se trata de escolar de 10 años con los siguientes antecedentes: a los 6 años y en varios análisis se detecta hematuria microscópica, con niveles IgA normales y normocalciuria. A la edad de 9 años persistía la hematuria en los análisis e incluso habían observado algún episodio recortado de hematuria macroscópica. Un año más tarde se detectó proteinuria moderada de 1250 mg/24 horas. En el momento de la consulta persisten las alteraciones en el sedimiento, pero la proteinuria es de rango nefrótico, con creatinina sérica de 1,3 mg/dl. Existen antecedentes familiares de nefropatía evolutiva con desarrollo de insuficiencia renal y de miopía familiar por “lenticonus”. El diagnóstico más probable es del menor es: a) Glomerulonefritis mesangial (enfermedad de Berger). b) Enfermedad poliquística autosómica dominante. c) Nefritis intersticial por hipersensibilidad. d) Enfermedad de Alport

En medicina, la enfermedad de Alport es una enfermedad genética ligada al cromosoma X, en la que una mutación en la síntesis del colágeno afecta los riñones, oídos, y ojos causando sordera y trastornos de la vista, incluyendo distrofia de la córnea y cataratas. Fue inicialmente identificado por el médico británico Cecil A. Alport en 1927, que describió una familia británica en la que muchos miembros desarrollaban enfermedades renales. Él describió que los hombres afectados en la familia morían a causa de enfermedades renales, mientras que las mujeres estaban menos afectadas.

Page 26: Examen final 20460

Se conoce ahora que muchos casos del síndrome de Alport son causados por una mutación en el gen del colágeno COL4A5, entre otras. Este gen codifica la cadena alfa-5 del colágeno tipo IV y está localizado en el cromosoma X. En ciertas familias se ven plaquetas de gran tamaño circulantes en la sangre, trombocitopenia e inclusiones leucocitarias que se parecen a las encontradas en la anomalía de May-Hegglin

El síndrome de Alport se caracteriza por tener afección renal, coclear y ocular. La principal señal de este síndrome es la hematuria microscópica (microhematuria). Los hombres con el síndrome Alport ligado al cromosoma X (XLAS) padecen microhematuria desde una edad muy temprana. Alrededor del 90% de mujeres con XLAS también la tienen. Hay 2 métodos para el diagnóstico clínico: secuenciación y análisis de deleción/duplicación. El análisis de secuenciación de COL4A5 identifica cerca del 80% de las mutaciones de individuos afectados con antecedentes familiares en herencia ligada al X. El análisis de deleción/duplicación del gen COL4A5 identifica deleciones (típicamente multiexónicas) cercanas al 10% de individuos afectados con antecedentes familiares ligada al X

Referencias

Behrman RE, Kliegman RM, Jenson HB. Nelson Textbook of Pediatrics. 17th ed. Philadelphia, Pa: Saunders; 2003. 23.- Masculino de 2 de dos meses de edad, con sospecha de menigoencefalitis por clínica y un líquido cefalorraquídeo con aumento de leucocitos, aumento de proteínas e hipoglucorraquia en el que no se tiene aún un germen aislado. El tratamiento de elección es: a) Dicloxacilina y amikacina b) Ampicilina y cefotaxime c) Penicilina sódica cristalina d) Vancomicina La meningoencefalitis es definida como la inflamación de las meninges y el encéfalo. Dentro de las causas de la meningoencefalitis purulenta la más frecuente es la bacteriana. La etiología varía dependiendo del grupo de edad, en menores de 2 meses el más común es Streptococcus del grupo b, después de esta edad el más frecuente es el S. pneumoniae. El cuadro clínico se caracteriza por 4 síndromes: infeccioso, de hipertensión endocraneana, meníngeo y de daño neuronal. Las complicaciones pueden ser agudas, subagudas y crónicas. El diagnóstico se hace con el estudio citoquímico y cultivo del LCR. Las medidas terapéuticas irán encaminadas al manejo de los signos y síntomas componentes de los 4 síndromes. El tratamiento específico se iniciará en forma empírica y se modificará de acuerdo a los resultados del cultivo. En la meningitis bacteriana el aspecto macroscópico LCR es turbio o incluso purulento debido al mayor contenido de células y proteínas. En la mayoría de los casos de meningoencefalitis bacteriana el número de leucocitos es superior a 500 x mm3; y el predominio de células polimorfonucleares es mayor a 50%. La presencia de hipoglucorraquia (< 50% de la glicemia central o glucorraquia < 40 mg/dL) e hiperproteinorraquia moderada (entre 200 y 500 mg/dL) sugiere etiología bacteriana. Sin embargo, en casos con cuadros neurológicos de evolución más prolongada, mayor de 10 días, el diagnóstico diferencial debe establecerse con meningoencefalitis de etiología tuberculosa.

Page 27: Examen final 20460

Enfermedades infecciosas y microbiología Número number 1 enero-marzo January-march 2002 volumen volume 22 Meningoencefalitis bacteriana Fortino solórzano santos,* maría guadalupe miranda novales,** Rita d díaz ramos* 24.- Un niño de 12 años en Yautepec Morelos, se encuentra dormido sobre una toalla a la orilla de una alberca, súbitamente presenta dolor intenso en muslo derecho y se observa una pequeña pápula eritematosa El niño refiere sensación de ardor intenso en el muslo, su madre aplica una pomada con antihistamínico y le administra un antihistamínico oral. Dos horas después el paciente inicia parestesias, nausea y vómito, comienza a presentar dolor abdominal intenso, en episodios, síntomas que en una hora se hacen más intensos, la causa más probable de este cuadro es: a) Picadura de viuda negra. b) Picadura de alacrán. c) Picadura de cara de niño. d) Picadura de nauyaca El veneno que inocula es 15 veces más potente que el veneno de una serpiente de cascabel y puede condicionar efectos sistémicos graves e incluso la muerte. El primer síntoma generalmente es un dolor similar a una punción con un alfiler y la sensación se experimenta cuando realmente se ha efectuado la picadura de la araña. Sin embargo, es posible que algunas personas no lo sientan. Puede haber hinchazón y enrojecimiento leve y una lesión en forma de diana.

De 15 minutos a una hora más tarde, un dolor muscular sordo se irradia desde el área de la picadura a todo el cuerpo. Síntomas

Page 28: Examen final 20460

Náuseas Ataque al estado general Diaforesis Contracturas musculares Dolor muscular Retención urinaria Estreñimiento Taquicardia Insuficiencia cardiaca Hipertensión arterial Inquietud Ansiedad Sensación de muerte Inminente Current Pediatric Diagnosis and Treatment 17 Ed Mc Graw Hill. Pag 346 . 2005 25.- Preescolar de 3 años es llevado a consulta por preentar hiporexia. Antecedentes: Originario de zona rural del estado de Guerrero, geofagia positiva, dolor abdominal, cólico desde hace varios meses. Las evacuaciones en los últimos 5 días son semilíquidas acompañadas de moco y pujo no sangre. E.F.: mala higiene personal, desnutrido, abdomen globoso blando y dolor a la presión en colon descendente, peristalsis aumentada. La complicación más frecuente que se puede presentar en este paciente es: a) Apendicitis. b) Obstruccion intestinal. c) Prolapso rectal, trichurosis d) Perforación intestinal. Introducción. La trichuriosis es una geohelmintiasis frecuente en zonas tropicales, rurales. Se estima que se encuentran infectadas unos 100 millones de personas en Latinoamérica y Caribe (Hotez PJ, et al., 2008). Predomina en niños en edad escolar, en quienes se asocia a colitis crónica y síndrome disentérico, retardo en el crecimiento y disminución de peso; la deficiencia en las funciones cognitivas y alteraciones conductuales se han relacionado con anemia ferropriva, altas cargas parasitarias y desnutrición. Los casos de la parasitosis en adultos que viven en zonas endémicas han aumentado, pero no se reportan usualmente. (Khuroo M, et al. 2010).

La ascariosis y la trichuriosis son las infecciones por geohelmintos más frecuentes en México.

NTDs en LAC: Prevalencia y distribución. Hotez PJ, et al, 2008.

LAC: Latin American and Caribbean Region

NTDs. Neglected Tropical Diseases.

Las lesiones intestinales y el cuadro clinico varían en relación directa al número de parásitos y factores dependientes del hospedero (edad, estado nutricional, infecciones concomitantes). En infecciones leves y moderadas el daño, apenas apreciable, consiste en compresión mecánica de las células de la mucosa colónica y se asocia a dolor abdominal de tipo cólico y episodios diarreicos. En infecciones masivas la mucosa intestinal se encuentra edematosa y friable, con sangrado fácil; es característica la degeneración y necrosis de las células cercanas a la cabeza del parásito, con pequeñas hemorragias subepiteliales e inflamación con infiltración difusa de linfocitos y eosinófilos.

Page 29: Examen final 20460

Induce, al igual que los otros geohelmintos, una respuesta de tipo Th2 y respuesta reguladora Th2/Treg (Jackson JA, et al. 2009).

Las manifestaciones clínicas varían de acuerdo a la masividad de la infección y la presencia de otros parásitos (poliparasitismo) e incluyen dolor abdominal, cefalea, hiporexia, pérdida de peso, diarrea crónica, disentería, pujo, tenesmo, prolapso rectal y signos y síntomas relacionados con anemia hipocrómica microcítica; cada tricocéfalo expolia alrededor de 0.005 ml de sangre/día y restos tisulares. Además, la irritación constante de las terminaciones nerviosas intramurales redunda en hiperperistaltismo.

Complicaciones. - Poliparasitismo - Prolapso rectal - Anemia - Apendicitis - Infección bacteriana 2aria - Retraso pondoestatural y déficit cognitivo en escolares.

TRICHURIOSIS

Dra. Teresa Uribarren Berrueta Departamento de Microbiología y Parasitología, Facultad de Medicina, UNAM October, 2010

Vínculos. - Mohammad S. Khuroo, Mehnaaz S. Khuroo, and Naira S. Khuroo. Trichuris dysentery syndrome: a common cause of chronic iron deficiency anemia in adults in an endemic area (with videos). Gastrointestinal Endoscopy, Jan 2010; 71(1):200-204. doi:10.1016/j.gie.2009.08.002 - Geary TG, Woo K, McCarthy JS, Mackenzie CD, Horton J, Prichard RK, de Silva NR, (...), Bundy DA. Unresolved issues in anthelmintic pharmacology for helminthiases of humans. Int J Parasitol 2010;40(1):1-13. doi:10.1016/j.ijpara.2009.11.001 Geohelmintos y otros nematodos. - Kyung-Sun Ok, et al. Trichuris trichiura Infection Diagnosed by Colonoscopy: Case Reports and Review of Literature. Korean J Parasitol. Sept 2009;47(3):275-280 DOI: 10.3347/kjp.2009.47.3.275 - Hu Y, Xiao S-H, Aroian RV. The new anthelmintic tribendimidine is an L-type (Levamisole and Pyrantel) nicotinic acetylcholine receptor agonist. PLoS Neglected Tropical Diseases 2009;3(8), art. no. e499. - Jackson JA, Friberg IM, Little S, Bradley JE. Review series on helminths, immune modulation and the hygiene hypothesis: Immunity against helminths and immunological phenomena in modern human populations: Coevolutionary legacies? Immunology 2009;126 (1):18-27. doi:10.1111/j.1365-2567.2008.03010.x

Page 30: Examen final 20460

26.- Acude al servicio paciente masculino de 66 años de edad el cual refiere disminución de la visión en ambos ojos con varios meses de evolución, esta disminución se presenta a la visión de lejos y más acentuada en la visión próxima o de lectura. Además ha notado mayor pérdida visual con luz solar intensa y se deslumbra con mayor facilidad. No refiere alteraciones en la percepción de los colores, aunque sí cree verlos más apagados, tampoco refiere metamorfopsias. El diagnóstico más probable es: a) Aumento de la presbicia. b) Desarrollo de cataratas. c) Desarrollo de neuropatía óptica anterior isquémica. d) Desarrollo de glaucoma. Cataratas. Cualquier opacidad del cristalino, conlleve o no incapacidad funcional. CATARATA CONGÉNITA. Por alteraciones a partir de la 4ª ó 5ª semana de embarazo. Aparecen al nacer o en los primeros tres meses de vida. Provocan más del 10% de las cegueras en edad escolar. Pueden ser: • Hereditarias (10-25%). Bien aisladas o asociadas a malformaciones oculares o sistémicas. Comportamiento familiar y generalmente bilaterales. • Embriopatías. Por infecciones intrauterinas, sobre todo durante el primer trimestre de embarazo, como rubéola, toxoplasmosis y citomegalovirus. • Metabólicas. - Galactosemia. Por déficit del enzima galactosa 1-P uridil transferasa, que se transforma en galactitol. Este se deposita en el cristalino, provocando una retención de agua que lo opacifica. A medida que el niño va tomando leche van apareciendo cataratas, hepatoesplenomegalia y retraso mental. Reversible, si se diagnostica en las primeras fases de la enfermedad . - Hipoparatiroidismo o pseudohipoparatiroidismo: cuando el Ca2+ es menor de 9,5 mg/100 ml, se puede originar catarata. • Tóxicas. Por clorpromacina, corticoides, hipervitaminosis D. • Carenciales. Por déficit de vitamina A, triptófano, ácido fólico o vitamina B12. • Cromosomopatías. Síndrome de Down (50% tienen cataratas), Turner. CATARATAS SECUNDARIAS. Aparecen en el curso de procesos oculares o sistémicos. Oculares: queratitis, uveítis, tumores coroideos, traumatismos, desprendimiento de retina y glaucoma, entre otras. Sistémicas: • Metabólicas. En diabetes, hipertiroidismo, enfermedad de Wilson, distrofia miotónica de Steinert • Sindermatóticas. En patología dermatológica: poiquilodermia, esclerodermia y eccema atópico. • Tóxicas. Por metales como talio, plata, mercurio, hierro, cobre, o fármacos como corticoides, mióticos, antimitóticos, ergotamina. CATARATA SENIL. Forma más frecuente de catarata y causa más frecuente de pérdida visual reversible en países desarrollados. Son bilaterales, aunque de desarrollo asimétrico. CLÍNICA. Se produce una disminución progresiva de la agudeza visual, sin dolor ni inflamación. Mejora en ambientes poco iluminados o tras instilar un midriático, y empeora en ambientes muy iluminados. En algunos pacientes los síntomas comienzan con una mejoría de la presbicia, por un aumento en el índice de refracción del cristalino. También pueden aparecer fotofobia, visión de halos coloreados y diplopía monocular (debidos a diferencias de refracción entre zonas de la lente). Puede ser nuclear, cortical (por hiperhidratación) o subcapsular posterior. Se denomina incipiente cuando la opacidad y el trastorno visual son mínimos, madura cuando la opacidad es total y el déficit visual severo, e hipermadura, cuando se produce disolución y licuefacción de las fibras, perdiendo su estructura y apareciendo de color gris homogéneo. La cápsula puede presentar pliegues y el núcleo caer hacia abajo.

Page 31: Examen final 20460

MUY IMPORTANTE En un paciente de edad avanzada que progresivamente desarrolla disminución de la agudeza visual que empeora con luz intensa y mejora de la presbicia, debemos sospechar catarata senil. COMPLICACIONES. Si la catarata no se opera, pueden aparecer las siguientes complicaciones: • Iridociclitis. Por salida de las proteínas del cristalino a través de la cápsula. • Glaucoma secundario agudo o facomórfico. El cristalino capta agua y aumenta de volumen, provocando un aplanamiento de la cámara anterior y un cierre angular con aumento de PIO. • Glaucoma facolítico. Proteínas cristalinianas que pasan a cámara anterior y provocan una obstrucción de la malla trabecular. • Luxación del cristalino. DIAGNÓSTICO. El diagnóstico general de cataratas se hace provocando midriasis farmacológica y observando a simple vista, apareciendo una leucocoria en los estadios avanzados; o bien observando mediante oftalmoscopia directa a unos 30 cm, apreciándose manchas oscuras sobre el reflejo rojo del fondo del ojo, o impidiendo ver este reflejo cuando son maduras; o bien con lámpara de hendidura, que permite ubicar la opacidad dentro del cristalino. TRATAMIENTO. El tratamiento es exclusivamente quirúrgico. La técnica más utilizada se denomina facoemulsificación 27.- Paciente femenino de 45 años que presenta palpitaciones, debilidad, palidez de tegumentos, taquicardia, glositis, parestesias dístales y después ascendentes; y que cursa con ataxia. Bh con Hb de 8; VCM 110 fL; leucocitos y plaquetas discretamente disminuidos. Lo primero que pensaría es:

a) Infección crónica. b) Leucemia. c) Hemorragia. d) Deficiencia de cobalamina.

Farreras. Medicina interna. Cap 206. Decimoquinta edición 2004, Pp. 1652-1653. Dado que la cobalamina en el trofismo adecuado de la piel y las mucosas y el mantenimiento de la mielinizaciòn adecuada, pueden aparecer alteraciones digestivas y neurológicas, como glositis (de Hunter) y trastornos gastrointestinales inespecíficos (flatulencia, digestiones pesadas), aunque algunos pacientes presentan diarrea que sólo cede con el tratamiento con cobalamina. Las manifestaciones neurológicas se deben a degeneración axonal y desmielinizaciòn de los cordones medulares posteriores (degeneración combinada subaguda). 28.- Femenino de 62 años con diagnóstico de miastenia gravis. De los siguientes medicamentos el que está dirigido al manejo de esta patología es: a) Neostigmina b) Quinidina c) Sumatriptán d) Succinilcolina Manejo farmacológico: Existen diferentes pautas terapéuticas dirigidas a contrarrestar los síntomas de la enfermedad o el mecanismo inmunológico. Los fármacos utilizados son:

- Inhibidores de la Acetilcolinesterasa (Neostigmina, Piridostigmina). Dirigidos al manejo sintomático de la MG, mejorando la fuerza motora pero no la progresión de la enfermedad. Su

Page 32: Examen final 20460

mecanismo de acción es la inhibición reversible de la acetilcolinesterasa, lo cual genera un aumento de ACh en la placa motora. La dosis a utilizar es variable y debe modificarse en distintas etapas de la enfermedad, incluso siendo frecuente no lograr un efecto uniforme en los diferentes grupos musculares en un mismo paciente. El objetivo por lo tanto será utilizar la dosis mínima con la que se genere la mejor respuesta clínica. El efecto se obtiene de 30 minutos a 2 horas de la administración y tiene una duración de hasta 6 horas. Las dosis recomendadas de Piridostigmina son de 15-60 mg cada 4-6 horas vía oral y de Neostigmina 0,5-2 mg/kg cada 4-6 horas intramuscular. Las reacciones adversas asociadas son: dolor abdominal, hipersalivación, aumento de las secreciones respiratorias y bradicardia y se relacionan con el efecto colinérgico generado, por lo que es necesario administrar concomitantemente atropina2,3.

Ponsetia JM: Miastenia Gravis. Manual Terapéutico. Barcelona; Springer Verlag Ibérica, 1995 2.- Ponsetia JM, Espina E, Armengola M: Diagnóstico y Tratamiento de la Miastenia grave. Med Clin (Barc) 2000; 115: 264-70. 3.- Drachman DB: Myasthenia gravis. N Engl J Med 1994; 330: 1797-810 4.- Andrews PI: Autoimmune myasthenia gravis in childhood. Semin Neurol 2004; 24: 101-10 5. - Anlar B: Juvenile myasthenia: diagnosis and treatment. Paediatr Drugs 2000; 2: 161-9. 6. - Gajdos P: Myasthenic syndrome. Diagnosis trends. Rev Prat 2000; 50: 419-23 7. -

29.- Femenino de 34 años la cual labora en lavandería, hace varios años presenta enrojecimiento de los pliegues proximales de las uñas de varios dedos de las manos, que ocasionalmente le supuran. El diagnóstico más probable de la paciente es: a) Dermatitis de contacto. b) Paroniquia candidiásica crónica. c) Liquen plano. d) Dermatoficia. Infección por Candida del lecho ungueal que se presenta con mayor frecuencia como una onicolisis asociada con paroniquia, aunque también se observa la destrucción completa del lecho ungueal y la erosión de la zona distal y lateral de las uñas de los dedos, sin distrofia ungueal total. La perionixis candidiásica se trata de la inflamación con eritema, edema, dolor y salida ocasional de pus blanquecino escaso y espeso del reborde ungueal de uno o varios dedos de la mano. Con frecuencia, se acompaña de onicopatía (discoloración, onicolisis, distrofia total, etc.).

Predisponentes: Todas las labores manuales que generen humedad. (muy frecuente en amas de casas, trabajadores de restaurantes, lavanderas, etc.)

Page 33: Examen final 20460

BIBLIOGRAFÍA: 1. Crespo Erchiga V. Protocolo diagnóstico de contaminantes. En "Micología para dermatólogos" Ed. Janssen, Madrid, 1994, pp:49-70. 2. Crespo V, De Luís B, Delgado V, Crespo A y Vera. Espectro etiológico de las onicomicosis en nuestro medio. CO7. II Congreso Nacional de Micología. Santiago de Compostela. Junio, 1994. 3. Crespo Erchiga V, Delgado Florencio V y Martínez García S. Micología dermatológica. Ed. M.R.A. Barcelona, 2006. 4. Daniel III CR. The Diagnosis of Nail Fungal Infection. Editorial Arch Dermatol 1991;127:1566-1567. 5. Delgado Florencio V. Protocolo de identificación de dermatofitos. En "Micología para dermatólogos". Ed. Janssen, Madrid, 1994, pp:27-41. 6. Delgado Florencio V. Estrategia en el diagnóstico y tratamiento de las micosis superficiales. Ed. Aula Médica, Madrid, 1994. 7. Delgado V, Abad Romero-Balmas J, Armijo Moreno M y Dulanto F. Scopulariopsis brevicaulis como agente de onicomicosis. Actas Deermo-Sif. 1976; 9-10:693-700. 8. English MP. Nails and fungi. Br J Dermatol 1976; 94:697-701. 9. Fevilhade de Chauvin M. Onicomicosis. Dermatología práctica. 1994; 9:1-2. 30.- Femenino que desde hace varias semanas le han aparecido de forma eruptiva unas máculas y placas eritematosas en el tronco, refiere que hubo una lesión más grande que precedió a las demás. Las lesiones presentan una descamación fina en la periferia y son dis-cretamente pruriginosas. No existe afectación palmoplantar. La serología luética es negativa. EL tipo de pitiriasis más probable ES: a) P. liquenoide crónica. b) P. rubra pilaris. c) P. rosada. d) P. versicolor

La pitiriasis rosada es una enfermedad exantemática autolimitada de causa desconocida que cursa con lesiones maculopapulosas de aspecto asalmonado en áreas proximales de las extremidades y en tronco, donde se distribuyen de forma paralela a las líneas de la hendidura (Stulberg DL, 2004).

Page 34: Examen final 20460

Existen algunas características que sugieren una etiología vírica (pródromos, predominio estacional, evolución variable, ausencia de recaídas, hallazgos histológicos...) que algunos autores relacionan con el herpes virus humano 7, aunque otros lo desmienten (Chuh A, 2004).

Afecta con más frecuencia a niños mayores y jóvenes adultos (10-35 años) y es ligeramente más frecuente en mujeres (Chuang TY, 1982).

El cuadro típico se inicia con una lesión redondeada u oval, de 4-8 cm. única, generalmente situada en el tronco, de color asalmonado, discretamente descamativa y en ocasiones ligeramente pruriginosa, llamada “madre”. Puede acompañarse de síntomas generales leves: astenia, anorexia, odinofagia, cefalea y artralgias.

En 1-2 semanas esta lesión se torna mas escamosa y clara en el centro al tiempo que aparecen otras de menor tamaño que se diseminan por las extremidades (áreas proximales) y por el tronco distribuyéndose simétricamente en forma arbórea a lo largo de las hendiduras costales. En la espalda es más evidente (“signo del árbol de Navidad”) (Stulberg DL, 2004)

La evolución de las lesiones es similar a las de la lesión “madre” desapareciendo en 5-6 semanas, aunque en algunos casos puede permanecer más tiempo y en otros verse áreas hiper o hipo pigmentadas durante algún tiempo. La tasa de recurrencia a los 5 años es inferior al 2% (Chuang TY, 1982).

Pitiriasis Rosada

Chuh A, Chan H, Zawar V. Pityriasis rosea--evidence for and against an infectious aetiology. Epidemiol Infect. 2004 Jun;132(3):381-90.

Chuh AA, Chan HH. Prospective case-control study of chlamydia, legionella and mycoplasma infections in patients with pityriasis rosea.

Chuh AA, Dofitas BL, Comisel GG, Reveiz L, Sharma V, Garner SE, Chu F. Interventions for pityriasis rosea. Cochrane Database Syst Rev. 2007 Apr 18;(2):CD005068.

Chuh AA. Quality of life in children with pityriasis rosea: a prospective case control study. Pediatr Dermatol. 2003 Nov-Dec;20(6):474-8.

Hsu S, Le EH, Khoshevis MR. Differential diagnosis of annular lesions. Am Fam Physician. 2001 Jul 15;64(2):289-96] Eur J Dermatol. 2002 Mar-Apr;12(2):170-3.

Miranda SB, Lupi O, Lucas E. Vesicular pityriasis rosea: response to erythromycin treatment. J Eur Acad Dermatol Venereol. 2004 Sep;18(5):622-5

Page 35: Examen final 20460

31.- Femenino de 39 años, acude a consulta refiriendo mareo a la bipedestación. Antecedentes: DM2 de 10 años de evolución bajo tratamiento médico. Exploración física: TA 105/60mmHg sentada y de pie 95/60mmHg, FC 76 x´, FR 16 x´. Usted solicita ES. Reportando: Na 164 mEq/l, osmolaridad urinaria 210 mOsm/l, osmolaridad sérica 330 mOsm/l, sodio urinario 45 mOsm /l. El diagnóstico de primera elección en este paciente es: a) Diabetes juvenil b) Diabetes insípida nefrogénica. c) Diabetes insípida central. d) Diabetes periférica Diabetes insípida nefrogénica: Se caracteriza por la falta de respuesta tubular a la AVP, en presencia de concentraciones normales de la misma. Causas: alteraciones del receptor V2, uso de litio, hipercalcemia, hipopotasemia, enfermedades renales que interfieren en la concentración de orina. Consecuencias: Poliuria hipoosmótica (>40ml/Kg./d) (<200 mOsm/Kg.), tendencia a la deshidratación hipertónica (285-290mOsm/Kg.), con sodio de 140-145mEq/l, exceso de sed en personas con conciencia conservada, la ingesta de sodio es igual a la secreción Hernando L, Nefrología Clínica, Ed. Panamericana, 3ªEdición, 2008, Pág. 42-43 32.- Mujer de 37 años. Acude a consulta por presentar cefalea, cansancio e irregularidades menstruales con ritmo de 36 a 50 x 2-3 días. No se ha podido embarazar después de 18 meses de actividad sexual regular. No tiene antecedentes importantes. EF: Campos visuales normales, tiroides aumentada de tamaño una vez y aumentada de consistencia, no tiene galactorrea. Resto normal. Laboratorio: química sanguínea, Bh y electrolitos normales. Prolactina 47 ng/dL (< 25), perfil tiroideo: TSH 18 mUI/ml, T4t: 50 nmol/L (57.9 a 154.4), T4L: 7.7 pmol/L (9 a 24), T3T: 1.06 nmol/L (1.2 a 2.9), T3L: 1.96 pmol/L (3 a 6.31) La etiología más frecuente de este problema es: a) Tiroiditis autoinmune crónica b) Microadenoma hipofisario c) Resistencia a la insulina d) Enfermedad de Graves Inicialmente el hipotiroidismo se diagnosticaba mediante la cuantificación por técnicas de Radio Inmuno Análisis (RIA) de las hormonas circulantes triyodotironina y tiroxina; el proceso era lento y sometido a muchos factores de error que hacían su sensibilidad y especificidad poco confiables. Posteriormente, se desarrollaron técnicas para la medición de la TSH hipofisiaria igualmente mediante el RIA lo que mejoró en forma importante la sensibilidad para el diagnóstico de esta enfermedad; sin embargo, los niveles de detección de la prueba se encontraban en el orden de 1 µIU/ml lo que hacía que la prueba no fuera sensible para valores menores de 1 µIU/ml. Debido a esto se crearon técnicas de segunda generación mediante la cuantificación de TSH por anticuerpos monoclonales y RIA, el IRMA (Immuno Radiometric with Monoclonal Antibodies) que permitió detectar valores de TSH en rangos de 0.1 µIU/ml; posibilitando desde entonces diagnosticar pacientes con hipertiroidismo primario; pero con la limitante de que para esta técnica era imposible detectar valores de TSH menores de 0.1 µIU/ml por lo que se creo la medición de TSH mediante quimioluminiscencia o métodos enzimáticos, es decir las técnicas de tercera generación, las cuales pueden detectar valores de TSH de 0.01 µIU/ml; con lo que se logra el espectro ideal para una prueba de laboratorio que tiene la capacidad de diagnosticar tanto la hipofunción como la hiperfunción(20).

Page 36: Examen final 20460

Además el avance no sólo fue en la medición de TSH sino también en las hormonas tiroideas que han evolucionado simultáneamente con la TSH y ya se miden incluso las fracciones libres de hormonas y las fracciones totales, lo que ha facilitado el manejo de estos pacientes. Gracias a esta evolución en técnicas de laboratorio, el diagnóstico de hipotiroidismo primario es bastante sencillo. Niveles de TSH superiores al valor máximo de la técnica serían diagnósticos de la disfunción; pero no es tan fácil. Cuando tenemos un paciente con toda la sintomatología del hipotiroidismo y la TSH se encuentra elevada el diagnóstico es obvio; pero podemos tener pacientes con síntomas muy inespecíficos como depresión y con examen físico normal a quienes se les encuentran valores de TSH por encima del límite superior y con hormonas tiroideas normales. Se trata de un hipotiroidismo o es un valor ligeramente elevado ocasional de una persona sana(21). Igualmente tenemos otra circunstancia que ha sido descrita con mayor frecuencia: pacientes con valores de TSH en el límite superior normal y con dislipidemia a quienes se les da tratamiento con hormonas tiroideas y su dislipidemia se corrige manteniendo valores de TSH en rangos normales. Todas las circunstancias anteriores han hecho que aparezca en el hipotiroidismo primario la expresión de hipotiroidismo subclinico, que ha sido objeto de reuniones y congresos dedicados exclusivamente a este tema. La sociedad Europea de Tiroides hace algunas recomendaciones para el manejo de esta situación que se consideran útiles como guía (Tabla ).

Tabla. Enfoque del paciente con disfunción tiroidea de acuerdo a los niveles de hormona estimulante de la tiroides (TSH).

Si TSH < 0.4m U/L Si TSH 0.4 a 2.0 mU/L Si TSH 2.01 a 5.0mU/L

Si TSH > 5.0 mU/L

Medir T3 y T4 totales o libres para diagnostico de hipertiroidismo.

Normal, Repetir cada cinco años

Medir T4 libre y anticuerpos antitiroideos

Dar tratamiento para hipotiroidismo

1. Si AAT (-) y T4 libre es normal repetir screening cada año. Si TSH es > 4.0mU/l en dos ocasiones dar tratamiento

2. Si AAT (+) y/o T4 libre esta baja o normal baja tratar si TSH es mayor de 3.0 mU/l y observar a los otros

Tomado de Koutras DA. Subclinical hypothyroidism. En G. Hennemann, E.P. Krenning, Thyroid International Merck KGaA, Darmstadt 1999 (3), 6-9

Page 37: Examen final 20460

HIPOTIROIDISMO

PRIMARIO: congénito: agenesiaautoinmunepostablativotiroiditis subagudadeficiencia de yodo

SECUNDARIOtumor hipofisiarioiatrogénico

TERCIARIO

33.- Se trata de masculino de 11 años de edad sin antecedentes de importancia para el padecimiento actual. Inicia hace 14 días con rinorrea hialina, tos en accesos, adinofagia, cefalea y febrícula. Es atendida por médico quién administra manejo sintomático con discreta mejoría. Hace 5 días inicia con dolor dental,la rinorrea se torna mucopurulenta con incremento de eventos de tos de predominio nocturno. A la exploración presenta faringe hiperémica con descarga posterior, halitosis, narinas con moco purulento. El diagnóstico más probable es: a) Faringoamigdalitis bacteriana b) Sinusitis aguda c) Difteria d) Faringoamigdalitis viral

SINUSITIS. Las inflamaciones de los senos paranasales constituyen una afección frecuente, aunque

generalmente banal, y con frecuencia son una extensión de la patología de las fosas nasales. Aproximadamente un 5 % de la población padece en algún momento una sinusitis crónica.

El seno más afecto es el maxilar, seguido del etmoides, frontal y esfenoidal: Durante el periodo pediátrico los senos más afectados son los etmoidales.

CLASIFICACIÓN: - Sinusitis aguda: Duración menor a 4 semanas - Sinusitis subaguda: Duración: 4 a 12 semanas

Page 38: Examen final 20460

- Sinusitis crónica: Duración superior a 12 semanas con posibles reagudizaciones. Los pacientes presentan síntomas respiratorios persistentes (tos, rinorrea...) con síntomas de sinusitis aguda en las reagudizaciones. Generalmente se asocian a otras patologías: alergias, poliposis, alteraciones estructurales... Sintomatología: 1) Cefalea: en general orbital, frontal y premaxilar; 2) Sensación de pesadez frontoorbital, refiere que al agacharse se manifiesta con mayor intensidad; 3) Rinorrea purulenta abundante uni o bilateral. Puede ir acompañando o no a 1 y 2. El paciente refiere con acento la abundante cantidad de mucosidad expelida por nariz o deglutida desde cavum; 4) Trastornos visuales vagos: en general son motivo de consulta con Oftalmología, se quejan de visión insuficiente, malestar ocular indefinido, a veces dolor en órbitas; 5) Insuficiencia respiratoria nasal, determinada por la congestión que produce el cuadro sinusal; 6) Hiposmia prolongada, generalmente asociada a CVAS permanente; 7) Tos o catarro de vías respiratorias inferiores: deglución de secreciones, respiración bucal por insuficiencia respiratoria nasal parcial o completa; 8) Sensación de oídos tapados: relacionable con obstrucción tubaria secundaria a CVAS; 9) Disfonía recidivante: asociada con antecedente de CVAS a repetición o constantes; 10) Edema facial premaxilar: es indicativo de una sinusitis aguda; 11) Halitosis: síntoma muy habitual pero que pasa desapercibido, pocas veces es referido en la consulta por el propio paciente, generalmente lo refiere el familiar acompañante; 12) Epistaxis: en general se relacionan más con las rinitis puras que con las sinusitis, no obstante como toda sinuitis entraña una rinitis asociada, suelen verse episodios por lo general no importantes; 13) Edema o abceso periorbitario: relacionado con sinusitis etmoidal, habitualmente niños, menos frecuentemente en adolescentes. Criterios:

Criterios mayores: Algia facial, obstrucción nasal, rinorrea posterior, hipósmia, rinorrea purulenta a la rinoscopia.

Criterios menores: Cefalea, fiebre, halitosis, astenia, dolor dental, plenitud ótica.

Los microorganismos implicados son:

Bacterias: Streptococos pneumoniae y Haemophilus Influenza son responsables de más del 50% de los casos de sinusitis aguda en adultos y niños. La Moraxella Catarralis, rara en adultos, es la tercera causa en niños. Un 10% de los casos en adultos pueden ser debido a anaerobios. Una infección polimicrobiana anaerobia sugerirá en el adulto una enfermedad dental subyacente. Streptococo pyogenes, Stafilococo aureus y Gram- como Pseudomona aeruginosa, E. Coli y Klebsiella spp son raramente encontradas. El Stafilococo aureus es más frecuente en procesos crónicos que en agudos.

Hongos: Aspergillus spp, Zygomices, Phaeohyphomyces. Su papel en pacientes inmunocompetentes no está claro; sin embargo en inmunocomprometidos (cetoacidosis diabética fundamentalmente) obliga a tratamiento inmediato, médico y quirúrgico.

Virus: Rhinovirus, Virus Influenza y Parainfluenza son los más comunes. Son los microorganismos más habituales en síntomas sinusitis-like.

Page 39: Examen final 20460

Bacterias Intervalo de aislamientos % Adultos Niños S. pneumoniae 20-43 35-42 H. influenzae 22-35 21-28 Streptococcus spp.* 3-9 3-7** Anaerobios 0-9 3-7 M. catarrhalis 2-10 21-28 S. aureus 0-8 Otras*** 4 Tomado de Antimicrobial Treatment Guidelines for Acute Bacterial Rhinosinusitis, 2000. *Incluyendo S. pyogenes. **S. pyogenes. ***Enterobacterias, P. aerugi - nosa, etc. (Gwaltney 1996) Bibliografía: •Agency for Health Care Policy and Research. Diagnosis and treatment of acute bacterial rhinosinusitis. [Internet].Boston: New England Medical Center; 1999. [acceso 18/5/ 2004]. •Agency for Health Care Policy and Research. Diagnosis and Treatment of Uncomplicated Acute Sinusitis in Children. [Internet]. Rockville: Agency for Healthcare Research and Quality; 2000. [acceso 18/5/ 2004]. Disponible en: Alberta Clinical Practice Guidelines Program. Guideline for the diagnosis and management of acute bacterial sinusitis. [Internet].Revised november 2001. [acceso 18/5/ 2004]. •American Academy of Otolaryngology-Head and Neck Surgery. Guide to Antimicrobial Therapy in Otolaryngology-Head and Neck Surgery.[Internet]. 11 ed. Alexandria (USA); 2003. [acceso 18/5/ 2004]. •American Academy of Pediatrics. Clinical practice guideline: manage-ment of sinusitis. [Internet]. 2001. [acceso 18/5/ 2004]. •American College of Physicians-American Society of Internal Medicine-Medical Specialty Society. Principles of appropriate antibiotic use for acute sinusitis in adults. Ann Intern Med 2001; 134(6):495-7. [ •American College of Radiology. ACR Appropriateness Criteria T for sinusitis in the pediatric population. [Internet]. 2002. [acceso 18/5/ 2004]. •Brunton SA. Managing acute maxillary sinusitis in the family practice. J Fam Pract. 2003 Supplement October; 4-11.

34.- Masculino de 42 años alcohólico con cirrosis y ascitis. Es hospitalizado por agitación y comportamiento extraño, así como asterisix de manos. Se sospecha de encefalopatía hepática. Su nivel de amonio en sangre es dos veces el basal. De los siguientes, el mayor factor precipitante para presentar este cuadro es: a) Várices sangrantes b) Mal uso de diuréticos c) Ingestión proteica insuficiente d) Ingesta reciente de alcohol

Page 40: Examen final 20460

ENCEFALOPATÍA

• Alteración en el estado mental, en el comportamiento y en el sueño que progresa a la desorientación y al coma.

• Indica una insuficiencia hepática severa. • Fisiopatologia

– Las toxinas son inactivadas por el hígado pero entran a la circulación portal por los cortos circuitos.

– Toxinas no son inactivadas y excretadas. – Las toxinas se encuentran elevadas en el líquido cerebroespinal.

• TOXINAS. • Amonio.

– Neuroexcitatorio y depresor del sistema nervioso central. • Aminoácidos aromáticos.

– Son precursores de neurotransmisores. – Si incrementan en sangre la encefalopatía no progresa.

• Ácido gammaaminobutírico (GABA). – Incrementado significativamente en la encefalopatía.

• Benzodiacepinas endógenas. – Compiten por los receptores de GABA y barbitúricos.

CUADRO CLINICO

El diagnóstico de la encefalopatía hepática depende de la existencia de una enfermedad hepática, desde la insuficiencia hepática aguda y la toxicidad por fármacos, hasta las enfermedades crónicas, tales como la hepatropatía alcohólica o la cirrosis.

En la historia clínica se hace énfasis especial en los antecedentes de enfermedades del hígado diagnosticadas con anterioridad, una historia de alcoholismo o de hepatitis. Transfusiones de sangre previas o abuso de drogas por vía intravenosa pueden sugerir una hepatitis crónica B o C. La utilización de fármacos hepatotóxicos, como la metildopa, la nitrofurantoina o la isoniácida, puede ser causa de una hepatopatía crónica, mientras que una dosis alta de paracetamol puede ser causante de una necrosis hepatocelular fulminante.

El cuadro clínico incluye tres elementos: cambios en el estado mental, hedor hepático y asterixis. El hedor hepático se refiere al aliento fétido del paciente. La asterixis es un temblor por sacudidas, irregular y bilateral de las manos, debido a una interrupción momentánea y brusca del tono muscular de los antebrazos.

El examen físico se centra en la búsqueda de los estigmas característicos de las enfermedades hepáticas: ictericia, nevus en araña, ginecomastia, atrofia testicular, venas distendidas en la pared abdominal (cabeza de medusa) y ascitis.

La exploración neurológica durante los estadíos precoces suele mostrar apraxia de construcción y dificultad para escribir. Otros hallazgos físicos son la rigidez de las extremidades e hiperreflexia. En el estadío de coma profundo suele haber pérdida del tono muscular y disminución de los reflejos tendinosos profundos.

RESUMEN:

• Petequias. • Rinofima. • Contractura de Dupuytren (aponeurosis palmar). • Telangiectasias en el tronco. • Ascitis. • Asterixis. • Eritema palmar. • Atrofia testicular.

Page 41: Examen final 20460

Tabla 4

Factores precipitantes de encefalopatía hepática

Várices esofágicas Infecciones

Hemorroides Ulcera péptica

Exceso de proteínas en la dieta

Constipación Hipocalemia inducida por diuréticos, déficit de la dieta, diarreas, hiperaldosteronismo

Hipovolemia inducida por diuréticos, náuseas, vómitos

Alcalosis inducida por hipocalemia

Uremia por insuficiencia renal Sedantes, tranquilizantes, analgésicos Narcóticos

Blibliografía Shakelford’s. Surgery of the alimentary tract. 5a. Ed. 2002. Tomo 3. Feldman´s. Gastroeneterology. 2002. Perez. Anatomía y fisiología del hígado. Univ. Católica de Chile. 2005. Bratiz. Serum laboratory test in cirrhosis. Journal of Hepatology. Slovakia. 2005. Paradis. Glycomics. Journal of hepatology. Ireland. Agosto 2005. 35.- Se trata de un joven de 18 años de edad, acude a su clínica un lunes por la mañana, el motivo de consulta es la presencia de más de 8 evacuaciones líquidas con sangre, moco y pujo, las últimas 24 horas, precedidas de dolor tipo cólico que cede posterior a la defecación, además de tenesmo rectal. Llama la atención que este cuadro inicio hace dos semanas, al principio solo existió disminución de la consistencia de la materia fecal, hasta que, alrededor del cuarto día, solo había evacuaciones líquidas. Afebril y sin vómitos. La exploración física demuestra mucosas orales humectadas. Dolor a la palpación de todo el abdomen, sin rebote, con ruidos intestinales intensos y abundantes. Su presión arterial es de 100/75. El diagnóstico más probable en este caso es:

a) Diarrea crónica. b) Diarrea aguda c) Colitis por parásitos. d) Enfermedad intestinal inflamatoria.

Definición: Se considera diarrea aguda a la presencia de heces líquidas o acuosas, generalmente en número mayor de tres en 24 horas y que duran menos de 14 días; la disminución de la consistencia es más importante que la frecuencia.1 El número de evacuaciones intestinales hechas en un día varía según la dieta y la edad de la persona. Los lactantes alimentados al seno materno tienen evacuaciones intestinales blandas frecuentes; ésto no es diarrea.

Page 42: Examen final 20460

Epidemiología La Organización Mundial de la Salud estima que cada año se presentan 1,300 millones de episodios de diarrea en niños menores de cinco años en países en desarrollo (África, Asia, excluida China, y América Latina), que ocasionan 4 millones de muertes, relacionadas en el 50-70% con deshidratación, lo que las ubica dentro de las principales causas de defunción en estos países. La mayoría de los niños que sobreviven quedan con algún grado de desnutrición y los desnutridos, no sólo padecen con mayor frecuencia de diarrea, sino que los episodios son más graves.3 El tercer gran problema asociado a las diarreas, en niños mayores, es el ausentismo escolar o laboral. Etiopatogénia

Los agentes etiológicos más frecuentes son, en orden decreciente, virus, bacterias y parásitos.

Los virus son la causa principal de las diarreas deshidratantes en niños menores de dos años, siendo los rotavirus del grupo A, serotipos G1 y G3, los responsables de la mayoría de los episodios.8 La diarrea osmótica que ocasionan se debe a que lesionan en forma focal las células de las vellosidades del intestino delgado, disminuyendo la producción de las enzimas encargadas de la absorción de la lactosa, entre otros disacáridos, lo que aumenta la osmolaridad en la luz intestinal y produce mayor secreción de agua que se pierde a través de las heces. Sin embargo, las células de las criptas encargadas de reparar las vellosidades lesionadas, migran para substituirlas en un periodo de 24 a 72 horas, con lo que desaparece la diarrea.9

Las enterobacterias, como Escherichia coli, Salmonella sp, Shigella, Campylobacter jejuni y Vibrio cholerae 01, producen diarrea a través de diversos mecanismos: 1. Liberación de enterotoxinas (V. cholerae 01, E. coli enterotoxigénica) que estimulan la adenilciclasa y aumentan la secreción intestinal de agua, sodio y cloro; 2. Enteroinvasión (E. coli enterohemorrágica) con disolución de la mucosa y del borde en cepillo y 3. Proliferación intracelular, previa invasión de la mucosa ( Shigella) con aparición de sangre en las evacuaciones, paso de microorganismos a la circulación sanguínea (bacteremia) y algunas veces sepsis. Shigella dysenteriae, produce además enterotoxinas que estimulan la secreción de agua y sodio en el intestino delgado y neurotoxinas que causan manifestaciones neurológicas, desde convulsiones hasta estado de coma. En los dos últimos decenios ha sido posible documentar el papel de otros patógenos como Campylobacter y Yersinia. Asimismo, se ha avanzado en los mecanismos fisio-patogénicos de las infecciones producidas por algunos tipos de Escherichia coli, Clostridium difficile, Salmonella y Aeromonas hydrophila.9

Algunos parásitos producen diarrea sanguinolenta (Entamoeba histolytica) o diarrea prolongada (Giardia lamblia). El mecanismo de producción de diarrea es a través de enteroinvasión (E. histolytica) o enteroadhesión (G. lamblia), en donde el daño más grave es de carácter nutricional.

El reservorio de los agentes de las diarreas infecciosas es el hombre, exceptuando el caso de Salmonella que es de origen animal. La fuente de infección es la materia fecal del hombre infectado, sintomático o asintomático, siendo este último más peligroso ya que no presenta datos clínicos que permitan identificarlo; en el caso de algunos virus, las secreciones nasofaríngeas pueden ser el origen. La infección genera inmunidad específica, la cual es de duración prolongada en la etiología viral y más corta en la bacteriana.

El mecanismo de transmisión clásico es el ciclo ano-boca, entendido éste como la salida del agente infeccioso con la materia fecal y su ingestión, casi siempre a través de las manos contaminadas o de fomites. Otro mecanismo de transmisión frecuente es la ingestión de alimentos contaminados o de agua, otras bebidas o hielo, no sometidos a algún proceso de purificación. Algunos factores de riesgo en el huésped son: el estado nutricional (círculo vicioso

Page 43: Examen final 20460

diarrea-desnutrición-diarrea); enfermedades previas de tipo anergizante (sarampión); ablactación temprana o ausencia de alimentación al pecho materno; saneamiento deficiente (carencia de agua potable, fecalismo); falta de educación y hábitos higiénicos; ignorancia o patrones culturales adversos (suspensión de alimentos, no aceptación de medidas sanitarias y de manejo oportuno del paciente), y económicos (limitantes a la incorporación de obras de saneamiento, adquisición de nutrientes y búsqueda de atención médica).

Referencias

1. World Health Organization. A manual for the treatment of acute diarrhoea for use by physicians and other senior health workers. Geneve: Program for Control of Diarrhoeal Diseases, World Health Organization, WHO/CDD/SER/80.2 Rev,2:1990.

2. World Health Organization. Division of Diarrhoeal and Acute Respiratory Disease Control. 25 years of ORS: Joint VMO/ ICDDR,B Consultative Meeting on ORS Formulation. Dhaka, Bangladesh, 10-12 December 1994. CDR/CDD/95.2

3. Sepúlveda J, Willet W, Muñoz A. Malnutrition and diarrhea. Alongitudinal study among urban mexican children. Am J Epidemiol 1988; 127: 365-376.

4. Mota HF, Tapia CR, Welti C, Franco A, Gómez UJ, Garrido MT. Manejo de la enfermedad diarreica en el hogar, en algunas regiones de México. Bol Med Hosp Infant Mex 1993; 50: 367-75.

5. Encuesta de manejo efectivo de casos de diarrea en el hogar. Dirección General de Epidemiología, SSA. Consejo Nacional para el Control de las Enfermedades Diarreicas. México, 1993 (Documento interno).

6. Glass RI, Lew JF, Gangarosa RE, Lebaron CW, Ho MS. Estimates of morbidity and mortality rates for diarrheal disease in American children. J Pediatr 1991; 118: 527-33.

36.- Femenino de 20 años, acude a su consulta con exámenes de laboratorio mostrando lo sig.: volumen corpuscular medio de 65 fl y en el frotis de sangre periférica se describe "anisopoiquilocitosis con punteado basófilo", siendo normal el nivel sérico de ferritina. El diagnóstico más probable es:

a) Anemia ferropénica. b) Déficit de glucosa-6-fosfato deshidrogenasa. c) Anemia hemolítica autoinmune. d) Talasemia.

Bases para el diagnóstico - Microcitosis desproporcionada respecto al grado de anemia. - Antecedentes familiares positivos o antecedentes personales de toda una vida con anemia mitoxrocítica. - Morfología eritrocitaria anormal con microcitos, acantocitos y células en diana. - En la p talasemia aumentan los valores de las hemoglobinas A2 o F. Consideraciones generales

Las talasemias son trastornos hereditarios caracterizados por disminución en la síntesis de las

Page 44: Examen final 20460

cadenas de globina (a o p), lo que origina una disminución en la síntesis de hemoglobina y al final se produce una anemia microcítica hipocrómica debida a la defectuosa incorporación de hemoglobina a los eritrocitos. Es posible incluir a las talasemias dentro de las anemias por defectos en la producción, las anemias hemolíticas y las anemias hipocrómicas asociada con la hemoglobina anormal, ya que todos estos factores participan en la patogénesis.

La prevención, el tratamiento y el manejo de la anemia drepanocítica fueron examinados por el Consejo Ejecutivo En su 117ª reunión, en la que adoptó la resolución EB117.R3 sobre esta enfermedad (véanse los documentos EB117/2006/REC/1 y EB117/2006/REC/2, acta resumida de la quinta sesión). La 59ª Asamblea Mundial de la Salud también Examinó la anemia drepanocítica (véase el documento A59/9).

37.- Characteristic to be met by subjects in a cohort study. a) Having the disease under study b) Have a family history of the disease under study c) They are adults with multiple illnesses to study d) Not having the disease under study

El estudio de cohorte es el diseño ideal para buscar la causa de una enfermedad por lo que compara a un grupo de sujetos, libres de la enfermedad en estudio expuestos a los factores de sospecha con otro grupo de sujetos también libres de la enfermedad, que no estén expuestos a los factores sospechosos.

Ruiz M. A. Epidemiología Clínica, 1ª. Ed. 2004; Pág.: 279,280 38.- Femenino de 30 años con sospecha diagnóstica de endometriosis, el síntoma característico de esta patología es:

a) Dispareunia b) Dismenorrea c) Metrorragia d) Leucorrea

ENDOMETRIOSIS:

Es relativamente frecuente en personas jóvenes, 25-30 años. Se define como la localización de la mucosa endometrial en un lugar ectópico.

La localización normal de endometrio es el útero. La mucosa endometrial sufre una serie de cambios durante el ciclo. Cuando la mucosa endometrial penetra en el músculo uterino (miometrio) se denomina: Adenomiosis.

Patogenia:

Page 45: Examen final 20460

No se conoce realmente por qué se produce y tampoco existe un tratamiento definitivo para ella.

Teorías: a) Teoría Metaplásica celómica (T. De Meyer): El celoma es un epitelio que recubre la cavidad abdominal (metaplasia).

b) Teoría Transplantativa (Sampson): La mucosa endometrial a través de las trompas, cuando se produce una menstruación si el orifico cervical está cerrado, la sangre refluye (por mecanismo retrógrado), y va a cavidad abdominal donde se producen implantes de células endometriales (en fondos de saco de Douglas, peritoneo...). También por intervenciones quirúrgicas. Esta teoría es la que parece más veraz.

c) Teoría embrionaria: Es poco importante. Dice que la endometriosis se produce por los restos embrionarios de Wolf o Müller.

d) Teoría inmunológica: Suele influir sobretodo en las personas estériles, con disminución de la respuesta inmunitaria por: - Disminución de la citotoxicidad de las células NK// - Aumento de resistencia de las células endometriales a la destrucción// - Aumento de la actividad de los Macrófagos.

Clínica:

Disfunción menstrual: Aparición de dismenorrea progresiva, que no aparece desde el principio (es tardía), es intermenstrual y aumenta al final más intensa. Va aumentando con los años (al contrario que la Dismenorrea normal).

Exploración: Retracción de los ligamentos uterosacros, nodulaciones, rugosidades y dolorosos al tacto. Dispareunia (dolor en las relaciones sexuales). Dolor en la defecación (porque la reacción inflamatoria produce adherencias y dolor en la movilización del útero). Dismenorrea.

Disfunción ovárica: Se hace quistectomía, son más conservadores en el tratamiento quirúrgico. Insuficiencia lútea. Hiperprolactinemia. Amenorrea// No son muy frecuentes ni específicas puede que no se den las tres.

Disfunción reproductiva: Es causa de esterilidad (estadios III y IV de la clasificación americana: Reacción inflamatoria que afecta a anejos, recto, sigma, apéndice, ureter...)// Se producen abortos de repetición e insuficiencia lútea.

Bibliografía

1. Stenchever A. Comprehensive Gynecology. 4th ed. St. Louis, Mo: Mosby; 2001:1065-1070.

2. Noble J. Textbook of Primary Care Medicine. 3rd ed. St. Louis, Mo: Mosby; 2001:325. 3. Chen C, Cho S, Damoskosh AI. Prospective study of exposure to environmental

tobacco smoke and dysmenorrhea. Environ Health Perspect 2000; 108(11): 1019-1022.

Page 46: Examen final 20460

39.- Se trata de paciente femenino de 37 años a quien se realiza diagnóstico de endometriosis, en ésta patología podemos encontrar elevación sérica del siguiente marcador que es: a) CA-125 b) Tc 99 c) AFP d) TGP

Beck W. (1997) NMS. Obstetrics and Gynecology. USA. Williams & Wilkins. Pag 278. Se

encuentra elevación sérica de CA-125 en pacientes con endometriosis, se trata de un antígeno

de superficie encontrado en derivados del epitelio celómico, incluyendo el endometrio, se

correlacionan sus niveles con el grado de enfermedad y la respuesta a tratamiento, pudiendo

ser un marcador de recurrencia de la enfermedad.

40.- Se presenta a consulta paciente de 25 años de edad refiere que desde hace varios días ha presentado flujo vaginal cuyas características son: blanquecino, grumoso, sin mal olor, muy irritante ¿El agente causal más probable en esta patología es?

a) Gardnerella b) Tricomonas c) Cándida albicans d) Gonococo Leucorreas micóticas: los hongos dan un flujo abundante, blanco, con grumos (aspecto como de quesillo cortado) sin mal olor y que es muy irritante de la piel de la región genital y por lo tanto genera gran ardor vaginal y prurito (picazón). Son muy frecuentes y por lo general son producidas por el hongo Candida albicans. .

Tabla I. Características del flujo vaginal según la causa

Cantidad Color Consistencia Olor

Candidiasis Escasa-moderada

Blanco-amarillento

Grumosa Indiferente

Tricomonas Aumentada Amarillo-verdoso

Espumosa Maloliente

Vaginosis Moderada Blanco-grisáceo

Homogéneo-adherente

Maloliente

Page 47: Examen final 20460

Speroff Leon and Fritz Marc A. Clinical Gynecologic endocrinology and infertility. 7ª ed. Philadelphia: Lippincott Williams and Wilkins, 2005. p. 25 – 44. 41.- Un lactante menor tiene el antecedente de traumatismos leves por caídas de su propia altura, lo que le ha ocasionado la formación de hematomas en las rodillas, los tobillos y los codos. Se solicitaron estudios de laboratorio que han revelado tiempo parcial de tromboplastina prolongado y número normal de plaquetas. El diagnóstico más probable es: a) hipoprotrombinemia. b) afibrinogenemia. c) disfibrinogenemia. d) hemofilia. Sospecha diagnóstica La enfermedad debe sospecharse en los varones con antecedentes familiares por línea materna de hemofilia. La edad de inicio de la sintomatología esta relacionada con el nivel de factor presente. Hemofilias graves: pueden presentar síntomas desde el periodo de recién nacido manifestándose como un simple hematoma en la región de vacuna BCG hasta una severa hemorragia intracraneana. En periodo de lactante aparecen equimosis en las zonas de presión y al iniciar la marcha comienzan la aparición de hematomas de magnitud exagerada para traumatismos leves y las hemartrosis. Hemofilias moderadas: puede iniciarse a temprana edad, incluso con presencia de hemartrosis y hematomas, pero en general las manifestaciones son evidentes frente a cirugías y traumas.

Page 48: Examen final 20460

Hemofilias leves: la clínica es muy escasa, puede haber antecedentes de epistaxis, equimosis fáciles o simplemente asintomático. Generalmente la sospecha diagnóstica es por exámenes preoperatorios de rutina, o bien cuando han sufrido traumas de cuantía o en relación a cirugía. Escenarios de sospecha Paciente de cualquier edad con hemorragias espontáneas o desproporcionadas al evento traumático. Inicialmente el manejo de estos pacientes es realizado por el médico no especialista. Exámenes recomendados: TTPA, TP, Hemograma con recuento de plaquetas. Si el TTPA y/o el TP son prolongados con hemograma normal en paciente sintomático, se plantea la sospecha de un déficit de coagulación congénito. La decisión de hospitalizar al enfermo dependerá de la gravedad del cuadro. En general, las hemartrosis y hematomas de baja cuantía no requieren hospitalización. Se recomienda el uso de antifibrinolíticos en sangrados de mucosa oral, nasal y digestiva. El uso de plasma está reservado para aquellas situaciones de sangrado grave en un paciente con alta sospecha de hemofilia aún no clasificada.

Page 49: Examen final 20460

RECOMENDACIÓN C. Se sugiere, antes de iniciar tratamiento, tomar una muestra de sangre en tubo citrato centrifugar y congelar el plasma a -70 ºC para el estudio inicial del caso. Intervenciones no recomendadas:

- Realizar tiempo de sangría. - Estudios de imágenes antes de tratar al paciente. - Trasladar pacientes sin haber sido estabilizados previamente. - Uso de antifibrinolíticos en hematuria.

En hemorragias de riesgo vital (ver más adelante), el traslado a un centro de atención de hemofilia debe ser realizado una vez estabilizado el paciente con la administración de plasma. Confirmación diagnóstica ¿Cuáles son los criterios de confirmación diagnóstica? El diagnóstico de hemofilia debe basarse en una anamnesis exhaustiva, asociada necesariamente a exámenes de laboratorio generales y específicos. • Un 30% de los enfermos no presentan antecedentes familiares conocidos. • Antecedente de hemorragia exagerado ya sea interno o externo, desproporcionado a la

magnitud del trauma que lo generó. • Hemartrosis, es el evento más frecuente y, representa el 65% a 80% de todas las hemorragias

en el paciente hemofílico • Hematomas y equimosis de diversa cuantía, no siempre relacionado con traumas. • Hematuria sin diagnóstico asociado evidente. Es frecuente que una persona con hemofilia a

partir de la edad escolar presente por lo menos un episodio al año de hematuria silenciosa. En la gran mayoría de los casos se considera "espontánea" sin poder demostrar origen evidente.

La confirmación diagnóstica de hemofilia es responsabilidad del hematólogo. Exámenes a realizar: 1. Tiempo de tromboplastina parcial activado (TTPK o TTPA).

Puede ser normal o prolongado en hemofilia A y B leves. Un TTPK prolongado se observa también en portadoras de hemofilia, enfermedad de von Willebrand, déficit de otros factores de coagulación de la vía intrínseca y común, presencia de un inhibidor que incluye el anticoagulante lúpico. Existen otras condiciones que alteran este examen y que pueden llevar a una sospecha diagnóstica equivocada, como es el manejo inadecuado de la muestra de sangre en la fase pre, intra y post analítica que resulta falso positivo y que debe descartarse.

2. Tiempo de protrombina (es normal). 3. Cuantificación de factor VIII y IX es la prueba que permite diagnóstico de confirmación. 4. El tiempo de sangría es normal. NO se debe realizar tiempo de sangría de IVY o de Ducke si

existe clínica sugerente de hemofilia, púrpura, trombocitopenia o durante un proceso hemorrágico.

5. Hemograma con recuento de plaquetas. 6. Exámenes de laboratorio para identificar otras coagulopatías: deben ser solicitados por

médico hematólogo especialista. Los exámenes deben realizarse en el centro de referencia nacional para diagnóstico de coagulopatías congénitas (red pública), u otro centro calificado. Se recomienda además estudiar a los hermanos, primos y tíos por línea materna, especialmente cuando la hemofilia es leve (porque puede cursar en forma asintomática).

Page 50: Examen final 20460

42.- Las características ante la sospecha en un recién nacido a término de ictericia fisiológica son:

a) Inicia entre el segundo y tercer día de vida, tiene una duración menor de 10 días y

la concentración máxima es de 12 mg/dl b) Inicia el primer día de vida, tiene una duración menor de 10 días y la concentración

máxima es de 20 mg/dl c) Inicia entre el segundo y tercer día de vida, tiene una duración mayor de 10 días y la

concentración máxima es de 12 mg/dl. d) Inicia a la semana de nacimiento y dura una semana más.

Manual CTO pediatría, 7° edición p. 1306. La ictericia fisiológica del recién nacido inicia entre el segundo y tercer día de vida, tienen una duración entre 5 y 7 días y alcanza una concentración máxima de 12 mgdl a los 2 ó 4 días. En cambio la ictericia no fisiológica inicia dentro de las primeras 24 horas de vida, tiene una duración superior de 10 a 15 días, la bilirrubina total en RNT es mayor de 12 mg/dl o mayor de 14 mg/dl en RNPT, el incremento de bilirrubina es mayor de 5 mg/dl en 24 horas y la bilirrubina directa es mayor de 1 mg/dl. 43.- Se trata de un recién nacido que cursa con lesión del plexo braquial que deja el brazo en “posición de mesero”, en un parto distócico. ¿Esta lesión es provocada a nivel de?

a) Cervicales 5-6- +/- 7 b) Cervicales -6-7 T1 c) Cervicales 6-7-8 d) Cervicales 7-8 T1

La parálisis del plexo braquial de los neonatos se clasifica de acuerdo con la localización anatómica y el tipo de lesión. Las más frecuentes son las lesiones de la porción superior del plexo. A pesar de que se han descrito los factores de riesgo asociados con la lesión, la incidencia global de este trastorno ha permanecido estable durante las últimas tres décadas. La verdadera tasa de recuperación completa después de una parálisis del plexo braquial es aún tema de discusión. El conocimiento de la clasificación de la lesión de un lactante determinado puede ayudar a predecir su pronóstico a largo plazo. El examen físico meticuloso es esencial para arribar al diagnóstico, evaluar las enfermedades asociadas, determinar el plan terapéutico y establecer la necesidad de derivación. En general no están indicados otros exámenes complementarios. El tratamiento inicial para el lactante con parálisis del plexo braquial consiste en asesoramiento, fisioterapia o terapia ocupacional y observación clínica del lactante. Un porcentaje de los niños afectados requiere una intervención quirúrgica u otros abordajes terapéuticos, como por ejemplo inyecciones de toxina botulínica. La lesión que afecta el tronco superior (raíces C5 y C6) y en ocasiones, el tronco medio (raíz C7). Los grupos musculares afectados por esta lesión son los rotadores externos y los abductores del hombro, los flexores del codo, los supinadores del antebrazo y a menudo los extensores de la muñeca. Esto genera el fenotipo clásico conocido como postura en “propina de mesero”, con aducción y rotación interna del hombro, extensión del codo, pronación del antebrazo y flexión de la muñeca. En esta lesión, además del compromiso del músculo bíceps, cuando el músculo tríceps está débil o ausente, el codo puede estar parcialmente flexionado. Fenotipo clásico asociado con la lesión superior del plexo braquial.

Page 51: Examen final 20460

Referencias bibliográficas 1. Borrero JL. Obstetrical Brachial Plexus Paralysis. 2nd ed. Lake Mary, Fla: Design and Print Progressive Communications; 2007 2. Seddon HJ. Three types of nerve injury. Brain. 1943:238–288 3. Rouse DJ, Owen J, Goldenberg RL, Cliver SP. The effectiveness and costs of elective cesarean delivery for fetal macrosomia diagnosed by ultrasound. JAMA. 1996;276:1480–1486 4. Pollack RN, Buchman AS, Yaffe H, Divon MY. Obstetrical brachial palsy: pathogenesis, risk factors, and prevention. Clin Obstet Gynecol. 2000;43:236–246 5. al-Qattan MM, Clarke HM, Curtis CG. Klumpke’s birth palsy. Does it really exist? J Hand Surg [Br]. 1995;20:19–23 44.- Masculino de 11 años acude a consulta externa por presentar fiebre de 8 hrs. de evolución, malestar generalizado, presenta erupción vesicular generalizada la cual inicia en tronco, se extiende a cara y miembros pélvicos acompañado de prurito. E.F.: presenta lesiones vesiculosas de predominio en tronco, escasas en cara y extremidades, de tipo macular, papular y vesicular. El diagnóstico más probable es:

a) Roseola b) Herpes Simple c) Varicela d) Rubeola

Varicela El virus varicela-zoster (VVZ) puede producir 2 enfermedades: la varicela que resulta de la infección primaria por el virus y el herpes zoster que se produce por su reactivación. El VVZ pertenece a la familia de herpesviridae con los que comparte la característica de persistir en el organismo luego de la infección primaria, pudiendo posteriormente reactivarse cuando por cualquier causa se produce una depresión de la inmunidad celular. Transmisión: Persona a persona, por vía aérea y contacto. Epidemiología La varicela es una enfermedad cosmopolita, muy contagiosa, frecuente en niños en quienes es habitualmente benigna. Es una afección endémica con períodos en que se torna epidémica. Prácticamente todos los individuos se infectan en el curso de su vida, estimándose que en el área urbana 90 % de los mayores de 30 años y casi todos los mayores de 60 tendrían anticuerpos frente a VVZ. Se contagia por contacto directo con las lesiones cutáneas y por inhalación de secreciones respiratorias que contienen el virus. Incidencia: Invierno y primavera. Periodo de incubación: 7 (10) a 21 (28) días. Periodo de contagiosidad: < 48 hrs. al inicio del exantema, hasta la presencia de vesículas. Infección latente. Clínica El período de incubación dura de 7 (10) a 21 (28) días y es asintomático. Le sigue el período prodrómico, que dura de horas a 2 días, durante el cual los síntomas son inespecíficos: fiebre poco elevada, cefaleas, anorexia, vómitos. El período de estado se caracteriza por la aparición de la erupción cutáneo-mucosas constituida por lesiones máculo-pápulo eritematosas que en 24 horas se transforman en vesículas. Las

Page 52: Examen final 20460

mismas son de tamaño y forma variable, tensas, con contenido líquido claro, simulando "gotas de rocío" y están rodeadas de una aréola rosada. Posteriormente pierden tensión y el contenido se hace turbio, aunque no purulento. Posteriormente se convierten en costrosas y en 4 a 6 días más se desprenden. Es característico de la varicela el polimorfismo lesional regional. Ello se debe a que las lesiones aparecen en brotes sucesivos en relación a la viremia y por lo tanto se encuentran en distintos estadíos evolutivos. Estas lesiones predominan en cabeza y tronco, por lo que la erupción de esta enfermedad es centrípeta. En el paciente inmunocompetente a los 5 días del comienzo de la erupción ya no presenta lesiones nuevas.

Reseña bibliográfica:

1) La revue du praticien. Infection par le virus de la varicelle pendant la grossesse. Lecuru F, Bernard JP, Parrat S, Taurelle R. 1995. N° 45: 1601-6.

2) Enfermedades infecciosas. Principios y práctica.Mandell, Douglas, Bennett. Editorial Médica Panamericana. 4° edición.

45.- Al realizar una prueba de apgar en un RN de 37sdg, usted recuerda los parámetros incluidos de esta prueba, los cuales son: a) Frecuencia cardíaca, reflejo pupilar, respiración, tono, color b) Tensión arterial, Tono, color, respiración, irritabilidad. c) Tono, color respiración, reflejos, frecuencia cardiaca d) Reflejos, Temperatura, color, frecuencia cardiaca. Este test lleva el nombre por Virginia Apgar, anestesióloga, especializada en obstetricia, quien ideó el examen en 1952 en el Columbia University´s Babies Hospital

Los cinco criterios del Apgar son:

0 puntos 1 punto 2 puntos Acrónimo

Color de la piel todo azul extremidades azules normal Apariencia

Frecuencia cardíaca

no posee <100 >100 Pulso

Reflejos e irritabilidad

sin respuesta aestimulación

mueca / llanto débil aser estimulado

estornudos / tos / pataleoal ser estimulado

Gesto

Tono muscular ninguna alguna flexión movimiento activo Actividad

Respiración ausente débil o irregular fuerte Respiración

Literatura

Apgar, Virginia (1953). «A proposal for a new method of evaluation of the newborn infant» Curr. Res. Anesth. Analg.. Vol. 32. n.º 4. pp. 260–267. PMID 13083014.

Page 53: Examen final 20460

Finster M; Wood M. (abril de 2005). «The Apgar score has survived the test of time» Anesthesiology. Vol. 102. n.º 4. pp. 855-857. PMID 15791116.

Casey BM; McIntire DD, Leveno KJ (15 de febrero, 2001). «The continuing value of the Apgar score for the assessment of newborn infants» N Engl J Med.. Vol. 344. n.º 7. pp. 467-471. PMID 11172187.

46.- Masculino de 43 años refiere pérdida de fuerza progresiva en miembros inferiores, de unos 7 días de evolución, dolores musculares y parestesias en pies y manos, como único antecedente refiere haber cursado con cuadro de faringoamiglatistis hace un par de semanas con mejoría al tratamiento. En su exploración se aprecia únicamente debilidad en los cuatro miembros, de predominio distal y en miembros inferiores, y arreflexia generalizada. El diagnóstico más probable de éste paciente es: a) Una miastenia gravis. b) Un proceso expansivo medular cervical. c) Una polimiositis. d) Una polirradiculoneuritis aguda. SINDROME DE GUILLAIN-BARRÉ El síndrome de Guillain-Barré fue descripto en el año 1834 por Ollivier y Wardrop, Landry, en 1859 hablo de una paralisis ascendente que seguía con insuficiencia respiratoria y muerte. Osler, en 1892 hablo de una polineuritis febril y Guillain-Barré y Strohl , en 1916 reconocieron una polineuritis benigna con disociación albuminocitológica. Tambien llamada polirradiculoneuritis aguda, caracterizada por alteraciones sensitivas, motoras, arreflexia y disociación albúmino citológica en el LCR (líquido cefaloraquideo). Es una enfermedad que puede aparecer a cualquier edad afectando por igual a ambos sexos. La característica neuropatológica corresponde a focos de desmielinización focal y segmentaria de los nervios periféricos en toda su longitud, comprometiendo incluso las raices. Los nervios del sistema nervioso periférico presenta infiltrados inflamatorios perivasculares, y es ahí donde se observan las lesiones mielínicas. Continúa en boga la teoría inmunológica como la responsable de estas polirradiculoneuritis inflamatoria. Muchas veces la enfermedad surge luego de un proceso de vacunación, aplicación de suero o de una infección vírica ( generalmente trancurre de 7 a 25 días entre el proceso que se cree causal y el comienzo de los síntomas), Los organismos que se han visto con más frecuencia relacionados con este transtorno fueron el virus de Epstein-Barr, el citomegalovirus, el micoplasma y el virus de la hepatitis B. La enfermedad se presenta, en su evolución, en 3 (tres) fases

1- FASE DE EXTENSIÓN 2- FASE DE ESTADO 3- FASE DE RECUPERACIÓN

1- FASE DE EXTENSIÓN El paciente comienza a sentir sensación de "hormigueo, las piernas se me duermen", "siento calambres en ambas piernas"... las parestesias en las extremidades y el deficit motor son comunes, pero más frecuente es que aparezca el compromiso motor. Se afectan los miembros inferiores precozmente y la extensión suele ser ascendente, de forma progresiva, en un tiempo estimado de 1 semana, 1 semana y media. Hay que tener mucho cuidado con estos pacientes por la posible aparición del compromiso de los músculos respiratorios o algun trastorno en la deglución (En estos casos son pacientes pasibles de internación y de cuidados intensivos).

Page 54: Examen final 20460

2- FASE DE ESTADO Es donde aparecen la mayoría de los signos neurológicos. El compromiso motor puede llegar al extremo, puede aparecer una tetraplejía con compromiso respiratorio y deglutorio. Otros casos pueden limitarse al compromiso de los miembros inferiores. Aparece arreflexia generalizada. Suceden parestesias, se compromete la sensibilidad tactil y profunda (por compromiso de los cordones posteriores, vía de Goll y Burdach). Toma nervios craneales, aparece una parálisis facial que puede ser uni o bilateral, puede verse parestesia trigeminal. Cuando toma los pares craneales IX-X (glosofaringeo y neumogástrico) aparecen los trastornos de la deglución. Cuando toma el sistema nervioso vegetativo se ve que el paciente cursa con hipertensión, trastornos en la repolarización cardíaca (visibles en el ECG), alteraciones en la glucemia, hiponatremia, SIHAD (secreción inapropiada de hormona antidiurética) o bradicardias (que puede llevar hasta un paro cardíaco) Un tema muy importante es el estudio del líquido cefalorraquídeo (LCR) que muestra una elevación de la proteinorraquia que no se acompaña de hipercitosis (aumento de las células en el LCR), esto es lo que se denomina disociación albuminocitológica. En el 80% de los casos la elevación corresponde a las gammaglobulinas. Esta disociación no es inmediata, por lo que puede tardar un tiempo en aparecer. (esto tiene que ver con que las punciones del LCR en un período temprano pueden dar normales). 3- FASE DE RECUPERACIÓN Los signos neurológicos regresan en orden inverso al que aparecieron luego de 1 a 2 semanas de estabilidad en los síntomas. Estos cambios lo hacen en forma más lenta en relación a como aparecieron. El tiempo de regresión puede durar desde semanas hasta meses. La recuperación puede ser total o dejar secuelas motoras o sensitivas hasta en un 20% de los casos. DIAGNOSTICO DIFERENCIAL Esta enfermedad es una de las polineuropatias más frecuentes, la que más rápido evoluciona y que tiene un alto potencial mortal. Cualquier polineuropatía, de evolución ascendente, que ponga en peligro de vida al paciente por compromiso respiratorio en el plazo de unos pocos días, nos deberá hacer sospechar de un sindrome de Guillain-Barré. Pearce, j.m.s. Barré-Liéou “syndrome”. J Neurol Neurosurg Psychiatry 2004: 75, 319. -Pritchard, J.; Hughes, R.A.C. Guillain-Barré syndrome. The Lancet 2004; 363: 2186-88. -Thiébaut, F.J.A. Barré (1880-1967). J Neurol Sci 1968; 6: 381-2. 47.- Una mujer de 28 años con antecedente de migraña se presenta con cefalea constante de tipo compresivo relacionada con fotofobia y temperatura de 39.1°C. Por medio de punción lumbar se extrajo líquido cefalorraquídeo con una concentración de proteínas de 62 mg/100 ml, glucosa de 76 mg/100 ml y 26 mononucleares/mm3. Dos días después la paciente se encuentra afebril y sólo tiene cefalea al levantarse ¿Cuál de los siguientes es el diagnóstico más probable de esta persona en este momento? a) Migraña b) Meningitis bacteriana c) Meningitis aséptica d) Cefalea posterior a punción lumbar Allen R. M. MMS Medicina Interna. 5ª. Edición. National Medical Series. Mc. Graw Hill. 2006. (capítulo 8 VA 1 a (1) (a)-(b); capítulo 11IV C 2). Lo más probable es que esta enferma haya tenido meningitis aséptica de resolución espontánea y sufra cefalea postural posterior a punción lumbar (LP) al despertarse. Otros síntomas pueden ser náuseas, visión borrosa, acúfenos y vómitos. La disminución de la presión de líquido cefalorraquídeo (LCR) puede ser la

Page 55: Examen final 20460

causa de cefalea relacionada con punción lumbar Las opciones terapéuticas son reposo en cama, analgésicos y parche sanguíneo epidural lumbar que en parte puede taponar un desgarro en la duramadre. Aunque la paciente tiene antecedente de migraña, es inusual que las cefaleas vasculares se presenten sólo en bipedestación. La meningitis bacteriana es una complicación rara de LP. Sin embargo, como la paciente está afebril y sólo ha tenido cefalea en posición erecta, este diagnóstico es improbable. Del mismo modo lo es que la cefalea se deba a meningitis aséptica persistente. Los enfermos pueden presentar hipersensibilidad y tensión de los músculos cervicales después de meningitis aséptica. Esto puede confundirse con meningismo persistente. Sin embargo, es más probable que esta persona sufra cefalea como resultado de complicaciones de LP. 48.- Se trata de masculino de 39 años que sufre colisión automovilística de frente, poli contundido y con fx expuesta de fémur llama la atención una equimosis periorbitaria por el trauma facial, este signo traduce fractura de: a) Piso posterior b) Piso medio c) Piso anterior d) Macizo facial

En la valoración ocular se deben tener en cuenta las heridas de los tejidos blandos de párpado, córnea y conjuntiva. El signo de mapache (equimosis periorbitaria bilateral) se encuentra frecuentemente asociado a las fracturas de la base anterior del cráneo. (Figura 3).

Figura 3. Equimosis periorbitaria luego de trauma facial contundente.

En las heridas del párpado la localización es de vital importancia y es necesario anotar cuidadosamente el sitio, si se compromete el borde libre o si se lesiona el lugar de paso de la vía lagrimal.

El tono ocular ayudará a definir si hubo herida abierta del ojo.

La presencia de enoftalmo (hundimiento del globo ocular) alertará sobre la posibilidad de una herida abierta del ojo o una fractura de las paredes orbitarias. El hipoftalmos (descenso del ojo en el eje vertical) puede estar relacionado con fractura del piso de la órbita o con fractura en el sitio donde se insertan los ligamentos suspensorios del globo ocular (tubérculo de Whitnall, ligamento de Lookwood).

H. Ric Harnsberger. 2004. ISBN 848174753x Translation of PocketRadiologist - Head & Neck: Top 100 Diagnoses.

Page 56: Examen final 20460

49.-Se trata de femenino de 17 años tiene exantema cutáneo rojo difuso; fiebre de 39.4°C y diarrea leve acuosa. En fechas recientes tuvo infección de garganta por la que se le administró sulfametoxazol. Comenzó sus menstruaciones hace tres días. En la exploración física se encuentran cambios eritematosos difusos de la piel con descamación temprana. La boca y las conjuntivas están eritematosas. La probabilidad diagnóstica de esta paciente es:

a) Bacteriemia por Salmonella b) Síndrome de choque tóxico (TSS) c) Tuberculosis d) Mononucleosis por virus de Epstein-Barr

Allen R. M. MMS Medicina Interna. 5ª. Edición. National Medical Series. Mc. Graw Hill. 2006. (Capítulo 8 VII D 1 a, 2 b). La bacteriemia por Salmonella, el síndrome de choque tóxico (TSS), la tuberculosis y la mononucleosis de Epstein-Barr pueden acompañarse de fiebre, pero la presencia de exantema descamativo difuso sugiere TSS, reacción farmacológica grave (p. ej., síndrome de Stevens-John-son); enfermedad de Kawasaki, o escarlatina. El exantema cutáneo relacionado con salmonelosis es muy sutil y evanescente (manchas de color rosa). La tuberculosis no se caracteriza por afección cutánea difusa y de mucosas o diarrea acuosa. Si bien la alergia al sulfametoxazol puede producir eritema cutáneo y de mucosas, no causa diarrea. 50.- Paciente masculino de 58 años de edad con antecedentes de DM tipo II e HTAS, tratado con amlodipino e hipoglucemiantes orales(glimeprida). Hace unas 2 semanas presentó un cuadro de lumbalgia por el que recibió tratamiento con diclofenaco. Actualmente trtatado con TMP SMZ por IVU. Acude a urgencias del hospital por presentar desde hace 48 horas una erupción cutánea confluyente en tronco y extremidades. A la exploración se observan lesiones maculares eritematovioláceas muy extensas (>70% de la superficie cutánea) sobre las cuales aparecen ampollas y erosiones. Existe afectación (lesiones erosivo-costrosas) de mucosa labial, oral y conjuntiva. El signo de Nikolsky es positivo. El diagnóstico probable es: a) Penfigoide ampolloso. b) Pénfigo vulgar. c) Pénfigo paraneoplásico. d) Necrolisis tóxica epidérmica. La necrólisis epidérmica tóxica (NET) y el síndrome de Stevens-Johnson (SSJ) constituyen un espectro de la misma enfermedad, compartiendo aspectos etiológicos, patogenéticos, histológicos y terapéuticos. Casi todos (si no todos) los casos son inducidos por fármacos. NET y SSJ pueden distinguirse clínicamente del eritema multiforme, que debe considerarse una enfermedad distinta. La patogenia de la NET y del SSJ es poco conocida, pero se acepta que intervienen reacciones inmunológicas y un mecanismo final de apoptosis masiva de queratinocitos epidérmicos. El tratamiento consiste en la retirada del fármaco causal y medidas de soporte, evitando la administración de corticosteroides. Se han descrito tratamientos que pretenden detener la evolución del cuadro, entre ellos ciclofosfamida, ciclosporina, plasmaféresis, pentoxifilina e inmunoglobulinas i.v. El cuadro típico de NET, descrito por Lyell (8), se caracteriza por la aparición brusca, tras un pródromo «catarral», de lesiones eritematosas, dolorosas, inicialmente distribuidas de forma simétrica en cara y tronco, aunque luego se puedan extender hacia las partes acras. Las lesiones comienzan siendo máculas eritematosas, pero pronto aparece el signo característico de la enfermedad: la necrosis y desprendimiento de la epidermis que produce ampollas flácidas

Page 57: Examen final 20460

y signo de Nikolsky con despegamiento de amplias zonas epidérmicas que dejan erosiones exudativas. La extensión de este despegamiento es uno de los factores pronósticos principales. Un pequeño porcentaje de enfermos presenta sólo eritema confluente y zonas erosivas, pero el 90% tienen además lesiones aisladas, salpicadas en la proximidad de las erosiones. Éstas son máculas de borde mal definido, con forma irregular, y pueden tener un centro más oscuro o ampolloso. Casi todos los pacientes presentan lesiones mucosas , incluyendo erosiones dolorosas orales y faríngeas, lesiones oculares (que conllevan un riesgo elevado de secuelas) y genitales. El daño de otros epitelios, como el respiratorio o digestivo, y las complicaciones de la insuficiencia cutánea aguda agravan el cuadro Signo de NIKOLSKY: desprendimiento de las capas de la piel, aparentemente sana, por efecto de la presión tangencial del dedo, con una ventosa o con un esparadrapo.

1. Avakian R, Flowers FP, Araújo OE, Ramos-Caro FA. Toxic epidermal necrolysis: a review. J Am Acad Dermatol 1991; 25:69-79. 2. Roujeau J-C, Chosidow O, Saiag P, Guillaume J-C. Toxic epidermal necrolysis (Lyell syndrome). J Am Acad Dermatol 1990;23:1039-58. 3. Champion RH. Disorders of blood vessels. En: Champion RH, Burton JL, Ebling FJG, eds. Textbook of dermatology, 5.a ed. Oxford: Blackwell Scientific Publications; 1992. p. 18.34-18.38. 4. Fritsch PO, Elias PM. Erythema multiforme and toxic epidermal necrolysis. En: Fitzpatrick TB, Eisen AZ, Wolff K, Freedberg IM, Austen KF, eds. Dermatology in general medicine, 4.a ed. Nueva York: McGraw-Hill; 1993. p. 585-600. 5. Böttiger LE, Strandberg I, Westernholm B. Drug induced febrile mucocutaneous syndrome. Acta Med Scand 1975; 198:226-33. 51.- Masculino de 25 años que ingresa a UTI por fractura de la base del cráneo. Aproximadamente 18 hr después de la lesión desarrolla poliuria, osmolalidad urinaria de 150 mOSm/L y osmolalidad sérica de 350 mOsm/L. Se detienen el manejo de líquidos intravenosos, pero 3 horas después las osmolalidades urinaria y sérica siguen sin cambios. Se administran 5 unidades de vasopresina, lo que eleva la osmolalidad urinaria a 300. El diagnóstico más probable es: a) Diabetes insípida central b) Intoxicación por agua c) Sobrecarga de solutos d) Síndrome de secreción inapropiada de hormona antidiurética

La alteración en la diabetes insípida central es el resultado de un déficit de la hormona antidiurética (vasopresina), que es la encargada de limitar la excreción de agua a nivel renal.

Page 58: Examen final 20460

Lo singular de esta hormona es que el hipotálamo la produce y luego es almacenada hasta ser liberada en el flujo sanguíneo por la hipófisis posterior.

La diabetes insípida neurogénica está determinada por un desorden del metabolismo del agua, caracterizado por la excreción de considerables volúmenes de orina hipotónica, que obliga a la ingestión de gran cantidad de líquidos para evitar la hiperosmolaridad plasmática y la deshidratación, causado fundamentalmente por una ausencia o producción deficiente de la vasopresina u hormona antidiurética (ADH). A esta entidad también se le conoce como diabetes insípida hipofisaria, central, craneal o hipotalámica. Además, también existe la denominada diabetes insípida nefrogénica o resistente a la vasopresina, en la que no se logra concentración urinaria significativa, en presencia de cantidades adecuadas de ADH y que responde favorablemente a la terapéutica con diuréticos tiazídicos.3

Las clásicas manifestaciones son la poliuria con nicturia y la polidipsia (siempre que el centro de la sed hipotalámico no esté alterado). Si la excreción de agua excede el aporte se producirá hipovolemia e hipotensión. Si el aporte de fluidos es adecuado puede que la natremia y la osmolaridad sean normales, en caso contrario se desarrollará una deshidratación hipertónica. La clínica suele ser precoz en la diabetes insípida central (3 h a pocos días postraumatismo o postquirúrgicos) pudiendo ser transitoria en la mayoría de los casos o permanente (30-40%). El paciente con diabetes insípida central describe la instauración brusca de poliuria y de polidipsia. Los volúmenes urinarios elevados son frecuentes, la nicturia está presente, el paciente suele manifestar astenia por trastornos del sueño, puede aparecer con buen estado general si las causas no han destruido la neurohipófisis. Con el mecanismo de la sed intacto y una ingesta apropiada de agua el paciente puede mantener una adecuada natremia y osmolaridad sanguínea. En otros casos existe poliuria hipoosmótica con aumento de la osmolaridad plasmática.

La incapacidad para obtener agua libre que ocurre frecuentemente tras un traumatismo craneoencefálico o anestesia puede producir una hipernatremia que amenace la vida. Una adecuada monitorización del volumen urinario y la osmolaridad plasmática en estos pacientes evitará esta complicación, raramente se produce también alteración del mecanismo de la sed del hipotálamo. Tras la cirugía de la neurohipófisis existe un período de 4-5 días de poliuria seguidos de otro período de 4-5 días con oliguria antes de que la poliuria y la polidipsia de la diabetes insípida central sobrevengan. Esta respuesta trifásica es paralela a la respuesta de las células neurosecretoras al daño o la lesión.

Cuadro de poliuria (3litros) hipotónica que resulta del déficit en la producción o acción de la hormona antidiurética, total o parcial. Excreción de orina diluida (hipoosmótica). Diagnóstico diferencial de Hipernatremia y/o síndromes poliúricos.

Tipos:

–Central.

–Nefrogénica.

–Polidipsia primaria (diagnóstico diferencial).

Clínica

1. Poliuria (>3litros/día o>50ml/kg/día).

2. Sed y polidipsia, si no hay alteración del centro de la sed.

3. Orina hipotónica (densidad<1005,osmolaridad<300mmol/kg).

Page 59: Examen final 20460

4. Osmolaridad y sodio plasmáticos elevados: Clínica de Hipernatremia: obnubilación, debilidad, irritabilidad, movimientos anormales, convulsiones, coma, muerte.

5. Corrección brusca: edema cerebral, herniación.

DI Central

Déficit de producción de ADH en el sistema neurohipofisario.

•Inicio brusco.

Causas:

–Genética: Sd.deWolfram, autosómico recesivo (DIDMOAD) mutación en gen WFSI

–Congénita: Defectos en la línea media craneal.

–Adquirida:

–TCE.

–Neoplasias: germinomas, craneofaringiomas, pinealoma, adenomahipofisario.

–Infiltrativo/autoinmune: HistiocitosisdecéldeLangerhans; hipofisitislinfocitaria, neurosarcoidosis.

–Drogas: Etanol, fenitoína,agentes alfa adrenérgicos, corticoides.

–Infecciones: meningitis, encefalitis.

–Neurocirugía.

–Idiopática. 30–40%probable origen autoinmune.

–EMBARAZO (VASOPRESINASA).

Diagnóstico

•Constatar la poliúria y cuantificarla.

•TESTDERESTRICCIÓNHÍDRICA:

–Periodo libre de ingestión hídrica, si poliuria leve inicio la anoche anterior, si es intensa se iniciará de día.

–Cada hora se de termina peso, TA, FC, Osmp, Vol diuresis y Osmo. Cada dos horas Nap, Osm pe Ion es en orina.

–La prueba se mantiene un total de 17 horas hasta que se produzca una de estas 4 situaciones:

–Osmo>800m Osm/kg

–Pérdida de peso inicial>5%

Page 60: Examen final 20460

–Osmo estabilizada en 3 determinaciones (diferencias menores de 30mOsm/kg)

–Osmp>300 hiponatremia de 148mmol/L

•Medición de ADH plasmática:

–Se debería medir en plasma en muestra basa ly al terminar e test de deshidratación, antes de administrar la DDAVP.

–Se excluye DI central si los niveles de ADH se elevan adecuadamente en respuesta al aumento de Osmp.

–Se excluye DI neurogénica si hay una apropiada elevación de la Osmo, cuando los niveles de ADH aumentan.

–Útil para diferenciar formas parciales de DIN y DIC.

Referencias Bibliográficas

1. Baylis PH. Investigation of suspected hypothalamic diabetes insipidus. CLin Endocrinol 1995;43:507-10.

2. Buonocore CM, Robinson AG. The diagnosis and management of diabetes insipidus during medical emergencies. Endocrinol Metabol Clin North Am 1993;22:411-23.

3. Lewis DJM, Thomas JP. Treatment of nephrogenic diabetes insipidus. N Engl J Med 1986;315:1292-3.

4. Singer I, Oster JR, Fishman LM. The management of diabetes insipidus in adults. Arch Intern Med 1997;157:1293-1301.

5. tment of nephrogenic diabetes insipidus. N Engl J Med 1986;315:1292-3. 6. Singer I, Oster JR, Fishman LM. The management of diabetes insipidus in adults. Arch

Intern Med 1997;157:1293-1301.

52.- Un varón de 65 años lee en el periódico que el antígeno prostático específico es una buena prueba de investigación para el cáncer y pide a su internista que se la haga. La prueba revela un aumento de antígeno prostático específico de 10.4 ng/ml. El tacto rectal revela una próstata de tamaño normal, pero en la ecografía se encuentra un área hipoecoica pequeña que mide 5x7 mm en el lóbulo derecho. ¿Cuál de las siguientes medidas es el siguiente paso apropiado? a) Repetir el análisis de antígeno prostático específico en tres meses para verificar si hay

aumento ulterior b) Practicar biopsia transrectal del área anormal encontrada en la ecografía c) Comenzar el tratamiento con leuprolida de depósito d) Efectuar una tomografía computadorizada del retroperitoneo, la pelvis y la próstata Allen R. M. MMS Medicina Interna. 5ª. Edición. National Medical Series. Mc. Graw Hill. 2006. (capítulo 4 X C 4 a (3), b (1)). El antígeno prostático específico es una prueba que puede usarse para detección de cáncer de próstata. Sin embargo, los valores de éste también pueden estar un poco altos en la hipertrofia prostática benigna. Una ecografía transrectal puede identificar lesiones pequeñas no palpables en la exploración rectal. Si los pacientes tienen PSA alto y se confirmó en la ecografía un área anormal, se puede practicar biopsia por vía transrectal bajo guía ecográfica. En general, a los varones con cáncer de próstata se les estudia en busca de metástasis con gammagrama óseo; radiografía de tórax; tomografía computarizada de retroperitoneo y pelvis; o resonancia magnética de retroperitoneo y pelvis con énfasis en la próstata, acompañados por estudios de laboratorio. El carcinoma prostático metastásico puede tratarse con leuprolide, un agonista de la hormona liberadora de hormona

Page 61: Examen final 20460

luteinizante que suprime la producción testicular de testosterona. El tratamiento con leuprolide equivale a orquiectomía o a terapéutica con estrógenos en cáncer prostático o metastásico. 53.- Se trata de masculino de 70 años que consulta por pirosis, disfagia leve ocasional y episodios de regurgitación nocturna desde hace 15 días. Comenta que desde hace 2 años viene presentando por ocasiones pirosis y regurgitación. Se reporta endoscopia alta con esofagitis erosiva grave. El tratamiento farmacológico más adecuado entre los siguientes, es: a) Antagonistas de los receptores H2. b) Sucralfato. c) Inhibidores de la bomba de protones. d) Tratamiento combinado con anti-H2 y sucralfato

Se ha demostrado que existe una relación directa entre la duración de la supresión del acido gástrico y la menor acidez del esófago. Los inhibidores de la bomba de protones (IBP) son los agentes preferidos para la curación de las lesiones agudas y para la mantención de remisión. Los bloqueadores H2 presentan como única ventaja su rapidez de acción, pero su potencia es menor. Su uso de rutina asociado con los IBP no se recomienda, ya que los IBP actúan sobre las bombas de ácido activadas y cualquier inhibición de la secreción por otro agente retarda su máximo efecto Los antiácidos son útiles para síntomas ocasionales y su uso no se contrapone con los IBP. De no existir factores significativos modificables en los hábitos o en la anatomía, la ERGE debe considerarse frecuentemente una patología crónica. No existe evidencia de peso que contraindique el uso crónico de IBP. Los pacientes con esofagitis erosiva son los más susceptibles a desarrollar complicaciones que los en GERD (endoscopy negative Gastroesophageal reflux disease).Todos los IBP son útiles pero no son necesariamente iguales. Puede haber diferencias étnicas en el número de células parietales o polimorfismo del citocromo p450 con diferente metabolización de los IBP. La infección por Helicobacter pylori confunde cualquier intento de comparar IBP. La gastritis de predominio antral puede producir hipergastrinemia e hipersecreción. Si es de predominio corporal puede disminuir la producción de ácido. Los estudios comparativos deben hacerse en individuos sin esta infección. La duración del efecto de los IBP es importante. Muchas publicaciones demuestran la mayor duración de la acción de la acción del esomeprazol, pero clínicamente doble dosis de los otros IBP pueden tener resultados comparables. Katz y cols compararon los 5 IBP disponibles en pacientes con ERGE, todos resultaron en pH > 4 por al menos 8 a 10 h, pero la duración de acción del esomeprazol fue superior en el rango de los pH intragástricos entre 2 a 6. En los pacientes con enGERD el efecto de los IBP no es tan dramático, puede deberse a que no hay tantos elementos objetivos de juicio como la curación de las erosiones. Los IBP pueden usarse una vez al día, en la mañana y antes del desayuno. Los pacientes con daño extraesofágico o cuadros severos obtienen mejor efecto con doble dosis fraccionada (Antes de desayuno y cena). Esto mejora el control de la acidez nocturna. En los pacientes cuya pHmetría demuestre que persiste escape nocturno algunos favorecen agregar un antagonista H2 al acostarse. Rackoff y cols demostraron que 74% de los pacientes presentaban mejoría de los síntomas nocturnos. Una nueva formulación de liberación y absorción rápida de omeprazol también podría ser útil en estos casos. La ausencia de síntomas no significa ausencia de daño. Los casos graves (y por supuesto el Barrett) deben tener seguimiento endoscópico para asegurar su evolución. Un porcentaje significativo de pacientes sin Barrett continúan presentando RGE patológico y bajo pH intragástrico a pesar de IBP bien llevado con total remisión de síntomas. El primer objetivo es la remisión de los síntomas de reflujo clásico. Esto debe lograrse en la primera semana si la prescripción es adecuada. Las manifestaciones extraesofágicas tardan varios meses en controlarse, pero deben exhibir una mejoría rápida en las primeras semanas que confirme la buena orientación de las medidas indicadas. La estrategia terapéutica de comenzar el tratamiento en forma poco agresiva y escalarlo si no hay respuesta, me parece poco adecuada.

Page 62: Examen final 20460

Al paciente se le debe insistir que los fármacos son una parte importante del tratamiento pero no la única y que los cambios de hábitos y costumbres, la baja de peso, el ejercicio Etc, son claves para el éxito. Bibliografía 1.- Miner P, Katz P, Chen Y, Sostek M. Gastric acid control with esomeprazole, lansoprazole, omeprazole, pantoprazole, and rabeprazole: A five-way crossover study. Am J Gastroenterol 2003; 98: 2616-20. 2. Katz P, Miner P, Chen Y, Sostek M. Effects of 5 marketed proton pump inhibitors on acid suppression relative to a range of pH thresholds. Am J Gastroenterol 2004; 99: S34. 3.- Rackoff A, Agrawal A, Hila A, et al. Histamine-2 receptor antagonists at night improve GERD symptoms for patients on proton pump inhibitor therapy. Am J Gastroenterol 2004; 99: S18. 4.- Castell D, Goldlust B, Morelli G, et al. Omeprazole immediate-release oral suspension is more effective than pantoprazole delayed-release capsules in reducing nighttime gastric acidity in GERD patients. Am J Gastroenterol 2004; 99: S39. 5. Milkes D, Gerson L, Triadafilopoulos G. Complete elimination of reflux symptoms does not guarantee normalization of intraesophageal and intragastric pH in patients with gastroesophageal reflux disease (GERD). Am J Gastroenterol 2004; 99: 991-6. 54.- Femenino de 44 años cursa con dolor epigástrico que la despierta por la mañana y se presenta también cuando tiene hambre. El comer o el tomar antiácidos lo mejora. Se la realiza una endoscopia que revela una úlcera duodenal. La etiología más probable de esta úlcera ES: a) Uso de AINES b) Estrés c) Síndrome de Zollinger Ellison d) Infección por H. pylori La etiología de EUP

H. pylori o Predisposición a Ca gástrico o Incidencia

o 70 a 95% gastritis crónica activa o 90 a 100% úlcera duodenal o 70 a 90% úlcera gástrica o Aumenta 10% cada 10 años

Anti Inflamatorios No Esteroideos o 2 a 4% desarrollan complicaciones graves o 30% de muertes por EUP son atribuíbles a estos fármacos

Tabaquismo activo o Prob. predisposición a infecc. x H.p o por disminución factores defensivos

Page 63: Examen final 20460
Page 64: Examen final 20460

55.- Mujer de 60 años con antecedentes de cuadro respiratorio alto hace 3 semanas. Desde hace dos presenta vértigo de inicio súbito intenso y persistente, con varias horas de duración e inestabilidad a la marcha, no refiere tinitus ni hipoacusia. El diagnóstico clínico más probable es: a) mastoiditis. b) otoesclerosis. c) neuronitis vestibular. d) otitis media.

En la Neuritis vestibular, los vértigos son atribuidos a una infección viral del nervio vestibular. El nervio vestibular conduce la información relacionada con los movimientos de la cabeza, desde el oído interno hasta el cerebro. Cuando uno de los dos nervios vestibulares es afectado, se produce un desequilibrio entre ambos oídos, que finalmente es el causante del vértigo. Neuronitis vestibular es otro término usado para el mismo síndrome clínico. Los múltiples

Page 65: Examen final 20460

términos utilizados para la misma entidad clínica probablemente reflejan nuestra incapacidad para localizar con exactitud el lugar de la lesión. El termino neuritis implica daño del nervio vestibular y el de euronitis , daño de las neuronas sensoriales del ganglio vestibular. Existe actualmente evidencia de ambos. Hay también cierta evidencia de daño de origen viral del núcleo vestibular (Arbusow et al, 2000), es decir una segunda potencial neuronitis . Como las neuronas vestibulares son distintas de las neuronas cocleares en cuanto a ubicación espacial en el tronco cerebral, (así como las del ganglio vestibular) hace más probable la afección de el nervio vestibular en personas sin pérdida auditiva asociada. Sin embargo, si el nervio fue comprometido luego de la separación del nervio coclear, la neuritis sería aún un razonable mecanismo. En estudios de autopsia no fue posible determinar el lugar de la lesión, y hasta el presente se prefiere el término Neuritis

En la neuritis vestibular, el virus que habitualmente causa la infección, pertenece a la familia del Herpes virus, del mismo grupo que causa las lesiones en la boca (herpes labial), así como otros trastornos (Arbusow et al, 2000). Hay algunas controversias acerca de esta teoría, sin embargo, es limitada la evidencia de infección directa (Matsuo, 1986). Se piensa también que la neuritis vestibular puede ser causada por una franca disminución del flujo sanguíneo del sistema vestibular (Fischer, 1967). Sin embargo, la idea actual es que se trata de una inflamación, presumiblemente de origen viral, esto sería mucho más probable que la disminución en el flujo vestibular. Persisten sin embargo, algunos desacuerdos. (Fattori et al. 2003).

56.- Femenino de 43 años que desde hace 1 mes inicia con debilidad a nivel de cintura escapular y pelviana. En la piel se aprecia edema palpebral y coloración eritematoviolácea periorbitaria así como lesiones eritematodescamativas sobre prominencias ósea de dorso de manos. El diagnóstico más probable en esta paciente es: a) Dermatomiositis. b) Eritema polimorfo. c) Lupus eritematoso sistémico. d) Artritis reumatoide. DERMATOMOSISTIS Cuadro Clínico: - Alteraciones musculares. Vienen marcadas por la presencia de debilidad muscular aguda o subaguda (generalmente de inicio insidioso), simétrica y difusa, con preferencia por musculatura proximal de extremidades (cintura pélvica y escapular), tronco y cuello. En la mayoría de los casos es indoloro. Con el tiempo, desarrollan atrofia, contracturas y disminución de los reflejos. - Alteraciones cutáneas. La más frecuente en la DM es una erupción cutánea eritematoviolácea que afecta a cuello, cara y tórax. Es característico también, el eritema heliotropo (en párpados), que puede extenderse a otras zonas fotoexpuestas), las pápulas de Gottron (localizadas en los nudillos), telangiectasias periungueales, a veces ulceración dérmica y calcinosis (fundamentalmente en la DM infantil). - Articulares. Artralgias, artritis transitorias, no erosivas, con tendencia a la simetría. - Otras. Afectación cardiaca variable (alteración ECG, arritmia, miocarditis), pulmonar (fibrosis intersticial asociada con anti Jo-1), renal (muy rara), fenómeno de Raynaud. Diagnóstico: - Analítica: aumento de VSG y de enzimas musculares, (CPK, aldolasa, GOT, GPT, LDH). La CPK es la más sensible y la que guarda una mejor correlación clínica con la actividad de la enfermedad y la valoración de recaídas. El FR es + en 20% y

Page 66: Examen final 20460

ANA es + en 10-30%. Si la destrucción muscular es intensa, puede producir mioglobinuria. - Destacan anticuerpos: • anti-Jo1: en casos de PM asociado a neumonitis intersticial (síndrome antisintetasa-miosistis, fibrosis pulmonar, artritis no erosiva y fenómeno de Raynaud). • anti-PM1 o PM-Scl: asociación con esclerodermia. • anti-Mi, en DM. • antimioglobina. BIBLIOGRAFÍA RECOMENDADA

Klippel JH, Stone JH, Crofford LJ, White PH, editors. Primer on the rheumatic diseases. 13th ed. New York: Springer-The Arthritis Foundation; 2008.

Martínez-Elizondo P, editor. Introducción a la Reumatología. 4a ed. México: Colegio Mexicano de Reumatología A.C./Intersistemas S.A. de C.V.; 2008. Firestein GS, Budd RC, Harris ED Jr, McInnes IB, Ruddy S, Sergent JS, editors. Kelley’s Textbook of Rheumatology. 8th ed. Philadelphia: Saunders Elsevier; 2009. 57.- Se trata de un niño de 7 años, con una talla más corta que la que debería corresponder a su edad, aqueja dolores moderados de su cadera derecha y claudicación con la carrera. A la exploración se aprecia una discreta atrofia del muslo y existe una limitación de los movimientos de rotación interna y abducción. Los estudios de laboratorio son negativos. La radiografía convencional sólo muestra una discreta osteoporosis del núcleo epifisario de la cabeza femoral. Con este cuadro clínico el diagnóstico debe orientarse hacia: a) Epifisiolisis femoral superior. b) Quiste óseo esencial del extremo superior del fémur. c) Artritis inespecífica de la cadera. d) Enfermedad de Perthes. Concepto: Es una necrosis avascular de la cabeza femoral durante la edad infantil, de grado variable, que se sigue de un proceso regenerativo, que puede o no, acabar en deformidad irreversible. Historia: La primera descripción fue de Waldeström (1909), que la relacionó con tuberculosis. Arthur Legg (EEUU), Jaques Calvé (Francia) y Georg Perthes (Alemania), en 1910, lo califican de enfermedad no infecciosa, distinta de la TBC, autolimitada, que produce secuelas (coxa plana). Debido a lo similar de la descripción, se la conoce como "enfermedad de Legg-Calvé-Perthes". Phemister, en 1921, describió los cambios patológicos aceptados hoy día. Epidemiología:

Sexo: más frecuente en niños que en niñas.(4/1). Monolateral más frecuente que bilateral (85-90%) Incidencia: cinco niños por cada 100000 menores de 15 años. Raza: más frecuente en japoneses, esquimales, mongoles y caucásicos. Poco

frecuente en negros y chinos. Más frecuentes en zonas urbanas que en zonas rurales. Edad: entre 2-10 años, más frecuente entre 4-8, pico en los 5,5 años. Se asocia a un retraso en la edad esquelética. Etología desconocida.

Page 67: Examen final 20460

Teorías:

1. Traumática: microtraumatismos de repetición provocarían lesiones vasculares ( parte anterior de epífisis y reborde cotiloideo ). Hay un traumatismo único e intenso que provocaría una sinovitis en el 30 al 50% de los niños con enfermedad de Perthes. La cabeza femoral de los niños sería más suceptible a los traumatismos que la de las niñas.

2. Infecciosa: provocaría una sinovitis por mecanismo directo que originaría una isquemia por aumento de la presión intracapsular.

3. Inflamatoria o de tapònamiento: una sinovitis provocaría un aumento de la presión intracapsular que comprimiría los vasos retinaculares. Vascularización de la cabeza femoral: Menor de 4 meses: a. vasos metafisarios. b. vasos epifisarios, van por los retináculos.

4. c. vasos del ligamento redondo.

Los vasos metafisarios disminuyen gradualmente hasta los cuatro años, los vasos del ligamento redondo aumentan a partie de los siete años, entre los cuartro y los siete años la irrigaciñon depende de loa vasos retinaculares que pueden quedar comprimidos si aumenta la presión intracapsular.

2. Factores estáticos: asociación con coxa valga.

1. Vascular: bloqueo total o parcial del aporte sanguíneo, por embolia , trombosis, etc. Actualmente se ha descrito un defecto en la coagulación que se encuentra en el 75% de los enfermos de Perthes

Hallazgos clínicos. Cojera espontánea ( lo más frecuente ) con o sin dolor. Dolor en ingle o rodilla. Limitación de la movilidad de la cadera, sobre todo la rotación interna y la abducción. Asintomático. Sinovitis. Bibliografía. Campbell: tratado de cirugía ortopédica pag: 1852-1861. Conceptos básicos de ortopedia infantil. J. Minguella. Pag: 52-55. Tratado de patología y clínica quirúrgica. H. Duran Sacristan. Pag: 3695-3708. Enfermedad de Legg-Calves-Perthes. Por Dr. D. Julio de Pablo. Monográfico de enfermedad de Perthes. Clinica Mapre. Año 1995. 58.- Acude a su consulta paciente femenino de 26 años que cursa el tercer trimestre del embarazo tiene edema en miembros inferiores sin sintomatología agregada, la medida más adecuada que se debe indicar a la paciente es:

a) Elevación de los miembros inferiores en decúbito lateral

b) Restricción Hídrica

c) Tiacidas

d) Diurético de asa

Page 68: Examen final 20460

DeCherney A. (1999) Diagnóstico y tratamiento ginecoobstétricos. México. Ed. Manual Moderno. Pag 237. El edema de las partes bajas producido por la impedancia del retorno venoso es común en la parte final del embarazo, La paciente debe tratarse solo si está molesta. La elevación de los miembros inferiores (especialmente en decúbito lateral) mejora la circulación. Están contraindicados los diuréticos. 59.- Mujer de 23 años G/1 la cual cursa con 5 semanas de gestación tomó cumarina hasta el día en que se enteró que estaba embarazada. Está preocupada de que la cumarina ocasione defectos congénitos en el producto. En el caso anterior el defecto congénito más frecuente tras el uso de cumarina es:

a) Hipoplasia nasal b) Espina bífida c) Anencefalia d) Acondroplasia

Tabla I Fármacos cuya teratogénia se ha comprobado en humanos

FÁRMACO MALFORMACIONES DETECTADAS RIESGO

ANTIBIOTICOS

Tetraciclinas Tinción dental, hipoplasia enamel. Posible retraso crecimiento óseo.

Alrededor de 50% de los expuestos a tetraciclina; 12,5% de expuestos a oxitetraciclina.

ANTICOAGULANTES

Cumarinas Síndrome fetal de la warfarina: hipoplasia nasal, condrodisplasia punctata, braquidactilia, defectos craneales, orejas anomalas, ojos malformados, malf. en sistema nervioso central, microcefalia, hidrocefalia, deformidades esqueléticas, retraso mental, atrofia óptica, espasticidad; malformación de Dandy Walker.

El 16% tiene malformaciones, un 3% hemorragias. Se producen hasta un 8% de abortos.

ANTIEPILEPTICOS

Carbamacepina Aumento del riesgo de sufrir defectos del tubo neural.

Se estima un riesgo del 1%.

Fenitoina Síndrome fetal de la fenitoina: aplanammiento puente nasal, pliegues epicantales internos, ptosis, estrabismo, hipertelorismo, orejas anormales o de baja implantación, hipoplasia de falanges distales y uñas, anomalías esqueléticas, microcefalia y retraso mental, deficiencias del crecimiento, neuroblastoma, defectos cardíacos, paladar hendido y labio

El 5-10% puede sufrir el síndrome típico. Un 30% parcialmente. Hasta un 7% de descendientes con coeficiente intelectual bajo.

Page 69: Examen final 20460

leporino.

Trimetadiona (Troxidona)

Síndrome fetal de la trimetadiona: retraso crecimiento intrauterino, anomalías cardíacas, microcefalia, labio y paladar hendidos, anomalías en orejas, facies dismorfica, retraso mental, fístula traqueo-esofágica, muerte postnatal.

Basándose en casos notificado: 83% y 32% de muerte infantil o neonatal.

Acido valproico Espina bífida con mielomeningocele, defectos en sistema nervioso central, microcefalia, defectos cardiacos.

Un 1% de riesgo de defectos del tubo neural.

ANTINEOPLASICOS

Alquilantes

Busulfan Clorambucil Ciclofosfamida Clormetina

Retraso del crecimiento, paladar hendido, microftalmia, hipoplasia ovárica, opacidad corneal, agenesia renal, malformaciones de los dedos, defectos cardíacos, otras anomalías múltiples.

Entre el 10-50% de los casos pueden presentar malformaciones, aunque este riesgo, extraído de series de casos puede estar exagerado.

Antimetabolitos

Aminopterina Azauridina Citarabina Fluorouracilo Mercaptopurina Metotrexato

Hidrocefalia, meningoencefalocele, anencefalia, malf. craneales, hipoplasia cerebral, retraso del crecimiento, malformaciones oculares y del oído, malformaciones nasales, paladar hendido, malf. en miembros y dedos.

Síndrome fetal de la aminopterina: disostosis craneal, hidrocefalia, hipertelorismo, anomalías de oído externo, micrognatia, paladar hendido.

Entre el 7-75% de los casos expuestos, aunque este riesgo, extraído de series de casos puede estar exagerado.

HORMONAS

Dietilestilbestrol Descendencia femenina:Adenocarcinoma de células claras vaginal o cervical en mujeres jóvenes expuestas "in utero" (antes de la semana 18); oligomenorrea, reducción de las tasas de embarazo, incremento de las tasas de embarazos pretermino, incremento de la mortalidad perinatal y de aborto espontáneo.

Descendencia masculina:Quistes epididimarios, criptorquidia, hipogonadismo, disminución de la espermatogénesis, estenosis de meato, hipospadias.

Cambios morfológicos congénitos en epitelio vaginal en el 39% de las expuestas. En exposición antes de la 18ª semana: riesgo de carcinoma ³ 1.4 por 1000 de expuestas.

NEUROLEPTICOS

Litio Mayor riesgo de anomalía de Ebstein; no se ha detectado un mayor riesgo de otro tipo de malformaciones.

PENICILAMINA

Page 70: Examen final 20460

Hiperclastosis cutánea (cutis laxa) Pocos casos. Riesgo desconocido.

RETINOIDES SISTEMICOS

Isotretinoina Etetrinato

Aborto espontáneo, deformidades craneales, de orejas, cara, corazón, extremidades e hígado; hidrocefalia, microcefalia, defectos cognitivos (incluso sin malformaciones aparentes).

Isotretinoina: 38%. El 80% son malformaciones del sistema nervioso central.

TALIDOMIDA

Focomelia, amelia, hipoplasia de miembros, defectos cardíacos congénitos, malformaciones renales, criptorquidia, parálisis VI par, sordera, microtia, anotia.

Alrededor del 20% cuando la exposición ocurre entre 4-8ª semana.

Bibliografía.

McBride WG. Thalidomide and congenital abnormalities. Lancet 1961; 2:1358. Lenz W. Thalidomie and congenital abnormalities. Lancet 1962;1:45. Shepard TH. Catalog of Teratogenic Agents. Johns Hopkins University Press London,

1987. Piper JM. Banur C, Kennedy DI. Prescription drugs use in pregnancy in a Medicaid

population. Am J Obstet Gynecol 1987; 157:148-156. Salvador J, Martínez-Frías ML, Rodríguez Pinilla E. Consumo de medicamentos por la

mujer embarazada en España. Ministerio de Sanidad y Consumo. Madrid, 1989. Grupo de trabajo DUP. Estudio multicéntrico sobre el uso de medicamentos durante el

embarazo en España (I). Métodos y características demográficas de la población estudiada. Med Clin 1990;95:764-767.

Grupo de trabajo DUP. Estudio multicéntrico sobre el uso de medicamentos durante el embarazo en España (II). Los fármacos utilizados durante la gestación. Med Clin 1991;96:11-15.

Grupo de trabajo DUP. Estudio multicéntrico sobre el uso de medicamentos durante el embarazo en España (III). Los fármacos utilizados durante el primer trimestre de la gestación. Med. Clin 1991;96:52-57.

60.- Primigesta de 23 años, en su séptima semana de gestación, que acude por metroragia escasa y dolor abdominal moderado. En la exploración se aprecia sangrado a través del OCE, útero aumnetado de tamaño, engrosamiento anexial izquierdo y discreta irritación peritoneal. El primer paso para confirmar su diagnóstico es:

a) Determinación seriada de B-HCG b) Examen ecográfico c) Reposo y reevaluación en una semana d) Metrotexate intramuscular.

Page 71: Examen final 20460

Diagnóstico ectópico. Dolor abdominal Sangrado o hemorragia Tumor anexial Retraso menstrual

61.- Acude a consulta recién nacido con sospecha de luxación de la articulación de la cadera, la siguiente maniobra que nos corrobora dicha patología es:

a) Signo de Galeazzi b) Signo de Barlow c) Signo de Ortolani d) Signo de Pistón

Prueba de Ortolani : con esta prueba se detecta una cadera ya luxada, se coge con la mano el miembro flexionado, la cadera se coloca en abducción mientra se levanta el fémur con cuidado y se sitúa los dedos a nivel del trocánter mayor. Si la prueba es positiva se siente la reducción de la cadera dentro del acetábulo. Prueba de Barlow: es una prueba inductora para identificar una cadera inestable pero aún localizada en su sitio; no es una prueba adecuada para diagnosticar luxación de cadera. La cadera en aducción ligera y con la palma de la mano se empuja suave y cuidadosamente hacia atrás, la presencia de un movimiento de “pistón” o al percepción de una cabeza femoral subluxada sobre el borde posterior del acetábulo. Prueba de Galeazzi: con el niño acostado se le flexionan las caderas y rodillas de modo que los talones se apoyen sobre la mesa y reconocer el acortamiento relativo del muslo.

Page 72: Examen final 20460

Skinner, H. Diagnóstico y tratamiento en Ortopedia. Ed. Manual Moderno. México, 2004. pp. 625 62.- Femenino de 19 meses, es llevada por su madre al servicio de urgencias de Pediatría por presentar exantema generalizado. Antecedentes: Fiebre de 38.5 , Irritabilidad, anorexia, y haber recibido tratamiento a base de acetaminofen 48 hrs previas a la consulta. Exploración física: Temp 36.8, TA 100/65mmHg, FC 97 x´, FR 26 x´, exantema en tronco, cuello y cara. En este paciente el diagnóstico más probable es: a) Eritema infeccioso. b) Roséola. c) Rubéola. d) Varicela. Roséola: Es una enfermedad febril exantemática leve, que ocurre casi en exclusiva durante la lactancia. El período prodrómico de la roséola suele ser asintomático, pero puede presentarse con signos leves del tracto respiratorio superior, entre ellos rinorrea mínima, inflamación faríngea ligera y enrojeciiento conjuntival leve, adenoparias cervicales. La enfermedad clínica suele ser precedida por temperatura alta, la fiebre persiste durnte 3 a 5 días, y después es típica la resolución brusca. El exantema aparece en 12-24 hrs siguientes a la resolución de la fiebre, comienza en forma de lesiones rosadas, un poco elevadas, pequeñas, y delimitadas en tronco, y se suele extender al cuello, cara, y regiones proximales de las extremidades, no suele ser pruriginoso. Nelson, Tratado de Pediatría, 17ª Edición, Ed. Elsevier, Pág. 1069-1071

Page 73: Examen final 20460

63.- Neonato prematuro quien tiene un parto difícil, con episodios de arritmia y sospecha de hipoxia-isquemia. Después del nacimiento, se encuentra letárgico y tiene periodos de apnea. Se sospecha hemorragia intracraneana. No se observa un traumatismo craneal obvio. El USG craneal identifica sangre en los ventrículos. De las siguientes estructuras la que da el origen más probable de la hemorragia es: a) Corteza cerebral b) Matriz germinal c) Tálamo d) Vasos del círculo de Willis

Debe diferenciarse entre la hemorragia del neonato pretérmino y la del de término. • La del recién nacido pretérmino es la más frecuente y se produce por sangrado de la matriz germinal subependimaria.

• La del neonato de término es menos frecuente y se produce por traumatismos, malformaciones vasculares, infecciones, tumores u otras causas.

Matriz germinal

• Es un tejido subependimario adyacente a los ventrículos laterales, altamente vascularizado, cuyos vasos muy delicados sangran fácilmente ante distintos tipos de estrés perinatal. • Es una estructura transitoria del cerebro fetal que involuciona normalmente hasta desaparecer hacia las 32-34 semanas de gesta. Previo a su desaparición sólo persiste en el surco caudotalámico. • Se denomina "matriz germinal" porque es el tejido que da origen a las neuronas, que posteriormente llegan a la corteza cerebral por el proceso llamado migración.

Clasificación de H. de la matriz germinal

• Grado I: hemorragia localizada en la matriz germinal. Puede ser unilateral o bilateral. • Grado II: hemorragia que se extiende al ventrículo lateral sin producir dilatación. • Grado III: hemorragia intraventricular acompañada de dilatación ventricular aguda y concomitante. Conviene aclarar que la hidrocefalia secundaria que se produce luego de varios días no se considera como grado III, pues se debe a dificultad en la reabsorción de LCR en las granulaciones de Paccioni, por la obstrucción producida por el sangrado. • Grado IV: compromete al parénquima cerebral vecino. • Esta clasificación en grados es importante y tiene valor pronóstico. A mayor grado, más frecuentes e importantes serán las secuelas neurológicas.

Cuadro Clínico La HIV tiene generalmente un inicio súbito, a las 24 - 48 horas del nacimiento, pudiendo aparecer en un prematuro con una clínica post-parto normal y lógicamente debe esperarse su aparición en los que tienen cuadros respiratorios asociados, en los nacidos de madres muy jóvenes, en los de peso muy bajo y cuando el período gestacional es más corto, recordar que el 90% de los nacidos con una edad gestacional de menos 32 semanas hacen HIV(37,39). El cuadro habitual es de aparición de convulsiones, paro cardíaco o apnea prolongada, con toma del sensorio, fontanela abombada y caída del hematocrito; pero hay pacientes que pueden tener poca o ninguna sintomatología(48,54) y por esto es importante descartar una HIV ante la presencia de una caída del hematocrito sin explicación aparente. Se conoce que una fontanela normotensa no descarta un cuadro de HIV. Nosotros tenemos pacientes RN que llegaron a nuestro servicio para tratamiento de una hidrocefalia aparentemente congénita(sin antecedentes de ningún tipo que sugiriera que fuera secundaria) y detectamos que

Page 74: Examen final 20460

eran portadores de un líquido ventricular hemorrágico; por eso es nuestra práctica en la actualidad realizar una punción ventricular para estudio del LCR(tanto del aspecto como citoquímico) a toda hidrocefalia que debuta en los primeros treinta días de la vida.

Diagnóstico Está bien establecido la vigilancia estrecha de los prematuros en general y en especial los que constituyen el grupo de "alto riesgo" (véase fisiopatología y patogenia). A todo prematuro se le debe realizar estudios con ultrasonidos diarios durante la primera semana de nacido y posteriormente semanales hasta el alta. Mucho se ha discutido acerca de la investigación ideal a realizar en estos pacientes. Algunos recomendaron la Tomografía Axial Computada(TAC) por los detalles que la misma puede aportar; sin embargo tiene el inconveniente de que al niño hay que trasladarlo hacia el Departamento de Imagenología y al mismo tiempo va a recibir una cantidad enorme de radiaciones, en un paciente inmaduro, lo que puede ser perjudicial. La Ultrasonografía(US) es indudablemente el estudio ideal, pues permite realizar el diagnóstico al lado de la cama y repetirlo las veces que se crea necesario sin efectos deletéreos sobre el paciente; debe tenerse en mente la necesidad de visualizar la fosa posterior. Para nosotros el mayor valor del US está dado por la evolutividad del mismo y claro está en los casos que ofrezca dudas se puede complementar con la TAC(11,12,37,39,40,41,48). Los estudios Ultrasonográficos, para la mayoría de los autores, permiten clasificar las HIV en cuatro grupos fundamentales(6,11,12,62): Grado I: Cuando el sangramiento está localizado en la matriz germinal, sin o mínima hemorragia intraventricular. Grado II: El sangramiento es intraventricular y ocupa entre el 10 y el 50 % del ventrículo. Grado III: El sangramiento intraventricular es mayor del 50 % y el ventrículo lateral está distendido. Grado IV: Cuando a lo anterior se asocia sangramiento intraparenquimatoso. Volpe(63) sólo reconoce los tres primeros grupos. Una vez establecido el diagnóstico de HIV se realiza seguimiento evolutivo con US, buscando la aparición signos de Hidrocefalia. James y colaboradores(30) sugieren los siguientes criterios para su diagnóstico, atrium ventricular mayor de 10 mm, Indice de Evans mayor de 0,35 y cuando el cuerpo del ventrículo lateral, en un plano sagital, a nivel del tálamo mide más de 10 mm.

64.- Masculino de 5 años de edad quien presenta padecimiento actual de un mes de evolución refiriendo márgenes de atención breves, descenso del rendimiento académico. Canalizado con facultativo quien lo hace correr ante lo cual el paciente presenta mirada fija y parpadeo durante 12 seg. El diagnóstico más probable es: a) Trastorno por déficit de atención b) Hipoacusia c) Desintegración familiar d) Epilepsia generalizada Las epilepsias son los trastornos paroxísticos neurológicos más frecuentes durante la infancia. Se manifiestan aproximadamente en uno de cada 100 niños, con una amplia variedad de síndromes epilépticos propios de cada edad pediátrica, que tienen una sintomatología clínica característica, un patrón electroencefalográfico específico, un tratamiento farmacológico bastante bien definido y un pronóstico muy diferente, pudiendo diferenciarse unos síndromes epilépticos benignos y otros de mal pronóstico. Por epilepsia se entiende el padecimiento de dos o más crisis paroxísticas de naturaleza indudablemente epiléptica; es decir, producidas por una actividad anormalmente elevada a nivel neuronal.

Page 75: Examen final 20460

Es un diagnóstico clínico, que sitúa al electroencefalograma (EEG) como elemento importante para ratificar y concretar el síndrome epiléptico, pero que por sí solo nunca debe condicionar el diagnóstico.

Convulsión : Un tipo de crisis caracterizada por movimientos intensos y bruscos de extensión y flexión en las extremidades asociado a perdida de la conciencia de diversas génesis

Epilepsia : Afección crónica, recurrente de crisis paroxisticas (epilepticas), desencadenadas por descargas de neuronas, asociadas a manifestaciones clinicas y paraclinicas Clasificación

Generalizadas * Tónico clónicas (gran mal) * Ausencias (pequeño mal) * Mioclonus bilateral * De la infancia - Espasmo infantil - Crisis atónicas - Crisis tónicas Clasificación

Parciales * Simples - Motoras (Jaksonianas) - Sensoriales (auras) - Con síntomas autonómicos - Afectivas

Parciales * Complejas - Lóbulo temporal - Con sintomas afectivos - Síntomas psicosensoriales - Síntomas de automatismo sicomotor

Parciales secundariamente generalizadas * Parcial simple sec. Generalizada * Parcial compleja sec. Generalizada * Parcial simple sec. Parcial compleja sec. Generalizada ESTUDIOS

ELECTROENCEFALOGRAMA MAPEO CEREBRAL VIDEOELECTROENCEFALOGRAMA EEG TELEMETRIA EEG CON ELECTRODOS DE PROFUNDIDAD

UTILIDAD DEL EEG

Page 76: Examen final 20460

La neurología y la psiquiatría trabajan con el mismo órgano blanco, el cerebro, y las mismas neuronas que pueden mal funcionar y dar un tipo específico de epilepsia, pueden provocar un trastorno psiquiátrico. El cerebro del niño es más susceptible a los cambios o las variaciones en su función eléctrica debido a factores intrínsecos de la edad como son la mielinización y los factores activos de plasticidad cerebral. El EEG es una herramienta muy útil en la epilepsia. Se debe realizar lo más pronto posible después de haberse presentado la crisis.3 Es posible que el EEG se encuentre normal en un paciente que tiene crisis epilépticas. Esto puede deberse a que la descarga se encuentre en áreas a las cuales el EEG convencional tiene poco acceso, o bien que no haya sido posible registrar la anormalidad en el espacio de tiempo en el que se practicó. Existen múltiples artículos que intentan establecer patrones eléctricos para patologías psiquiátricas específicas, pero todos ellos concluyen que no son patognomónicos del trastorno.9,10 En diversas series se han reportado incluso anormalidades eléctricas en niños clínicamente sanos. En un estudio que publicamos en la Revista de Neurología,11 analizamos 1000 estudios de EEG en niños con trastornos psiquiátricos infantiles, eliminando a todos aquellos que presentaban epilepsia. Los estudios normales fueron 378 (37.8%) y los anormales 619 (61.9%), de los cuales se observaron datos inespecíficos de inmadurez en 548 (54.8%), incremento en la excitabilidad tálamo–cortical en 45 pacientes (4.5%) y actividad epileptiforme en 26 niños (2.6%). El artículo concluyó: a) No existen hallazgos constantes en todos los pacientes con similares cuadros clínicos, b) no son específicos de un trastorno psiquiátrico, c) no pueden ser utilizados como indicadores del pronóstico, d) mucho menos como pauta para decidir un tratamiento farmacológico especifico, pues este tipo de decisión debe realizarse conforme a los datos clínicos de cada paciente. Las anormalidades en el EEG de pacientes con trastornos psiquiátricos infantiles son frecuentes, indican disfunción cerebral, pero sabemos que existe mucha controversia en lo que pueden representar clínicamente. 65.- Recién nacido cuarto día de vida extrauterina es referido al hospital por médico de centro de salud. La madre comenta que el menor no ha presentado evacuaciones desde su nacimiento y presenta vómito verdoso y distensión abdominal desde hace dos días. Exploración física. En malas condiciones generales, ojos hundidos mucosas orales secas, con sonda orogástrica con drenaje de líquido verdoso, cardiopulmonar, fc 170 lpm, fr 120 rpm, abdomen: piel brillante, resistencia por distensión abdominal, timpánico franco, no visceromegalias, extremidades con llenado capilar de 12 segundos. El estudio de gabinete más sensible para establecer el diagnóstico es:

a) Radiografia de abdomen. b) Invertograma. c) Ultrasonografia abdominal. d) Tomografia abdominal.

Respuesta: rx abdomen. Nos encontramos ante un caso de atresia duodenal. Método diagnóstico, signo de la doble burbuja en abdomen. La atresia duodenal es una embriopatía del intestino craneal que conduce a la ausencia total del lumen duodenal. La incidencia de la atresia duodenal se estima entre 1/10.000 y 1/6.000 nacimientos vivos, afectando casi por igual a hombres y mujeres. En el 30-52% de los casos se trata de una anomalía aislada, pero es habitual que esté asociada a otras anomalías congénitas. Aproximadamente el 20-30% de los niños con atresia duodenal son portadores de la trisomía 21, y alrededor del 20-25% presentan anomalías cardíacas. Otras malformaciones asociadas que se han descrito habitualmente son: deficiente crecimiento del duodeno, páncreas anular (ver este término), que son formas clínicas particulares de la atresia duodenal, y anomalías del tracto biliopancreático o quistes de colédoco. La atresia duodenal se clasifica en tres tipos: tipo I (diafragma duodenal), relacionado con la presencia de una membrana diafragmática mucosa con pared muscular intacta; tipo II (atresia duodenal completa), caracterizado por una cuerda fibrosa corta que conecta los dos finales del duodeno atrésico; y tipo III (también atresia duodenal

Page 77: Examen final 20460

completa), que corresponde a la separación completa de los dos finales del duodeno, en ocasiones, junto con páncreas anular. La presentación clínica depende del grado de atresia. Las obstrucciones importantes se manifiestan durante los primeros días de vida, por vómitos biliosos, si el obstáculo es infra-vateriano, que comienzan horas después del nacimiento, y por intolerancia alimenticia. Los síntomas principales son: pérdida de peso, deshidratación y alcalosis metabólica hipoclorémica. Las obstrucciones menos graves pueden manifestarse durante varios meses, o incluso varios años, después del nacimiento, con vómitos biliosos sin distensión abdominal (signo principal); sin embargo, también es posible que el único signo sea el retraso en el crecimiento. En la mayoría de los casos, la atresia duodenal es esporádica, pero en algunas familias se ha sugerido un patrón autosómico recesivo. La causa exacta permanece desconocida, pero las anomalías vasculares, las anomalías en la migración de las células nerviosas y un fallo en la recanalización del lumen duodenal podrían estar implicados en su origen. El diagnóstico clínico se confirma por una radiografía abdominal que muestra una imagen característica en `doble burbuja', con aire retenido en la primera porción del duodeno y del estómago, debido a la distensión simultánea del estómago y la primera porción del duodeno (por encima de la estenosis). El diagnóstico es prenatal en el 80-90% de los casos (ecografía, que revela la imagen característica en `doble burbuja', en el séptimo mes de embarazo o antes). El diagnóstico diferencial incluye: estenosis pilórica de aparición tardía en casos de diafragma incompleto, y otras formas de atresia intestinal, vólvulo intestinal en el mesenterio común, y duplicación duodenal. El manejo implica la reanimación neonatal y la corrección quirúrgica en periodo neonatal. Las complicaciones post-operatorias son raras, pero las complicaciones tardías (megaduodeno, síndrome del asa ciega, reflujo duodenogástrico, esofagitis, pancreatitis, colecistitis y colelitiasis) ocurren en casos muy raros. 66.- Paciente masculino de 42 años, con antecedente de alcoholismo, diabético, hospitalizado por TCE. Durante su estancia intrahospitalaria inicia con fiebre en picos (38.5º-39º C), escalofríos, aparición de soplo y la biometría hemática muestra leucocitosis (16,300) se descarta IVU y neumonía. Se sospecha endocarditis infecciosa. La conducta inicial más apropiada es: a) Tomar hemocultivos seriados b) Ecocardiograma transtorácico c) Hemocultivo a través de catéter d) Retiro de catéter intravascular

La endocarditis infecciosa es una infección microbiana que se localiza sobre las válvulas cardíacas o sobre el endocardio mural. A pesar que la mayoría de estas infecciones son causadas por bacterias, se considera más apropiado denominarla en forma global como endocarditis infecciosa debido a que también puede ser producida por hongos, rickettsias o clamidias.

La endocarditis bacteriana es la forma más reconocida y se clasifica en aguda o subaguda según su presentación clínica. La endocarditis micótica se detecta casi exclusivamente en drogadictos o en pacientes con prótesis valvulares.

Hemocultivos con antibiograma Se recomiendan tomar tres muestras a la llegada del paciente (preferiblemente sin tomar antibióticos), las muestras de sangre separado por un intervalo mínimo de 1 h, en un período de 24 horas; al día siguiente debe repetirse el mismo proceder. Debe realizarse una correcta antisepsia previa de la piel y cambiar de aguja cuando se vaya a verter la sangre en los frascos. El cultivo de sangre se realiza en un tubo especializado para hemocultivos automatizados. � Se prefiere dejar al paciente, si el estado clínico así lo tolerara, hasta un mínimo de 72 horas sin antibióticos, antes de tomar la muestra, pero si el paciente por su situación de sepsis avanzada y repercusión hemodinámica no se le puede retirar el antibiótico entonces entonces se utiliza el frasco de hemocultivo automatizado con concentración mínima inhibitoria del

Page 78: Examen final 20460

antibiótico, precisando en la orden el ó los tipos de antibióticos que se están empleando en la terapéutica antimicrobiana. � Si al realizar los hemocultivos automatizados estos todos vienen negativos a pesar de que la sintomatología del paciente no mejora se pueden indicar los hemocultivos automatizados con medios de cultivos especiales para gérmenes de crecimiento lento, por ejemplo en medio de Sabureao. � A estos frascos se le añade sangre venosa, entre 5-10 mL en adultos. Se llevará la muestra lo más rápido posible al laboratorio de Microbiología. La toma de muestra se realizará independientemente de la temperatura del paciente o cuando se presentan los escalofríos y no en pico febril. � Para confirmar criterio de curación de la infección previo al egreso hospitalario se realizan 6 hemocultivos automatizados una semana después de suspendidos los antibióticos. La endocarditis infecciosa (EI) es una enfermedad en la que uno ó varios mi-croorganismos infectan el endocardio, las válvulas ó las estructuras relaciona-das, generalmente sobre una lesión ante-rior: cardiopatía reumática en otros tiempos, cardiopatías congénitas en la actualidad. Puede ser súbita y aguda, pero más frecuentemente es subaguda y larvada lo que retrasa el diagnóstico. Tiene morbilidad y mortalidad impor-tante a pesar de los tratamientos antimi-crobianos y de la difusión de su profi-laxis entre los niños susceptibles. Nue-vos grupos de riesgo son los pacientes sometidos a cirugía cardiaca, los niños que precisan catéteres intravasculares, especialmente neonatos, los inmunode-primidos y los adictos a drogas por vía intravenosa. La ecocardiografía- Dop-pler aporta actualmente criterios básicos para el diagnóstico y seguimiento. Pue-den ser útiles a niños nuevos recursos en el tratamiento y una profilaxis preci-sa. 67.- Si el estudio de triple marcador en suero materno indica una elevación de AFP (alfa feto-proteína) respecto a la edad gestacional, usted debe sospechar como primera opción:

a) Trisomía 21 (Down) b) Trisomía 18 (Edwards) d) Defecto de cierre de tubo neural o de pared abdominal e) Labio hendido

Alfa feto proteína (AFP)

VALORES ALTOS

• Defectos abiertos del tubo neural

• Onfalocele• Gastrosquisis• Bandas amnióticas • Desprendimiento de 

placenta• Hemorragias fetomaternas• Procedimientos invasivos

VALORES BAJOS

• Alteraciones cromosómicas fetales 

• Síndrome de Williams.*

*Chodirker y cols.

Alfa-fetoproteína sérica materna

Page 79: Examen final 20460

El feto en desarrollo tiene dos proteínas mayores: albúmina y alfa-fetoproteína. Ya que los adultos tienen solamente albúmina en la sangre, la prueba de detección de la alfa-fetoproteína puede ser utilizada para determinar los niveles de esa proteína provenientes del feto. Ordinariamente sólo una pequeña cantidad de AFP llega al líquido amniótico y cruza la placenta hacia la sangre de la madre. Sin embargo, cuando existe una alteración del tubo neural en el feto, producido por un defecto en una parte del tubo embriológico neural que no se cierra, entonces se produce una vía de escape de más AFP hacia el líquido amniótico. Para que la prueba de la AFP tenga su mayor utilidad, se debe conocer con certeza la edad gestacional, lo que es debido a que la cantidad de AFP se incrementa con la edad gestacional ( ya que conforme se incrementa el tamaño del feto la cantidad de AFP también lo hace ). También es importante identificar la raza de la madre y la presencia de diabetes gestacional , debido a que la AFP puede ser modificada por estos factores. Entre mayor sea, es más probable que el defecto se encuentre presente. La AFP tiene la mayor sensibilidad entre las 16 y las 18 semanas de gestación, pero puede continuar siendo útil entre las 15 y 22 semanas. Sin embargo, la AFP puede estar elevada por diversas razones que no se encuentran relacionadas con defectos del tubo neural o de la pared abdominal, de tal manera que no es 100% específica. La causa más común de una AFP elevada es una estimación equivocada de la edad gestacional del feto. Utilizando la prueba de tamizaje de la AFP y la ultrasonografía, casi todos los casos de anencefalia y espína bífida pueden ser detectados. Los defectos del tubo neural pueden ser distinguidos de otros defectos fetales (tales como defectos de la pared abdominal) por medio del uso de la prueba de la acetilcolinesterasa realizada en el líquido amniótico obtenido por amniocentésis. Si la acetilcolinesterasa se encuentra elevada junto con la AFP, se hace más probable que exista una defecto del tubo neural. Si no es detectada, se sospecha otro defecto fetal. La alfafetoproteína puede ser útil también para el tamizaje del síndrome de Down y otras trisomías. La AFP tiende a ser baja cuando se encuentra el Síndrome de Down u otras anormalidades cromosómicas.

Cortés H, Velez JF. Diagnostico ecográfico prenatal de la Pentalogia de Cantrell. Rev

Coloma. Obstetr Ginecol mar 2003;vol 54 n.6 : 34-41.

Bankowsky BJ, Hearne AE. Johns Hopkins Ginecología y Obstetricia. 2da ed. Madrid; Marban libros,2005.

Medina C, Rivas F, Fernández R. Riesgo de malformaciones congénitas en mujeres gestantes expuestas a plaguicidas en el estado de Nayarit Mexico. Rev. Mex Ginecol. Obstet. Nov. 2002; vol 70 n. 11: 30-41.

68.- Masculino de 42 años de edad con diagnóstico de herpes genital activo y recurrente, presenta de manera súbita numerosas máculas eritematosas, pápulas, vesículas. Las manos, pies y cara están extensamente afectadas. A la exploración física minuciosa demuestra que muchas de las lesiones tienen una aparencia de “tiro al blanco”. Algunas afectan labios y mucosa oral. El diagnóstico más probable es: a) Granuloma anular b) Pénfigo c) Necrolisis tóxico epidérmica d) Eritema multiforme

Page 80: Examen final 20460

El eritema multiforme es una enfermedad de la piel y las mucosas que se manifiesta con lesiones eritematosas y de tipo vesículo-ampollar. Las lesiones Vesículo-ampollares y erosivas a nivel de la cavidad oral y la piel pueden ser causadas por un amplio grupo de patologías. La etiología de las mismas también, puede ser muy variable, desde una causa traumática o química por contacto, hasta una causa autoinmune. Las lesiones producidas por el eritema multiforme son circulares y pueden presentarse como anillos concéntricos (se las conoce como lesiones de tiro al blanco). Estas lesiones también pueden asociarse con otras condiciones médicas tales como infección por herpes, infección estreptocócica, tuberculosis (TB), o como una reacción a sustancias químicas o medicamentos.

Gavaldá-Esteve C, Murillo-Cortés J, Poveda-Roda R. Eritema multiforme. Revisión y puesta el día RCOE, 2004, Vol 9, Nº4, 415-423 69.- Masculino de 33 años de edad, acude a consulta por presentar diferentes lesiones en el tronco. Refiere que inició con una lesión única en el tórax, la cual creció, y después otras lesiones aparecieron. A la EF presenta múltiples lesiones eritemoescamosas en el tórax y espalda. La más grandes (que el paciente dice que fue la primera) mide 5cm de diámetro, es oval y tiene un coloración rosada. Presenta un borde ligeramente elevado y se observa alrededor de los límites de la lesión. Muchas placas pequeñas de aproximadamente 1cm de diámetro y de apariencia similar, también son vistas. El diagnóstico más probable es: a) Molusco contagioso b) Pitiriasis rosada de Gibert c) Pityriasis rubra d) Escabiosis DEFINICIÓN La pitiriasis rosada de Gibert es una dermatosis pápulo-escamosa autolimitada que se presenta generalmente en adolescentes o adultos jóvenes CLÍNICA La mayoría de los casos ocurren en personas jóvenes y sanas, con una máxima incidencia entre los 10 y los 35 años. Es muy rara en recién nacido y anciano y parece ser que es ligeramente más frecuente en varones. En su forma típica, la erupción comienza por una lesión única con morfología de placa anular de borde descamativo que asienta preferentemente en el

Page 81: Examen final 20460

tronco y puede alcanzar varios cms de diámetro. Esta lesión inicial, denominada placa heraldo o placa madre, va seguida en el plazo de unos pocos días de la erupción de múltiples máculo-pápulas, de coloración rosada y tendencia a adoptar una morfología circular u ovalada que se distribuyen de manera simétrica por el tronco y raíces de los miembros (Figura 1). Habitualmente, la cara, las mucosas y las zonas distales de las extremidades, incluyendo palmas y plantas están respetadas. Estas lesiones múltiples que aparecen después de la placa heraldo 2) salpicadas por el tronco suelen ser de menor tamaño que la lesión inicial, pero también tienen un fino collarete descamativo en su periferia. En general, las lesiones tienen tendencia a distribuirse siguiendo las líneas de tensión de la piel y la morfología general puede adoptar una distribución en la espalda que se ha comparado con un árbol de Navidad. La mayoría de los pacientes no presentan ninguna sintomatología sistémica y las lesiones son escasamente pruriginosas, aunque algunos pacientes pueden mostrar malestar general, cefaleas, febrícula y artralgias durante los primeros días de la evolución. Habitualmente la erupción dura entre 6 y 8 semanas y se resuelve espontáneamente sin dejar ninguna lesión residual. De todas formas, se han descrito también casos típicos de pitiriasis rosada más duraderos, que han persistido hasta 5 meses antes de regresar. El cuadro típico es muy característico y no se presta a confusión con otras patologías. De todas formas se han descrito también formas clínicas atípicas de pitiriasis rosada de Gibert. En estas formas atípicas y debidas a que no existe ninguna prueba analítica que nos permita confirmar el diagnóstico, el diagnóstico de pitiriasis rosada de Gibert siempre se establece con menos seguridad que en los cuadros típicos. Se han descrito variantes urticariformes, purpúricas, vesiculosas, pustulosas, similares al eritema exudativo multiforme, invertidas (con afectación preferente de los pliegues axilares e inguinales) y pitiriasis rosada con afectación de la cara o de la mucosa oral. PITIRIASIS ROSADA Dr. L. Requena 70.- Masculino de 52 años de edad, jardinero, acude a consulta debido a que presenta unas lesiones en su mano y antebrazo. La lesión inicial fue una pápula pequeña, no dolorosa que se expandió lentamente y desarrolló un área central necrótica. Posteriormente el paciente presentó numerosos nódulos subcutáneos en su antebrazo. Los nódulos aumentaron en tamaño progresivamente, refiere que uno de los nódulos se ulceró (aproximadamente 1 mes después de la aparición de las lesiones). No refiere síntomas sistémicos. También notó que los nódulos se encuentran acomodados como en cadena. El diagnóstico más probable es:

a) Aspergilosis b) Candidiasis c) Mucormicosis d) Esporotricosis

DEFINICIÓN La esporotricosis es una infección subaguda o crónica causada por el hongo dimorfo Sporothrix schenckii. Es una enfermedad cosmopolita y probablemente la micosis subcutánea más frecuente en todo el mundo.1,2 Micosis subcutánea, se localiza preferentemente en cara y extremidades, afecta piel y vasos linfáticos, se caracteriza por nódulos o gomas que dan lugar a lesiones fijas verrugosas o linfangíticas, de evolución subaguda o crónica,. En raras ocasiones es extracutánea o sistémica afectando pulmón, huesos o articulaciones. EPIDEMIOLOGÍA

México: Sur del D. F.,Puebla, Guanajuato,San Luis Potosí y Estado de

Page 82: Examen final 20460

México. Jalisco y Nayarit. Se han comunicado epidemias familiares y en empacadores de loza.

Clima templado y húmedo, promedio entre 20 y 25°C y humedad relativa superior a

90%. La mayoría de los casos se presenta a finales de otoño y principios de invierno. Se ha aislado del suelo, detritus vegetales, madera, hojas y ramas ya sean frescas o

secas, paja y zacate.

Campesinos, jardineros, floristas y carpinteros. Se presenta en igual proporción en hombres que en mujeres. Grupos de edad más afectados: niños entre 5 y 15 años en un 30% de los casos y

entre los 16 y los 35 años de edad en un 50% de los casos.

El período de incubación en los casos cutáneos es de una semana a un mes. En los raros casos pulmonares se desconoce.

Vía de ingreso por un traumatismo cutáneo PATOGENIA

Esporotricosis cutánea

La lesión inicial es un chancro esporotricósico, constituido por una lesión nodular o gomosa, ulcerada,dos semanas después se presenta un complejo cutáneo linfangítico, siguen los linfáticos regionales, puede involucionar y presentar cura espontánea.

Cuando el proceso continua se extiende por contigüidad presentando una cadena de gomas eritematovioláceas,no dolorosas que siguen los vasos linfáticos regionaleshasta el linfático mayor.

CUADRO CLÍNICO

Linfangítica ascendente de las extremidades

• Presencia de lesiones gomosas dstribuidas a lo largo del trayecto linfático, en especial de extremidades superiores, siguiendo de manera paralela la extremidad.

CUADRO CLÍNICO

CUTÁNEO FIJA (20 a 30%) Forma crónica

No tiende a la diseminación Lesión única Verrugosa o vegetante Borde eritemato-violáceo Bordes bien limitados Cubierta con escamas y costras melicéricas Asintomática Tendencia a la curación

DIAGNÓSTICO DE LABORATORIO

Examen directo no se emplea da resultados negativos Cultivo son el mejor método. Sabouraud y micosel agar Incubación 28°C Crecimiento 5 a 8 días

Page 83: Examen final 20460

En medios de cultivo ricos (gelosa sangre, BH etc) se obtienen colonias levaduriformes.

TRATAMIENTO

Yoduro de Potasio es la terapia de elección

Dosis Niños 1 a 3 g. al día Dosisn adultos 3 a 6 g. al día 20 g. de KI en 300 cc de agua Frasco obscuro (15 cc de solución (una cucharada sopera) da una concentración de 1 g). Tiempo de Tx. 3 meses promedio y continuar 2 meses más.

Rodríguez GH, Magaña RMC, Juárez L, Arenas R. Esporotricosis cutánea diseminada: comunicación de un caso. Dermatol Rev Mex 2008;52(5):228-30. La versión completa de este artículo también está disponible en: www.revistasmedicasmexicanas.com.mx

71.- Femenino de 46 años, acude a consulta por presentar cefalea frontal de predominio matutino. Antecedentes ginecoobstétricos: ciclos menstruales irregulares. E.f.: estatura 1.56 m, peso 82 kg, T/a 140/100, glucemia en ayunas de 120 mg/dl, triglicéridos 245 mg/dl, Hdl 37, colesterol total 320 mg/dl.

El diagnóstico es :

a) Obesidad mórbida. b) Síndrome metabólico. c) Hipertensión arterial esencial. d) Síndrome premenopáusico.

Page 84: Examen final 20460

Referencia:

OMS

DM o glucosa de ayuno anormal o intolerancia a la glucosa o resistencia a la insulina y 2 más:

72.- Femenino de 27 años, acude al servicio de consulta externa refiriendo presentar desde hace varias semanas temblor fino distal, sensación de angustia, sudoración palmar, ha perdido peso en los últimos meses. Sin antecedentes de importancia. Exploración física: T-A 130/86 mm hg, peso 54 kg., talla 160 cm., exoftalmos, sudoración palmar, piel húmeda y caliente, FC 110 lpm, abdomen con ruidos peristálticos incrementados, con aumento en el número de evacuaciones:

El tratamiento de primera elección para esta paciente es:.

a) Yodo 131. b) Yoduro. c) Metimazole. d) Tiroidectomía.

_ El bocio tóxico difuso (BTD) constituye la forma más frecuente de hiperfunción de la glándula tiroidea (70 % de los casos), que puede aparecer a cualquier edad, aunque por lo general aparece entre la tercera y cuarta década de la vida. Esta enfermedad es más frecuente en la mujer, donde se observa un predominio de 7:1 en relación con los hombres en regiones no bociógenas. Esta relación se reduce en las zonas de bocio endémico. Los factores genéticos desempeñan un papel esencial en la etiología y existe una predisposición familiar a esta enfermedad de Graves-Basedow. El BTD se caracteriza por la presencia de hipertiroidismo, bocio difuso y elástico, oftalmopatía, dermopatía, acropaquia tiroidea y onicolisis. Es importante el diagnóstico y tratamiento precoz del hipertiroidismo para evitar complicaciones, principalmente las cardiovasculares. TABLA I 1. Piel : • Piel fina, caliente y sudorosa. • Prurito. • Pelo fino y frágil. Onicolisis. 2. Sistema cardiovascular: • Taquicardia, palpitaciones y fibrilación auricular. • Insuficiencia cardiaca, angor pectoris, disnea de esfuerzo, vasodilatación. • Disminución de la respuesta a la digital.

Page 85: Examen final 20460

3. Aparato digestivo: • Hiperdefecación. • Disfunción hepática: hipertransaminasemia. 4. Aparato locomotor: • Debilidad y atrofia de la musculatura proximal. • Osteoporosis. • Aumento de la maduración ósea en niños. • Hiperreflexia, temblor distal, mioclonias. 5. Sistema nervioso: • Irritabilidad, nerviosismo e insomnio. • Psicosis, hipercinesia. 6. Otros: • Pérdida de peso a pesar de la polifagia. • Intolerancia al calor. • Alteraciones menstruales y disminución de la fertilidad en mujeres. • Pruebas de función tiroidea: a) Determinación de hormonas tiroideas: – L-tiroxina (T4) que circula en plasma unida en su mayoría a la proteína transportadora (TBG), y menos del 0,1% libre. Aunque sólo esta pequeña porción está libre, es la concentración de T4 libre más que la T4 total la que indica la actividad tiroidea; por lo tanto, en la mayoría de los casos sólo es necesario determinar la T4 libre, que estará elevada en los casos de hipertiroidismo. – L-triyodotironina (T3): se produce por la desyodación periférica de T4; es regulada por factores independientes de la función tiroidea. En algunos pacientes hipertiroideos la concentración de T3 está elevada cuando no lo está la de T4.

– TSH, producida por células de la adenohipófisis, que controla la función tiroidea por acción directa positiva y que es controlada a su vez por la

– TRH hipotalámica con efecto positivo estimulador. En los casos de hipertiroidismo primario la TSH estará inhibida por el sistema de retroalimentación debido al exceso de hormonas tiroideas periféricas (T4 y T3).

La TSH tiene una mayor sensibilidad para el diagnóstico del hipertiroidismo que la T4 libre. Sin embargo, no es del todo específica, es decir, una TSH baja no siempre indica hipertiroidismo. – TRH sintetizada a nivel hipotalámico. No se suele determinar en la práctica clínica. Existen algunos casos en los que la concentración de T3 yT4 se encuentran en el límite superior de la normalidad y persiste la sospecha de hipertiroidismo. En estos casos la administración de TRH no produce ningún in cremento en los niveles de TSH en los hipertiroidismos primarios (test de TRH para TSH). Así pues, para el diagnóstico de un hipertiroidismo utilizaremos básicamente los niveles de TSH y T4 libre. TRATAMIENTO

• Drogas antitiroideas metimazol 15 a 75 mg/día propiltiouracilo 150 a 750 mg/día durante 18 a 24 meses 40 a 60% de recaídas

• I 131 • Cirugía • Sintomático: bloqueo

Hipertiroidismo: clínica, diagnóstico y tratamiento I. M.ª RECHE MOLINA, B. VALERA, C. HIDALGO, L. LEÓN, G. PIÉDROLA Servicios de Medicina Interna y Endocrinología. Hospital Universitario Virgen de las Nieves. Granada

Page 86: Examen final 20460

73.- Masculino de 14 años de edad inicia con fiebre de 38.Oc y dolor en el cuadrante inferior derecho del abdomen, náusea sin vómito. A la EF se encuentra abdomen blando, no hiperestesias, a la palpación del cuadrante inferior derecho dolor en el punto de McBurney, Psoas y Obturador positivos, timpanismo normal y perístasis con algunos tonos metálicos en cuadrantes derechos: La etiología más frecuente de la apendicitis aguda es:

a) Infecciosa b) Vascular c) Primaria d) Obstructiva

ETIOPATOGENIAETIOPATOGENIA

Obstrucción del lumen apendicular. Hiperplasia linfoide. Obstrucción por fecalito, parásito o cuerpo extraño.

Inicialmente: isquemia, edema y acumulación de moco Pus por las bacterias. Destrucción de la pared apend. (apend. Aguda supurada), dolor en FID.

ETIOPATOGENIAETIOPATOGENIA

Más tarde: trombosis de vasos sanguíneos, necrosis de la pared y gangrena (migración de bacterias), irritación peritoneal localizada o generalizada.

Finalmente: Se perfora con escape de contenido purulento; peritonitis. Plastron o absceso apendicular.

Page 87: Examen final 20460

74.- Masculino atendido en el servicio e hospitalización. Presenta el siguiente hemograma: Hb 9 g/dl, VCM 120 fl, HCM 34 pg, reticulocitos 12%, leucocitos 9.500/mcrl, plaquetas 320.000/mcrl. En el frotis se observa hipersegmentación de neutrófilos. El tipo de anemia que presenta este paciente es:

a) Aplásica. b) Hemolítica. c) Ferropénica. d) Megaloblástica.

Exámenes de laboratorio: Hemograma: En la serie roja: Macrocitosis con un Volumen corpuscular medio > 100 fL, y generalmente la Hemoglobina corpuscular media está elevada (12) Ovalositos, dacriositos, cuerpos de inclusión: En el frotis de sangre periférica (Howell- Jolly y anillos de Cabot) Incremento del índice de anisocitosis. En la serie blanca: Leucopenia en casos severos Un signo precoz de megaloblastosis carencial es la hipersegmentación de los Neutrófilos. Recuento de plaquetas: no suele alterarse Pero puede haber trombocitopenia severa (4, 12).

ANEMIA MEGALOBLASTICA: Revisión bibliográfica Jorge Gustavo Romero Valdez, Carlos Eduardo Sandoval Benetti, César Luis Sánchez Dra. Laura Adriana Acosta Revista de Posgrado de la VIa Cátedra de Medicina. N° 177 – Enero 2008 75.- Femenino de 30 años, acude a consulta externa y al realizar un examen rutinario se detecta anemia. Exploración física tiene subictericia conjuntival y mínima esplenomegalia. Antecedentes: Historia familiar de litiasis biliar en edades tempranas.

El diagnóstico más probable en esta paciente es:

a) Esferocitosis hereditaria. b) Talasemia menor. c) Deficiencia de vitamina B12 y/o ácido fólico. d) Deficiencia crónica de hierro.

La EH es una enfermedad muy heterogénea desde el punto de vista clínico. Se puede observar desde el portador asintomático hasta pacientes que presentan una anemia hemolítica crónica con grandes requerimientos transfusionales.24,25 Dependiendo de la severidad del cuadro clínico, de las cifras de hemoglobina, los niveles de bilirrubina y el conteo de reticulocitos, esta enfermedad se clasifica en 4 formas: portador asintomático, EH ligera, EH típica y EH severa.26,27 Portador asintomático. En algunas familias se ha señalado un patrón de herencia autosómico recesivo. En estos casos, los padres de un paciente afectado no presentan ninguna alteración. En ocasiones la afectación es muy leve, como ligero incremento de las cifras de reticulocitos, escasos esferocitos en periferia o fragilidad osmótica incubada alterada y puede no ser detectada por los exámenes de rutina. Debe tenerse en cuenta también que pueden ocurrir nuevas mutaciones dentro de una familia aparentando un patrón de herencia autosómico recesivo, por lo que siempre es importante un estudio minucioso de todos los miembros de la familia.3,4 EH ligera. Comprende entre el 20 y 30 % de todos los pacientes con EH autosómica dominante, los que pueden presentar una hemólisis ligera compensada.

Page 88: Examen final 20460

3,27 Los individuos son frecuentemente asintomáticos y algunos casos son difíciles de diagnosticar, ya que la anemia y la esplenomegalia son muy ligeras y en ocasiones pueden estar ausentes.28 Muchos de estos pacientes se diagnostican durante estudios familiares o cuando en la etapa adulta aparece el íctero y la esplenomegalia. Episodios hemolíticos pueden presentarse en el curso de algunos procesos infecciosos como mononucleosis, parvovirus o citomegalovirus, así como durante el embarazo, por esfuerzos físicos intensos o por sangramientos.27-30 EH típica. Entre el 50 y 60 % de los pacientes con EH autosómica dominante tienen esta forma clínica. Presentan una hemólisis compensada incompleta y una anemia de ligera a moderada. El íctero es común en niños, aunque se puede ver también en los adultos y está asociado con infecciones virales ligeras, debido a la estimulación reticuloendotelial y a un aumento de la hemólisis. Los requerimientos 10 transfusionales son esporádicos. La esplenomegalia está presente en el 50 % de los niños y en el 75 % de los adultos. Rev Cubana Hematol Inmunol Hemoter 2002;18(1):7-24 Artículos de revisión Instituto de hematología e Inmunología ESFEROCITOSIS HEREDITARIA: ASPECTOS CLÍNICOS, BIOQUÍMICOS Y MOLECULARES Lic. Mayelín Herrera García y Dra. Marianela Estrada del Cueto 76.- Un hombre de 45 años, cuyos únicos antecedentes son una hipercolesterolemia sin control y una ingesta de aproximadamente 100 gramos de alcohol diarios, consulta por dolor muy intenso en el muslo de inicio brusco hace 2 semanas, sin antecedente traumático. El paciente camina a duras penas con marcada cojera y ayudándose de 2 muletas. El diagnástico más probable es:

a) Osteoporosis transitoria.

b) Fractura por estrés del cuello femoral.

c) Espondilitis anquilosante con afectación monoarticular.

d) Necrosis isquémica de la cabeza femoral.

Aunque la NICF se produce como resultado de múltiples alteraciones, puede trazarse un cuadro clínico general en aquellos casos no traumáticos. La NICF no traumática afecta generalmente a varones en una proporción de 5:1, con predominio en la etapa madura (30-50 años) y en la raza blanca (1). Se presenta con una alta frecuencia de bilateralidad, que oscila en torno al 50% en las diversas series (2,3,4,5,6), aunque rara vez se manifiestan las lesiones simultáneamente al comienzo. . El primer síntoma es el dolor de la cadera, que aparece en la ingle o zona glútea y puede irradiarse al muslo o rodilla. Clásicamente se ha descrito como de aparición brusca, a modo de puñalada que obliga al paciente a detener su actividad inmediatamente (6,8), pero con frecuencia es un dolor de instauración insidiosa y progresiva (9,10). Característicamente el paciente refiere presentar recrudescencia nocturna de las molestias y a veces aumento con la tos. La exploración física no revela al principio más que una limitación discreta de la movilidad, predominando en las rotaciones. Esta aparente incongruencia entre una buena movilidad (especialmente la flexión) y las quejas del paciente, constituye un rasgo clínico llamativo, que puede explicarse por la conservación de las superficies articulares. Lo habitual es que no existan todavía cambios radiológicos manifiestos y que el dolor preceda entre dos y seis meses a los primeros cambios (8,10). En algunos casos no ocurre esto, observándose ya un colapso cefálico con los primeros síntomas.

Page 89: Examen final 20460

La evolución clínica es variable, con periodos de disminución o exacerbación de la sintomatología. En general el dolor va adquiriendo características cada vez más mecánicas y la cojera se acentúa. Disminuyen las rotaciones y, en menor medida, la abducción y adducción, mientras que la flexión sigue conservándose. Se puede decir que, en conjunto, la movilidad está menos limitada que en la artrosis hasta estadios avanzados, en los que ya no existen diferencias. Las manifestaciones radiológicas aparecen cuando los fenómenos reparativos, a partir del tejido vivo adyacente, modifican la estructura mineralizada del hueso. Pero debe recordarse que cualquier necrosis ósea pasa a través de un estadio prerradiológico, en el que ya pueden existir síntomas. Por este motivo, la radiología tiene un papel limitado en el diagnóstico precoz de la NICF, tardando varios meses en aportar signos . Las proyecciones radiológicas habitualmente utilizadas son la antero-posterior y la axial . Con ellas podemos conocer las características, dimensiones y posición de la lesión necrótica dentro de la cabeza. Diagnóstico de la Necrosis Isquémica de la Cabeza Femoral: Conceptos actuales. L. AGUILELLA FERNANDEZ y J. CAÑADELL CARAFI. Clínica Universitaria de Navarra. Departamento de Cirugía Ortopédica y Traumatología. (Prof. J. Cañadell Carafi).

77.- A 55-year-old female is brought to the emergency room with a sudden onset of severe chest, back, and abdominal pain that began 20 minutes ago. The pain is described as being very sharp with a tearing-like character. Her previous medical history is notable for hypertension treated with hydrochlorothiazide. On examination, her blood pressure is 145/95 mm Hg in the right arm and 119/75 mm Hg in the left arm; pulse, 105 beats/minute; temperature, 37.3°C.; and respirations, 17 breaths/minute. A chest X-ray (CXR) demonstrates a widened mediastinum, and electrocardiography (ECG) reveals nonspecific ST and T wave changes. What is the most appropriate next step in the management of this patient? a) Emergent surgery

b) Thoracic computed tomography

c) Intravenous labetalol

d) Aortography

DISECCIÓN DE AORTA: • Los predictores clínicos de mayor sensibilidad para el diagnóstico son: dolor torácico súbito e intenso, ausencia o diferencia de pulsos y ensanchamiento mediastinal.

Page 90: Examen final 20460

• El manejo médico está dirigido a disminuir la presión arterial media y de manera muy importante, a disminuir el aumento de la presión pulsátil que golpea directamente la aorta (dp/dt).

• Las medidas a tomar para este objetivo incluyen reposo absoluto, control de las cifras de presión arterial, administración de oxígeno y monitorización completa no invasiva.

• Los objetivos iniciales de reducción de las cifras de tensión sistólica deben alcanzar los 100-120 mm Hg o menos, manteniendo la perfusión a los órganos vitales.

• Los medicamentos antihipertensivos de elección son los beta bloqueadores IV, combinados con nitroprusiato de sodio. Una terapia alternativa a esta combinación, es el uso de labetalol que tiene efecto bloqueador alfa y beta.

• Khan IA, Chandra N. Clinical, Diagnostic, and management perspectives of aortic dissection. Chest 2002; 122: 311-328.

78.- Se trata de femenino de 17 años, con 6 semanas de retraso menstrual, acude por sangrado vaginal, en la ecografía se observa un saco gestacional, sin presencia de actividad cardíaca embrionaria, localizado a nivel infundibular. La paciente está hemodinamicamente estable. El tratamiento más adecuado para la paciente es: a) Administración de Metrotexate. c) Actitud expectante. d) Laparotomía urgente y anexectomía. e) Alta y revisión en una semana. Con estabilidad hemodinámica: a). El dolor será más intenso y puede irradiarse a otros sitios, a todo el abdomen o al hombro (signo de Laffon). b). La hemorragia, generalmente, no variará. c). la palpación de la tumoración anexial es muy dolorosa. d). Al examen, el fondo del saco de Douglas puede estar abombado o ser muy doloroso. e). Cuando se sospeche la rotura, no debe diferirse s confirmación por punción del fondo del saco de Douglas o del abdomen. La punción también puede realizarse en pacientes sin evidencias de complicación para poder asegurar que no existe un hemoperitoneo pequeño o de poco tiempo de evolución. f). En dependencia del tiempo de evolución del hemoperitoneo, se encontrarán signos de irritación peritoneal e intestinal por la sangre libre. g). Ante la sospecha o el diagnóstico presuntivo de embarazo ectópico, en una paciente estable, debe realizarse una laparoscopia confirmativa que puede ser seguida de cirugía laparoscópica o de una laparotomía para la operación convencional

Page 91: Examen final 20460

EMBARAZO ECTÓPICO

SITIOS DE IMPLANTACIÓN

AmpularItsmicaIntersticialInfundibular

EMBARAZO ECTÓPICO

TRATAMIENTO CON METOTREXATO Mayor éxito:

Embarazo menor a 6 SDG Masa tubaria menor de 3.5cm Feto sin latido cardíaco

Teran JMetotrexate y conducta expectante en el embarazo ectópico no rotoRev Obstet Ginecol Venez.2002:62:3

EMBARAZO ECTÓPICO

TRATAMIENTO QUIRÚRGICO

Salpingocentesis Expresión de las

fimbrias Salpingectomía Salpingostomia

lineal Resección

segmentaria

Hajenius PJ, Mol BWJ, Bossujt PMM, Ankum WN, Vander Veen F. Intervenciones para el embarazo ectópico tubaricoEn: The Cochrane Library Isue 1; Oxford Update Software.

Page 92: Examen final 20460

79.- Se trata de paciente femenino de 28 años de edad, con embarazo de término, sin antecedentes de control prenatal. G 3. C-1. Se ingresa al servicio de obstetricia por presentar actividad uterina regular y dolorosa. Ef.: Deambulante, tranquila, adecuada coloración de tegumentos, abdomen con fondo uterino a 32 cm. con producto único vivo en situación transversa dorso inferior FCF 150, al tacto vaginal cérvix dilatado a 3 cm. y membranas íntegras. Se realiza cesárea con retención de placenta la cual penetra mas de la mitad del espesor del miometrio. La alteración placentaria que presenta esta paciente es:

a) placenta increta b) placenta acreta c) placenta percreta d) placenta marginal

Es la penetración y adherencia anormal de la placenta en la pared uterina.

Se divide en: Placenta acreta. Placenta increta. Placenta percreta.

ACRETA: Las vellosidades se adhieren al miometrio.

INCRETA: Penetran más de la mitad del espesor del miometrio.

PERCRETA: Atraviesa todo el espesor del miometrio, llegando a la

serosa, incluso atravesándola y adhiriéndose a órganos vecinos. Factores:

Endometriósis previa. Tumores submucosos. (Miomas) Cicatríz uterina previa. (Cesárea, miomectomía) Implantación baja. (Placenta previa) Malformaciones placentarias. (Placenta extracorial) Legrado enérgico previo. Extracción manual previa de una placenta.

Diagnóstico transparto-:

Placenta retenida por más de 20 minutos. Imposibilidad para encontrar un plano de separación placentaria cuando se intenta su

extracción manual. Hemorragia incontrolable después de la pseudoextracción. El diagnóstico histopatológico corrobora el diagnóstico clínico. Escenario menos deseable.

Tratamiento:

Histerectomía Obstétrica.

Page 93: Examen final 20460

Constituye una cirugía no planeada y secundaria al hallazgo del acretismo placentario con sangrado incohercible.

Cesárea-Histerectomía. (Con diagnóstico previo)

Cirugía planificada ante un correcto diagnóstico prenatal.

Recomendación ACOG: Maduración pulmonar intrauterina. Inyectar al cordón umbilical 50 mg de metrotexate. Ligar el cordón en el nacimiento placentario y dejar la placenta in-situ. Embolización inmediata de arterias uterinas bilaterales, así como de ramas de

la división anterior de la arteria iliaca interna con alcohol polivinílico. Continuar con 5 dosis I.M. de 50 mg de metrotexate y cuantificar niveles de

βhCG. Programar Histerectomía Total Radical Abdominal y/o Cistectomía parcial y/o

resección pared anterior recto. Lee et al. Conservative Management of Placenta Percreta. Obstet Gynecol, 112(2):421-424 80.- Gestante de 38 semanas que ingresa con trabajo de parto. Durante el período de dilatación presenta cuadro de dolor brusco. A la exploración usted objetiva metrorragia escasa obscura y aumento del tono uterino a la palpación abdominal que resulta muy doloroso. El diagnóstico más probable es:

a) Rotura uterina. b) Desprendimiento de placenta. c) Crioamnionitis hemorrágica. d) Rotura de vasos previos.

Diagnóstico diferencial entre DPPNI y placenta previa.

Síntoma o Signo Placenta previa DPPNI

Inicio del cuadro Hemorragia Dolor espontáneo o a la presión Contracciones uterinas AbdomenPalpación de partes fetales Frecuencia cardiaca Fetal Signos de toxemia asociados Tacto Vaginal

Lento, solapado Abundante, recidivante, color rojo claro No En general, no Blando, depresible Si En general, normal Raros (según incidencia general) Se tacta placenta

Brusco, tempestuoso Sangre oscura y escasa, hemorragia persistente y única Si Si Contraído, duro Difícil o imposible Negativa con frecuencia Frecuentes (50% de casos aproximadamente) No se tacta placenta

Page 94: Examen final 20460

81.- Preescolar de 3 años es llevado a consulta por preentar hiporexia. Antecedentes: Originario de zona rural del estado de Guerrero, geofagia positiva, dolor abdominal, cólico desde hace varios meses. Las evacuaciones en los últimos 5 días son semilíquidas acompañadas de moco y pujo no sangre. E.F.: mala higiene personal, desnutrido, abdomen globoso blando y dolor a la presión en colon descendente, peristalsis aumentada. El tratamiento de elección más adecuado para este paciente es:

a) Piperazina. b) Pamoato de pirantel. c) Mebendazol. d) Metronidazol

Introducción. La trichuriosis es una geohelmintiasis frecuente en zonas tropicales, rurales Tratamiento. Los fármacos de elección son albendazol (400 mg) y mebendazol (500 mg). La evidencia clínica sugiere que las dosis únicas de ambos fármacos tienen excelente eficacia en el tratamiento de la ascariasis, pero son menos efectivos contra N. americanus y T. trichiura. La tribendimina, fármaco originario de China, parece prometedor en el tratamiento de las helmintiasis transmitidas por el suelo, principalmente en infecciones con Ascaris lumbricoides y uncinarias no resistentes a levamisol y pirantel. (Shu-Hua Xiao, et al., 2005; Hotez PJ, et al., 2006; 2009 Hu et al.). El prolapso rectal incompleto puede reducirse manualmente. De manera simultánea deben tratarse la desnutrición y anemia. Es importante la profilaxis, que contempla la educación para la salud, con la promoción de la higiene personal y ambiental.

TRICHURIOSIS

Dra. Teresa Uribarren Berrueta

Departamento de Microbiología y Parasitología, Facultad de Medicina, UNAM October, 2010

Vínculos. - Mohammad S. Khuroo, Mehnaaz S. Khuroo, and Naira S. Khuroo. Trichuris dysentery syndrome: a common cause of chronic iron deficiency anemia in adults in an endemic area (with videos). Gastrointestinal Endoscopy, Jan 2010; 71(1):200-204. doi:10.1016/j.gie.2009.08.002 - Geary TG, Woo K, McCarthy JS, Mackenzie CD, Horton J, Prichard RK, de Silva NR, (...), Bundy DA. Unresolved issues in anthelmintic pharmacology for helminthiases of humans. Int J Parasitol 2010;40(1):1-13. doi:10.1016/j.ijpara.2009.11.001 Geohelmintos y otros nematodos. - Kyung-Sun Ok, et al. Trichuris trichiura Infection Diagnosed by Colonoscopy: Case Reports and Review of Literature. Korean J Parasitol. Sept 2009;47(3):275-280 DOI: 10.3347/kjp.2009.47.3.275 - Hu Y, Xiao S-H, Aroian RV. The new anthelmintic tribendimidine is an L-type (Levamisole and Pyrantel) nicotinic acetylcholine receptor agonist. PLoS Neglected Tropical Diseases 2009;3(8), art. no. e499. - Jackson JA, Friberg IM, Little S, Bradley JE. Review series on helminths, immune modulation and the hygiene hypothesis: Immunity against helminths and immunological phenomena in modern human populations: Coevolutionary legacies? Immunology 2009;126 (1):18-27. doi:10.1111/j.1365-2567.2008.03010.x

Page 95: Examen final 20460

82.- Masculino de 12 años acude a consulta externa por presentar fiebre de 8 hrs. de evolución, malestar generalizado, presenta erupción vesicular generalizada la cual inicia en tronco, se extiende a cara y miembros pélvicos acompañado de prurito. E.F.: presenta lesiones vesiculosas de predominio en tronco, escasas en cara y extremidades, de tipo macular, papular y vesicular. En el transcurso de las siguientes 48 horas, acuden a consulta un total de 10 estudiantes con misma sintomatología y como dato relevante conviven en el mismo dormitorio. De acuerdo a la información referida en el caso, nos encontramos ante un (a):

a) Epidemia. b) Endemia. c) Brote. d) Incidecia.

De acuerdo a la NOM- 017 un brote se define como la ocurrencia de dos o más casos asociados epidemiológicamente (tiempo, lugar y persona) entre sí. La medida cuantitativa de la extensión de un brote es la Tasa de Ataque (TA) que se calcula dividiendo el número de casos nuevos entre el total de personas expuestas por 100.

Norma Oficial Mexicana NOM-017-SSA2 -1994, Para la vigilancia epidemiológica. Apartados 3.1.6 y 3.1.78.1

83.- Masculino de 5 años de edad quien presenta padecimiento actual de un mes de evolución refiriendo márgenes de atención breves, descenso del rendimiento académico. Canalizado con facultativo quien lo hace correr ante lo cual el paciente presenta mirada fija y parpadeo durante 12 seg. La siguiente medida que se debe tomar en es éste paciente es: a) Electroencefalograma b) Tomografía cerebral c) Valoración por Psiquiatría d) Cambio de Médico Las epilepsias son los trastornos paroxísticos neurológicos más frecuentes durante la infancia. Se manifiestan aproximadamente en uno de cada 100 niños, con una amplia variedad de síndromes epilépticos propios de cada edad pediátrica, que tienen una sintomatología clínica característica, un patrón electroencefalográfico específico, un tratamiento farmacológico bastante bien definido y un pronóstico muy diferente, pudiendo diferenciarse unos síndromes epilépticos benignos y otros de mal pronóstico. Por epilepsia se entiende el padecimiento de dos o más crisis paroxísticas de naturaleza indudablemente epiléptica; es decir, producidas por una actividad anormalmente elevada a nivel neuronal. Es un diagnóstico clínico, que sitúa al electroencefalograma (EEG) como elemento importante para ratificar y concretar el síndrome epiléptico, pero que por sí solo nunca debe condicionar el diagnóstico.

Page 96: Examen final 20460

Convulsión : Un tipo de crisis caracterizada por movimientos intensos y bruscos de extensión y flexión en las extremidades asociado a perdida de la conciencia de diversas génesis

Epilepsia : Afección crónica, recurrente de crisis paroxisticas (epilepticas), desencadenadas por descargas de neuronas, asociadas a manifestaciones clinicas y paraclinicas Clasificación

Generalizadas * Tónico clónicas (gran mal) * Ausencias (pequeño mal) * Mioclonus bilateral * De la infancia - Espasmo infantil - Crisis atónicas - Crisis tónicas Clasificación

Parciales * Simples - Motoras (Jaksonianas) - Sensoriales (auras) - Con síntomas autonómicos - Afectivas

Parciales * Complejas - Lóbulo temporal - Con sintomas afectivos - Síntomas psicosensoriales - Síntomas de automatismo sicomotor

Parciales secundariamente generalizadas * Parcial simple sec. Generalizada * Parcial compleja sec. Generalizada * Parcial simple sec. Parcial compleja sec. Generalizada ESTUDIOS

ELECTROENCEFALOGRAMA MAPEO CEREBRAL VIDEOELECTROENCEFALOGRAMA EEG TELEMETRIA EEG CON ELECTRODOS DE PROFUNDIDAD

UTILIDAD DEL EEG La neurología y la psiquiatría trabajan con el mismo órgano blanco, el cerebro, y las mismas neuronas que pueden mal funcionar y dar un tipo específico de epilepsia, pueden provocar un trastorno psiquiátrico. El cerebro del niño es más susceptible a los cambios o las variaciones en su función eléctrica debido a factores intrínsecos de la edad como son la mielinización y los

Page 97: Examen final 20460

factores activos de plasticidad cerebral. El EEG es una herramienta muy útil en la epilepsia. Se debe realizar lo más pronto posible después de haberse presentado la crisis.3 Es posible que el EEG se encuentre normal en un paciente que tiene crisis epilépticas. Esto puede deberse a que la descarga se encuentre en áreas a las cuales el EEG convencional tiene poco acceso, o bien que no haya sido posible registrar la anormalidad en el espacio de tiempo en el que se practicó. Existen múltiples artículos que intentan establecer patrones eléctricos para patologías psiquiátricas específicas, pero todos ellos concluyen que no son patognomónicos del trastorno.9,10 En diversas series se han reportado incluso anormalidades eléctricas en niños clínicamente sanos. En un estudio que publicamos en la Revista de Neurología,11 analizamos 1000 estudios de EEG en niños con trastornos psiquiátricos infantiles, eliminando a todos aquellos que presentaban epilepsia. Los estudios normales fueron 378 (37.8%) y los anormales 619 (61.9%), de los cuales se observaron datos inespecíficos de inmadurez en 548 (54.8%), incremento en la excitabilidad tálamo–cortical en 45 pacientes (4.5%) y actividad epileptiforme en 26 niños (2.6%). El artículo concluyó: a) No existen hallazgos constantes en todos los pacientes con similares cuadros clínicos, b) no son específicos de un trastorno psiquiátrico, c) no pueden ser utilizados como indicadores del pronóstico, d) mucho menos como pauta para decidir un tratamiento farmacológico especifico, pues este tipo de decisión debe realizarse conforme a los datos clínicos de cada paciente. Las anormalidades en el EEG de pacientes con trastornos psiquiátricos infantiles son frecuentes, indican disfunción cerebral, pero sabemos que existe mucha controversia en lo que pueden representar clínicamente. 84.- Masculino de 15 meses de edad es atendido en urgencias por cuadro de crisis convulsivas generalizadas, de 3 minutos de duración. Sin antecedentes anteriores de convulsiones. Exploración física: peso 11 kg, TALLA 78 CM, TEM 39 ºC, FC 160 LPM, FR 34 rpm, saturación de oxígeno 98%. Bien hidratado, fontanela normotensa, ambos tímpanos de aspecto normal, faringe sin inflamación, examen cardiopulmonar normal. Abdomen no distendido, blando, no visceromegalias. Tres días después el paciente presenta un exantema en el tronco, la fiebre desaparece. El agente etiológico en este caso es:

a) Virus de la rubeola. b) Herpes virus tipo 6. (roséola) c) Estreptcoco beta hemolítico del grupo a. d) Parvovirus b19.

La roséola, roseola o exantema súbito,1 es una enfermedad de los niños, especialmente de los menores de dos años,2 cuyas manifestaciones exteriores se suelen limitar a una erupción transitoria o "exantema", que se produce luego de fiebre durante tres días.

Hasta hace poco, su origen era desconocido, pero ahora se sabe que es causada por dos herpesvirus de los humanos, HHV-6 y HHV-7, los cuales se conocen como virus de la roséola. Hay dos variantes del HHV-6 y los estudios en Estados Unidos, Europa y Japón han demostrado que el exantema súbito es causado principalmente por el HHV-6B que infecta a más del 90% de los bebés de 2 años o menos. La investigación indica que la mayoría de bebés con infección congénita por el virus HHV-6A, heredó el virus en un cromosoma.3

Generalmente la enfermedad afecta a niños de entre seis meses y dos años de edad, y comienza con una fiebre alta repentina (39 a 40° C). En casos raros puede causar convulsiones febriles debido a la repentina subida de la temperatura corporal, pero en muchos más casos el niño parece normal. Después de unos días ha cedido la fiebre y, en la medida que el niño parece estar recuperándose, aparece una erupción roja. Esta por lo general comienza en el tronco, extendiéndose hacia las piernas y el cuello. La erupción no da comezón y puede durar entre 1 y 2 días.4 En contraste, un niño que sufre de sarampión por lo general parece más enfermo y

Page 98: Examen final 20460

presenta síntomas como, conjuntivitis y tos y luego la erupción afecta la cara durante varios días. Disfunción hepática puede ocurrir en casos excepcionales.

Los pocos adultos que presentan reactivación del HHV-6 pueden mostrar signos de mononucleosis.5

Autores: José Francisco Médico Pediatra. Doctor en Ciencias Médicas. Profesor de Puericultura y Pediatría. Escuela Vargas. UCV. Belkys Blanco Dermatóloga Natilse Rondón Lárez Dermatóloga

CAPITULO 112: Enfermedades infecciosas eruptivas

Publicado el 3 abril, 2011 por admin

85.- Masculino de 6 años, ingresa al servicio de urgencias por presentar tos y dificultad respiratoria desde hace 24 horas. Sin rinorrea ni dolor en garganta, afebril. Antecedentes: múltiples visitas a urgencias desde los 3 años por cuadros similares, no recibe tratamiento actualmente. E.F.: Fr 35 rpm, Fc 112 lpm, saturación de oxígeno 92%. Alerta, puede hablar en frases cortas, uso de músculos accesorios leve, en tórax: tiene sibilancias tanto inspiratorias como espiratorias. Se inicia administración de oxígeno, además de éste, el tratamiento inmediato de primera elección para este paciente es:

a) Aplicación de epinefrina por vía subcutánea. b) Antileucotrieno vía oral. c) Nebulizaciones con salbutamol. d) Nebulizaciones con budesonida.

Los cuadros obstructivos del lactante son la causa más frecuente de consulta tanto a nivel primario como en los Servicios de Urgencia y son también causa frecuente de hospitalización en los Servicios de Pediatría del país, hecho que es más acentuado en los meses más fríos de cada año. Además de una alta incidencia de cuadros agudos, el sindrome bronquial obstructivo (SBO) del lactante puede presentarse en forma recurrente o recidivante, constituyendo una patología crónica que origina gran demanda asistencial y complicaciones o secuelas a largo plazo. El SBO plantea problemas en la determinación del diagnóstico etiológico debido a la similitud de la forma de presentación y a la gran variedad de causas que pueden producirlo (alrededor de 20 causas, siendo 6 a 8 las más frecuentes). Las infecciones virales son el factor gatillante más comun de los episodios obstructivos, sin embargo varios otros factores son también responsables de la gran frecuencia de estos cuadros. Dentro de estos factores se cuentan la contaminación intradomiciliaria, en especial es necesario mencionar la importancia que tiene el tabaquismo familiar y el uso de tipos de calefacción con combustión impura como son el brasero y las estufas de parafina. Por otra parte, en los últimos años la contaminación ambiental en las grandes ciudades ha jugado un rol agravante muy importante en los cuadros obstructivos Los cuadros obstructivos en el lactante se manifiestan por síntomas y signos clínicos caracterizados por tos, sibilancias, espiración prolongada, aumento de diámetro anteroposterior del tórax, retracción costal, hipersonoridad a la percusión. Estos hallazgos al examen físico son inespecíficos y no nos orientan hacia una etiología determinada. La tabla 2 detalla algunas características clínicas que hacen sospechar algunas etiologías. TABLA 1. Condiciones congénitas y adquiridas que se asocian a sibilancias en niños pequeños

Page 99: Examen final 20460

Frecuentes Poco frecuentes Raras

Infecciones virale(SAIV)

Bronquiolitis: 1eepisodio

Episodios recidivantes Asma bronquial

Fibrosis quística Displasia

broncopulmonar Cardiopatías congénita Cuerpo extraño en ví

aérea

Masas mediastínica(tumores, TBC)

Inmunodeficiencias Disquinesia ciliar Bronquiolitis

obliterante Bronquiectasias Síndromes aspirativos Malformaciones: Anill

vascular Maladenomatoide quística

Quiste broncógenos TABLA 2 Causas de Síndrome bronquial obstructivo secundario y algunas características orientadoras en su etiología Etiología Características Laboratorio

Displasia broncopulmonar prematurez radiografía de tórax

SDRI del RN

ventilación mecánicperíodo RN

dependencia de oxígeno 28 días

Fibrosis quística desnutrición electrólitos en sudor

síndrome de malabsorció

neumopatías a repetición

Cardiopatía congénita soplo radiología

insuficiencia cardíaca ECG

ecocardiografía, doppler

Aspiración cuerpo extraño episodio asfíctico brusco radiología

signos pulmonareasimétricos

broncoscopía rígida

Reflujo gastroesofágico epacientes predispuestos

vómitos recurrentes radiología

pH metría

Trastornos de la deglución neumopatía recurrenteprolongada

cintigrafía de aspiraciópulmonar

daño neurológico

Malformaciones pulmonares hallazgo radiológico ecografía prenatal

radiografía de tórax

TAC

angiografía

resonancia magnética

Malformación vascular estridor radiología, esofagograma

endoscopia

eco doppler

angiografía

resonancia magnética

Disquinesia ciliar patología sinusal radiología

situs inverso biopsia epitelirespiratorio

Page 100: Examen final 20460

TABLA 1. Puntaje clínico modificado de Tal y cols. (modificación nacional)

PuntajeFrec. resp.< 6 m. > 6m.

Sibilancias* Cianosis Uso musculatura

0 <40 > 30 No No No

1 41-55 31-45 Sólo final espiración Peri oral Conllanto

(+) Subcostal

2 56-70 46-60 Esp Insp. con estetoscopio Peri oral En reposo

(++) Sub eintercostal

3 > 70 > 60 Esp Insp sin estetoscopio oausentes

Generalizada En reposo

(+++) Supraesternal Sub eintercostal

* Sibilancias pueden no auscultarse en obstrucción muy grave. Obstrucción Leve: 0 a 5 Moderada: 6 a 8 Grave: 9 a 12 ------------------------------------------------------------------------------------------------------ La valoración adecuada de los signos y síntomas de gravedad de la crisis obstructiva, permitirá racionalizar su tratamiento para disminuir la morbimortalidad por este síndrome, así como programar junto a los padres la prevención y reconocimiento precoz de nuevos episodios obstructivos. Se debe considerar en forma especial a los pacientes del grupo de riesgo. Grupo de Riesgo

1. Menor de 3 meses 2. Hospitalización previa por SBO severo 3. Antecedentes de ventilación mecánica por SBO 4. Prematurez 5. Paciente con SBO secundario 6. Requerimiento previo de curas con esteroides sistemáticos 7. Falta de cumplimiento del tratamiento, o conflicto entre los padres y el equipo médico en

relación al tratamiento. I.- MANEJO EN EL SERVICIO DE URGENCIA: Objetivos:

1. Corregir la hipoxemia. 2. Corregir rápidamente la obstrucción bronquial. 3. Indicar claramente el manejo ulterior en domicilio y derivación apropiada.

Tratamiento Inicial A) Oxigenoterapia: el objetivo es lograr una Sa02 > 95%. El oxígeno debe administrarse desde el principio con puntaje > 6, incluso cuando no se dispone de oximonitor de pulso, con mascarilla, naricera o halo según la tolerancia individual. B) Broncodilatadores: Salbutamol: en aerosol de dosis medida 2 puffs (200 mcg) cada 10 min. 3 veces o en nebulización con solución al 0.5%, 0.05 ml/kg (0.25 mg/kg), (mínimo 0.25 ml, máximo 1 ml) cada 20 min. por 3 veces. En episodios leve y moderado se debe privilegiar la aerosolterapia. Preferir la nebulización en pacientes graves o con Sa02 < 90% respirando aire. 1.- Técnica de Aerosolterapia

Debe utilizarse aerocámara de aproximadamente 450 ml de volumen y 18 cms de longitud.

Agite el inhalador presurizado y colóquelo en posición vertical en la parte posterior de la aerocámara.

Aplique la aerocámara sobre la boca y nariz, con el niño sentado. Administre un puff y permita que el niño respire 5 a 6 veces o espere 10 segundos sin

retirar la aerocámara. Espere 3 a 5 minutos para aplicar un segundo puff.

Page 101: Examen final 20460

2.- Técnica de Nebulización Nebulizador tipo Hudson o similar. Gas: Oxígeno Flujo: 6 8 lt/min Duración o volumen total: completar a 4 ml con suero fisiológico.

3.- Esteroides sistémicos: deben prescribirse: Si el episodio es grave (hospitalización UCI, Intermedio) Si el episodio forma parte de un cuadro obstructivo frecuente.

Se usa prednisona o prednisolona 2 mg/kg/dosis, oral, por 1 vez, dosis máxima 40 mg o Hidrocortisona 10 mg/kg/dosis o metilprednisolona 2 mg/kg/dosis, por vía intravenosa. Privilegiar los corticoides por vía oral, ya que son tan eficaces como los administrados por vía intravenosa. Criterios de hospitalización

Hospitalizar de inmediato a pacientes con: Puntaje > 10 Insuficiencia respiratoria global Compromiso de conciencia Convulsiones o sospecha de agotamiento Cuadro clínico asociado a apneas Persistencia de una obstrucción grave o saturación que no mejora post tratamiento

inicial. Respuesta insuficiente después de 2 horas de tratamiento. Condiciones adversas en el hogar que determinan falla en la accesibilidad a la atención

médica de urgencia frente a un agravamiento, o falla en el cumplimiento del tratamiento. BIBLIOGRAFÍA:

Morgan WJ, Martínez FD. Risk factors for developing wheezing and asthma in childhood. Ped Clin N Am 1992;39:1185-1203.

Martínez FD, Morgan WJ, Wright AL, et al. Initial airway function is a risk factor for recurrent wheezing respiratory illnesses during the first three years of life. Am Rev Respir Dis 1991;143:312-316.

Wennergren G, Kristjansson S. Relationship between respiratory syncytial virus bronchiolitis and future airway diseases. Eur Respir J 2001;18:1044-58.

Panitch HB. Bronchiolitis in infants. Curr Opin Pediatr 2001; 13:256-60. Godden DJ, Ross S, Abdalla M, et al. Outcome of wheeze in childhood. Am J Respir Crit

Care Med 1994;149:106-112. 86.- Se trata de paciente femenino de 6 años de edad la cual presenta ceguera nocturna (hemeralopia) en la exploración oftalmológica se observa constricción del campo visual con escotoma anular, pérdida de la agudeza y electrorretinograma anómalo así mismo arterias estrechadas. De las siguientes patologías es la más probable ES: a) Retinoblastoma. b) Catarata congénita o infantil. c) Retinosis pigmentaria (retinitis pigmentaria). d) Persistencia de vítreo primario Retinosis pigmentaria. Bilateral, de curso lento y progresivo, comienza en la edad escolar, pudiendo causar ceguera hacia los 40 años. Es una alteración de los bastones. Puede presentarse aislada o asociada a otras malformaciones, como la polidactilia, el síndrome de Laurence-Moon-Bield. Se hereda bajo diferentes patrones. Clínicamente presentan mala visión nocturna (hemeralopía) y escotoma anular en el campo visual .Oftalmoscópicamente se aprecian arterias estrechadas, atrofia de papila y acumulaciones de pigmento que asemejan osteocitos en la retina periférica. Evolucionan hacia una reducción del campo visual y finalmente ceguera. Además presentan complicaciones adicionales como aparición temprana de cataratas y glaucoma.

Page 102: Examen final 20460

Retinosis pigmentaria Retinosis pigmentaria, preguntas y respuestas, Universidad Miguel Hernández de Elche, 2007 Science Daily (ed.): «Retina Transplants Show Promise In Patients With Retinal Degeneration» (11-7-2008). 87.- Femenino de 37 años que está siendo evaluada por probable esclerosis múltiple. Empezó con parestesias intermitentes en ambas piernas hace varios meses y ahora tiene incontinencia urinaria. De los siguientes hallazgos físicos el típico de esclerosis múltiple es: a) Pérdidad de memoria b) Constipación c) Oftalmoplegia internuclear d) Temblor Manifestaciones clínicas

Puede ser asintomática y descubrirse como hallazgo en RMN Primer ataque no tiene premonición y puede ser monosintomático o polisintomático 20% inicio agudo con deficit máximo en minutos u horas Remisión frecuente del cuadro despues del primer ataque Las recurrencias reprsentan recrudenscencias de lesiones más tempranas o los

efectos de otras nuevas Signos y sintomas:

Debilidad o insensibilidad de una extremidad o facial Perdida de la visión monocular Oftalmoparesia – plejía internuclear Diplopia – atrofia óptica Vértigo , ataxia y nistagmo Disartraria Incontinencia urinaria Convulsiones en 3 a 4 % Cambios psiquicos

Referencias bibliográficas

1. Noseworthy JH, Lucchinetti C, Rodríguez M, Weinshenker BJ. Multiple sclerosis. N. England J Med. 2000;343:938-52. Disponible en:.

2. Omari KM, John GR, Sealfon SC, Raine CS. CXC chemokine receptors on human oligodendrocytes: implications for multiple sclerosis. Brain 2005;128:1003-15.

3. Mi S, Miller RH, Lee X, et al. LINGO-1 negatively regulates myelination by oligodendrocytes. Nat Neurosci. 2005;8:745-51.

Page 103: Examen final 20460

4. John GR, Shankar SL, Shafit-Zagardo B, et al . Multiple sclerosis: re-expression of a developmental pathway that restricts remyelination. Nat Med. 2002;8:1115-21.

88.- Masculino recién nacido que presenta una conjuntivitis neonatal, para indicar el tratamiento usted relaciona los siguientes gérmenes por ser los de mayor frecuencia causantes de esta enfermedad: a) Clamidia, gonococo, estafilococo aureus b) Clamidia, treponema, gonococo c) Estreptococo grupo A, listeria d) Listeria, pseudomonas

Las conjuntivitis neonatales (CN) siguen siendo uno de los motivos más frecuentes de consulta, pudiendo llegar a comprometer seriamente la capacidad visual del bebé, conduciendo incluso a la ceguera. La incidencia de esta enfermedad oscila entre 1,6%-12% en el primer mes de vida. Las CN pueden ser de origen infeccioso o químico. Las CN de origen infeccioso pueden ser adquiridas durante la gestación, durante el parto por contacto con la secreción uretro-vaginal, o bien en el post-parto y el ámbito de convivencia diaria. Las CN que se manifiestan entre las 24 y las 48 horas del nacimiento son de origen químico, debidas a la profilaxis efectuada por instilación de solución de nitrato de plata en el saco conjuntival del neonato, para prevenir la proliferación de Neisseria gonorrhoeae . Se debe tener presente la adquisición de la infección perinatal a partir del ambiente en que se halla el bebé. La etiología más frecuente en este caso corresponde a Staphylococcus aureus, Haemophilus influenzae y Streptococcus pneumoniae). En las últimas décadas, Chlamydia trachomatis ha resultado ser el microorganismo prevalente entre los agentes causales de enfermedades de transmisión sexual, con una prevalencia del 2 al 35% en embarazadas. La transmisión vertical se da entre el 60 y 70% en los hijos de madres infectadas .El riesgo de adquirir conjuntivitis en estos bebés es del 18 al 60%, mientras que el de neumonía es del 10 al 25%. En el caso de N. gonorrhoeae , la infección ocular puede complicar el cuadro con ceguera y artritis. Desde el año 1995, en nuestro hospital se lleva a cabo en forma rutinaria la búsqueda de gérmenes comunes (GC) y C. trachomatis en todo recién nacido con conjuntivitis.

Bibliografía

Krachmer, Manis, Holland. Cornea and External Disease: Clinical Diagnosis and Management, Vol II. Mosby, 1997.

Yanoff, Ophtalmology. Mosby, 1999.

89.- Acude a consulta masculino de 9 años de edad por presentar un rash muy prurítico por 3 días. Estuvo en un campamento durante el cual utilizó unicamente playeras de manga corta y shorts. A la EF, con temp 37.2°C, TA 96/62mmHg, FR 12x’, FC 65x’. Presenta numerosas pápulas y vesículas eritematosas en las 4 extremidades. La mayoría con un patrón lineal. El tratamiento más apropiado es: a) Cefalexina VO b) Difenhidramina tópica c) Mupirocina tópica d) Hidrocortisona tópica al 1%

Page 104: Examen final 20460

La respuesta más probable es que se trate de un PRURIGO POR INSECTOS…. En cuyo caso una loción o crema a base de hidrocortisona es suficiente.

La palabra prurigo define un proceso patológico cuya lesión elemental es la pápula y el síntoma principal es el prurito. El espectro clínico abarca un rango que va desde las pápulas (prurigo papular), nódulos (prurigo nodular) entre otros tipos de prurigo podemos mencionar al actínico, el atópico o de Besnier, el de Sutton y el pigmentoso.

A. PRURIGO SIMPLE POR INSECTOS O URTICARIA PAPULOSA

La urticaria papular o prurigo por insecto es causada como su nombre lo indica por la picadura de algún insecto. No está relacionada por factores dietéticos. La naturaleza alérgica de la condición fue demostrada por Mellanby en 1946, pero no fue hasta 1954 cuando los cambios histológicos producidos por picaduras de insecto a nivel experimental demostraron que eran idénticos a los de la urticaria papular.

Etiología y Patogénesis

Las causas más frecuentes en la dermatología pediátrica, afecta igual a ambos sexos y a cualquier etnia. Numerosos insectos causan prurigo, siendo los más frecuentes el Cemex lectularius (chinche) produciendo la Cimiasis; la pulga (Pulex irritans) que ocasiona la puliciasis y las picaduras por mosquitos (Culicidae).

Los alergenos presentes en la saliva del insecto inducen una sensibilización del paciente con formación de anticuerpos específicos. El tipo e intensidad de la reacción originada por la picadura dependerá de si el paciente ha estado ya expuesto al insecto y de la capacidad del huésped de responder al estímulo antigénico.

Las lesiones tempranas de prurigo por insecto se deben a una respuesta de hipersensibilidad tipo I causada por la liberación de IgE. Posteriormente interviene un mecanismo de hipersensibilidad tipo IV dependiente de linfocitos T que produce las lesiones tardías.

Manifestaciones Clínicas

El prurigo por insecto afecta cualquier área corporal, se presentan vesículas en la fase inicial posteriormente aparecen pequeñas pápulas eritematosas, las lesiones son muy pruriginosas por los que se observan costras hemáticas por rascado. Se observan lesiones en diferentes estadios y evolucionan por brotes.

Diagnósticos Diferenciales

La escabiosis, la acropustulosis de la infancia, la varicela y la dermatitis atópica son las patologías que con más frecuencia pueden confundirse el prurigo por insectos.

Tratamiento

Las medidas generales son de gran importancia, la explicación a los padres acerca del padecimiento, su predisposición atópica, la cronicidad y evolución por brotes.

Se recomienda además el uso de pijamas largos, mosquiteros, insecticidas cuando no están los niños. Tópicamente se utilizan lociones antipruriginosas y esteroides tópicos.

Por vía oral se utilizan antihistamínicos no sedantes. En algunas ocasiones se utiliza Tiamina por vía oral, la cual se ha recomendado empíricamente por mucho tiempo "tal vez sea un repelente natural que se excreta por la piel", dosis 200mg a 600mg por día, lo más importante es evitar la infección secundaría.

Page 105: Examen final 20460

Bibliografía

1. Salazar, A. Prurigo actínico en la niñez. Dermatol Pediatr Lat 2005; 3(3): 193-200.

2. Cazarí J, Román, D. Robles, M. Magaña. Talidomida en niños con prurigo solar refractario. Actas Dermatol 2002; 2(1-2):11-15.

3. Magaña, M. Prurigo solar. Bol Med Hosp Infant Mex 58 (6); 409-419

4. Tamayo SL, Bettencourt M; Harper J. Lichen Simplex Chronicus and Prurigo. Textbook of pediatric dermatology. Vol 1, 2000: 4.7:279-283.

5. Ferguson J. The Idiopathic Photodermatoses.Textbook of Pediatric Dermatology. Vol 1, 2000:15.1:898-900

6. Beirana A. Prurigo por Insectos. PAC Dermatologia. Sociedad Mexicana de Dermatologia: Dermatologia Pediátrica. Ed...1era, Nº 5:29-30. 2000

90.- En la hiponatremia intensa (inferiores 120 mEq/L se debe de tratar con soluciones salinas hipertónicas mediante el cálculo del déficit, reponiendo de la siguiente manera: a) Pasar el déficit de sodio en las primeras 6 hr. b) Pasar el déficit de sodio en las primeras 12 hr. c) Pasar el déficit de sodio, la mitad en las primeras 12-24 hr y se revalora. d) Pasar el déficit de sodio en las primeras 3 hr y después se revalora. Farreas. Medicina Interna. Cap. 222, Decimoquinta edición 2004, Pp. 1844-1847. La hiponatremia con volumen extracelular disminuido se trata con la administración de soluciones salinas isotónicas (al 0.9%). La cantidad necesaria de miliequivalentes de sodio se calcula de acuerdo a la siguiente formula: Na (mEq)= Na x agua total corporal, es decir Na (mEq)= (140- Na actual) x (0.6 x peso en Kg) Donde: Agua total corporal= 60% del peso = 0.6 x peso en kg En la práctica, suele administrase la mitad de esa cantidad en el transcurso de las primeras 12- 24 h y luego se efectúan reevaluaciones correctoras. 91.- Femenino de 40 años de edad acude al servicio de urgencias quejándose de calambres en las piernas y parestesias en los dedos de las manos. Un año antes se le sometió a una operación del cuello, pero no está segura qué fue lo que se le hizo. El signo de Chevostek es positivo: la percusión sobre el nervio facial por delante de la oreja desencadena una contracción espasmódica del labio superior. ¿Cuál de los siguientes trastornos sugiere un signo positivo? a) Hipercalciemia b) Acidosis c) Hiperpotasemia d) Hipocalciemia

Page 106: Examen final 20460

SINTOMATOLOGÍA Neuromuscular: la hipocalcemia aguda se manifiesta por parestesia (hormigueo y adormecimiento de los dedos y región peribucal) y calambres o contracturas musculares. La sintomatología subclínica de tetania, evidenciada por el signo de Chvostek, (ocurrencia de espasmo facial, especialmente del orbicular de los labios, al percutir el nervio facial a mitad de distancia entre la comisura labial y el oído). El signo de Trousseau es un espasmo doloroso del carpo, que se presenta luego de mantener por tres minutos una presión >20 mmHg por encima de la sistólica, siendo un signo más de tetania. La hipocalcemia crónica se presenta con irritabilidad, confusión, demencia e incluso, en infantes, como retardo mental. También se reportan movimientos coreicos, distonías y convulsiones. Se ha reportado calcificación de ganglios basales en la radiografía de cráneo, que no es reversible al tratamiento. Cardiovascular: prolongación de la fase de potencial de acción y por lo tanto prolongación del segmento ST en el ECG. En casos de severa deficiencia se presentan arritmias, hipotensión o falla cardiaca; la hipocalcemia aumenta la cardiotoxicidad de los digitálicos. Pulmonar: broncoespasmos y laringoespasmos vistos, sin embargo, con poca frecuencia. Dermatológica: piel seca, uñas quebradizas y caída del cabello como signos no específicos en la hipocalcemia crónica LECTURAS RECOMENDADAS 1. Carlstedt F, Lind L. Hypocalcemic syndromes. Crit Care Clin 2001; 17:139-153. 2. Gibbs M, Wolfson A, Tayal V. Electrolyte disturbances. En: Rosen´s Emergency Medicine. Concepts and Critical Practice. J Marx, Hockberg R, Walls R, et al (eds). Fith edition. Mosby. St Louis,1998. 3. Kapoor M, Chan GZ. Fluid and electrolyte abnormalities. Crit Care Clin 2001; 17:503-529. 4. Lind L, Carlstedt F, Rastad J, et al. Hypocalcemia and parathyroid hormone secretion in critically ill patients. Crit Care Med 2000; 28:93-99. 5. Lo CY. Postthyroidectomy hypocalcemia. J Am Coll Surg 2003; 196:497-498. 6. Marx SJ. Hyperparathyroid and hypoparathyroid disorders.N Engl J Med 2000; 343:1863-1875. 92.- Mujer de 42 años. Acude a consulta por cefalea y acúfenos. Tiene antecedente de DM en padre. EF: peso 78 kg, estatura 1.62, perímetro abdominal 108 cm, TA: 140/100. Laboratorio: glucosa de ayuno 116, prueba de tolerancia a la glucosa, resultado a las 2 horas de 189. Los diagnósticos que se establecen en la paciente son:

a) Obesidad y diabetes mellitus b) Síndrome metabólico con intolerancia a la glucosa c) Obesidad e intolerancia a la glucosa d) Síndrome metabólico y diabetes mellitus GLUCOSA: <100 mg/dl = normal 100 y < 126 = glucosa de ayuno alterada (GAA) 126 mg/dl = diabetes mellitas Glucosa a las 2 horas postcarga: 140 mg/dl = normal

Page 107: Examen final 20460

140 a 199 mg/dl = intolerancia a la glucosa 200 mg/dl = diabetes mellitus El síndrome metabólico es la agrupación de diferentes factores de riesgo asociados con el síndrome de resistencia a la insulina. En la literatura existen variaciones importantes en la prevalencia de este síndrome, dependiendo de los criterios o definición que se empleen para su diagnóstico. En la actualidad hay más de cinco definiciones del síndrome metabólico; la descrita en 2001 y su actualización en 2005 por el Programa Nacional de Educación para el Colesterol, Panel de Tratamiento del Adulto III (NCEP-ATPIII), es la más empleada en la práctica clínica. Esta definición establece que se requiere la presencia de tres o más de los siguientes cinco criterios: glucosa en ayuno ≥ 100 mg/dl, triglicéridos ≥ 150 mg/dl, colesterol HDL bajo (< 40 mg/dl en el hombre o < 50 mg/dl en la mujer), tensión arterial ≥ 130/85 mm Hg o en tratamiento para la hipertensión y obesidad abdominal detectada mediante medición del perímetro abdominal (> 102 cm en hombres y > 88 cm en mujeres).1 La obesidad ha alcanzado proporciones epidémicas en los países occidentales y, por lo tanto, es un importante problema de salud. De hecho, México ocupa el segundo lugar a nivel mundial en sobrepeso y obesidad. La resistencia a la insulina que se promueve por la obesidad abdominal o fenotipo de obesidad androide se considera la responsable de algunos factores de riesgo que se agrupan en el síndrome metabólico.2 Estudios recientes indican que la prevalencia del síndrome metabólico en la población general en México es de aproximadamente 26.6 %, similar a los resultados informados por la Tercera Encuesta de Evaluación Nacional en Salud y Nutrición en población de Estados Unidos, que fue de 26.7 %.3 Sin embargo, la prevalencia aumenta conforme avanza la edad, llegando a ser hasta de 44 % en individuos mayores de 50 años.4 El síndrome metabólico no sólo incluye algunos factores de riesgo cardiovascular tradicionales, sino otros componentes que representan aspectos involucrados en el desarrollo y progresión de enfermedades cardiovasculares, como la disfunción endotelial, marcadores de inflamación y alteraciones en la coagulación. La sinergia de la combinación de estos factores se considera causante del aumento en el riesgo de morbilidad y mortalidad asociada con enfermedades cardiovasculares en este grupo de pacientes. Los pacientes que completan criterios para síndrome metabólico tienen tres veces más riesgo de padecer enfermedad arterial coronaria y cerebrovascular.5 Más aún, en pacientes con enfermedad vascular sintomática se encontró que el síndrome metabólico se asociaba con la extensión del daño vascular.6,7 El síndrome metabólico es uno de los principales problemas de salud pública del siglo XXI. El diagnóstico es sencillo, principalmente con base en datos clínicos y bioquímicos sistemáticos, los cuales están al alcance de cualquier sistema de salud Síndrome metabólico, impacto clínico y angiográfico en pacientes con síndrome coronario agudo Volumen 78, No. 2, Marzo-Abril 2010 Alejandra Madrid-Miller,* Antonio Alcaraz-Ruiz,* Gabriela Borrayo-Sánchez,** Eduardo Almeida-Gutiérrez,* Rosa María Vargas-Guzmán,* Ricardo Jáuregui-Aguilar*** 93.- Mujer de 22 años que acude a consulta por pérdida de peso, diarrea, palpitaciones, temblor y labilidad emocional. No tiene antecedentes importantes. Refiere evacuaciones diarreicas de 5 a 7 por día, sin moco o sangre. EF: peso 52 kg, estatura 1.59, FC 108x’, TA: 120/65, retracción palpebral, hiperemia conjuntival y de carúncula, cuello con tiroides aumentada de tamaño 3 veces de forma difusa, extremidades superiores con hiperhidrosis palmar, temblor fino distal, reflejos exaltados Los estudios que debemos solicitar para hacer el diagnóstico es: a) Coproparasitoscópico, coprológico y coprocultivo b) Ultrasonido tiroideo c) Pruebas de función tiroidea d) Electrocardiograma y ecocardiograma

Page 108: Examen final 20460

EVALUACIÓN DE UN PACIENTE CON SOSPECHA DE ENFERMEDAD TIROIDEA

Page 109: Examen final 20460

94.- Masculino de 34 años, con historia de cólicos nefríticos de repetición asociados a expulsión de cálculos, se realiza estudio metabólico que muestra una hipercalciuria idiopática no influenciada por el ayuno ni la ingesta. De los siguientes, El tratamiento más adecuado para éste paciente es: a) Fosfato de celulosa. b) Dieta hipoproteica y ortofosfatos. c) Alopurinol d) Ingesta abundante de líquidos y tiacidas.

Como es posible aducir, la hipercalciuria idiopática es un grupo de padecimientos que representan a una proporción no despreciable de pacientes con cálculos del tracto urinario. Debe entenderse como hipercalciuria idiopática, una situación en la que la excreción urinaria de calcio se encuentra incrementada, mientras que las concentraciones plasmáticas de este ion son normales y no se ha identificado otra causa de dicho fenómeno. La gran mayoría de los pacientes con litiasis cálcica e hipercalciuria presentan este padecimiento como etiología del evento calculoso. Hoy se sabe que la hipercalciuria idiopática no es un padecimiento monogénico, sino que obedece a alteraciones identificadas en diversos loci. Aunque se han descrito varios mecanismos fisiopatológicos, el más frecuente de ellos se enmarca en el contexto de una hipersensibilidad de la célula intestinal a la acción de la vitamina D activa o calcitriol (1,25-(OH)2 -D3 ), lo que da como resultado un estado de hiperabsorción intestinal de calcio, hipercalcemia transitoria y la consiguiente hipercalciuria. El resto de los casos se deben a defectos tubulares en la retención de calcio o fosfatos. La restricción en el consumo de calcio en la dieta diaria es el pilar del tratamiento en estos casos. Asimismo, los diuréticos tiazídicos son de gran utilidad gracias a su efecto en la retención tubular de calcio.

Hipercalciuria idiopática en pacientes con litiasis del tracto urinario

ANALES MEDICOS Vol. 52, Núm. 1 Ene. - Mar. 2007 pp. 30 - 36

95.- Femenino de 44 años cursa con dolor epigástrico que la despierta por la mañana y se presenta también cuando tiene hambre. El comer o el tomar antiácidos lo mejora. Se la realiza una endoscopia que revela una úlcera duodenal. El mejor tratamiento para el manejo de esta úlcera es: a) Amoxacilina + Clindamicina +Bismuto b) Ranitidina + Metronidazol+ ampicilina c) IBP+ Claritromicina+ amoxicilina d) IBP+ Bismuto+ claritromicina Regímenes de tratamiento de primera línea ��Regímenes de tratamiento de triple terapia: IBP + dos antibióticos: amoxicilina y claritromicina o metronidazol y claritromicina ��Utilizados y aceptados a nivel mundial ��La terapia estándar en base a IBP fracasa en 30% de los pacientes; las tasas de erradicación han caído a 70-85% en los últimos años, en parte debido al aumento de la resistencia a la claritromicina ��Una mayor duración del tratamiento puede aumentar las tasas de erradicación, pero sigue siendo controvertido; los estudios sugieren un aumento a 14 días, en vez de 7 días ��Las consideraciones relativas al costo y los problemas de adherencia pueden aún favorecer la terapia de 7 días ��Algunos grupos sugieren el tratamiento durante 10 días. ��Terapia cuádruple: IBP + bismuto + dos antibióticos: amoxicilina + claritromicina o metronidazol + tetraciclina

Page 110: Examen final 20460

��Puede ser más barata que la terapia triple ��Más difícil de ingerir que la terapia triple ��Tasas de erradicación equivalentes o superiores Guías prácticas de la Organización Mundial de Gastroenterología Helicobacter pylori en los países en desarrollo Agosto de 2010 Equipo de revisión R.H. Hunt, Presidente (Canadá) S.D. Xiao (China) F. Megraud (Francia) R. Leon-Barua (Perú) F. Bazzoli (Italia) S. van der Merwe (Sudáfrica) L.G. Vaz Coelho (Brasil) M. Fock (Singapur) S. Fedail (Sudán) H. Cohen (Uruguay) P. Malfertheiner (Alemania) N. Vakil (EEUU) S. Hamid (Pakistán) K.L. Goh (Malasia) B.C.Y. Wong (Hong Kong) J. Krabshuis (Francia) A. Le Mair (Países Bajos) 96.- Masculino de 4 años de edad con antecedentes de otitis media a los doce meses de edad, actualmente cursa con otalgia, otorrea e hipoacusia, acompañado de malestar general. A la exploración en conducto izquierdo se aprecia Hiperemia timpánica, hipervascularización y engrosamiento timpánico. El tratamiento de elección en esta patología es: a) Antihistamínicos de forma exclusiva b) Solo medidas generales c) Drenaje, AINES y esteroides locales d) Antibióticos, antihistamínicos y AINES. Otitis media que se acompaña con secreción y signos locales, sistémicos, o ambos. Instituto Nacional de la Nutrición. “Salvador Zubiran” Manual de terapéutica médica y procedimientos de urgencias. Cuarta edición. Pag. 443 – 450. Mac Graw-Hill Interamericana. México. La otitis media se define como la inflamación del oído medio que se acompaña de secreción y signos locales, sistémicos, o ambos, de enfermedad aguda. Esta enfermedad predomina en la infancia, sin embargo también existe en el adulto. Su distribución tiene una clara periodicidad estacional, sobre todo en otoño e invierno. Los agentes de mayor prevalencia en nuestro país son: Streptococcus pneumoniae, Haemophilus influenzae, Morexella catarrhalis estreptococo del grupo A y S. aureuse. Los antibióticos más aceptados en estos casos son la amoxicilina, la amoxicilina con clavulanato, la calritromicina y el trimetropim con slfametoxazol. Se acepta el uso de antihistamínicos y antiinflamatorios no esteroideos para disminuir la congestión de la mucosa para resolver la obstrucción de la trompa faríngotimpánica; en realidad no se ha demostrado efecto sobre la duración de los síntomas. El drenaje del derrame solo esta indicado cuando éste persiste por más de tres semanas.

Page 111: Examen final 20460

97.- Masculino de 12 años que acude a consulta por presentar una protuberancia ósea en matáfisis del fémur, refiere moderado dolor en articulación de rodilla a dolor leve desde hace un mes, con datos probables de bursitis, se envía para realizar Rx los que demuestran una tumoración ósea localizada en metáfisis distal del fémur izquierdo. El diagnóstico más probable en éste paciente es: a) Quiste óseo aneurismático. b) Fibroma condromixoide. c) Encondroma. d) Osteocondroma. El osteocondroma es el tumor óseo primario más frecuente, pertenece a la serie cartilaginosa y se conoce también como exostosis osteocartilaginosa, es de localización metafisaria y por lo general su crecimiento se detiene cuando se cierra la fisis.2 Su localización más frecuente es en la metáfisis distal del fémur, en la metáfisis proximal del húmero y en la proximal de la tibia (Figura 1a y b). Existen otras localizaciones menos frecuentes como pueden ser omóplato o costillas. Metáfisis distal de fémur Metáfisis proximal de húmero tibia Metáfisis proximal de peroné Escápula, pelvis, costillas Solitario 75% 30 años Múltiples 95% 20 años H:M 1.6:1 Osteocondromatosis

‐ Autosómico dominante Tamaño 1-15cm (3-7cm) Protuberancias Asintomáticas Dolor

‐ Irritación mecánica ‐ Compresión neural ‐ Fractura del tallo ‐ Transformación maligna ‐ Asociado a pseudoaneurisma ‐ Asociado a bursitis

El osteocondroma puede o no ser sintomático, dependiendo de su localización. Una lesión sintomática, puede causarse por la irritación los tejidos blandos que lo envuelven (tendón o vientre muscular) y pueden formar un bursa llena de líquido que puede confundirnos con una masa del tejidos blandos. El osteocondroma cerca de una articulación puede limitar el rango de movimiento. Otras causas adicionales de síntomas son compresión de un nervio, la fractura del tallo, o el desarrollo de un pseudoaneurisma. Busque una masa consistente, no sensible, inmóvil que se levanta cerca del extremo de un hueso largo.

Page 112: Examen final 20460

CARACTERISTICAS ESQUEMATICAS HABITUALES DE LOS TUMORES OSEOS

TUMOR EDAD MAYOR FRECUENCIA

TOPOGRAFIA OBSERVACIONES

Condroma 10-30 Manos y pies Diáfisis Puede ser múltiple

Osteocondroma < 30 Rodilla, húmero Metáfisis Puede ser múltiple

Condroblastoma < 20 Húmero, fémur, tibia

Epífisis Frecuencia mayor en varones

Fibroma condromixoide

< 20 Tibia, fémur, tarso Metáfisis Puede recidivar

Condrosarcoma > 35 Huesos axiales, fémur

Diáfisis Puede ser secundario a osteocondroma

Osteoma < 25 Huesos faciales, calota

Senos paranasales

No en otros huesos

Osteoma osteoide < 30 Huesos largos, vértebras

Intracortical

Osteoblastoma < 30 Vértebras, huesos largos

Metáfisis

Osteosarcomas

Genuino < 25 Rodilla, húmero Metáfisis

Paraostal > 25 Fémur, húmero Metáfisis Rx típico

Posradiación muy variable

Sitio de radiación Sitio de radiación

> 3 años> 3000 r

Sarcoma de Ewing 3-20 Fémur, tibia, pelvis Diáfisis A veces múltiple

Linfomas 20-45 Cualquiera Diáfisis

Mieloma > 40 Cualquiera Múltiples

Tumor de células gigantes

> 20 Rodilla, extremo inferior del radio

Epífisis Recidiva. Frecuencia mayor en mujeres

Fibroma 20-40 Huesos largos Metáfisis

Histiocitoma fibroso maligno

> 20 Huesos largos, pelvis

Metáfisis Hay dos cúspides en edad

Hemangioma muy variable

Calota, vértebras Rx. típica

Cordoma

jóvenes < 20 Región cervical Intervertebral

adultos > 40 Región sacrococcígea

Intervertebral

Capítulo 12. Anatomía Patológica Osteoarticular. Dr. Martin Etchart

Page 113: Examen final 20460

98.- A 20-year-old man comes to the physician he has noticed blood in his urine on several occasions in the past year. Each episode of hematuria occurred in association with an upper respiratory tract infection or a flulike illness. Physical examination is unremarkable. A urine dipstick test shows mild proteinuria and microhematuria. Serum levels of electrolytes, creatinine, and blood urea nitrogen are within normal limits. Serum levels of IgA are elevated. Which of the following is the most likely diagnosis? a) Berger disease b) Goodpasture syndrome c) Henoch-Schönlein purpura d) Postinfectious glomerulonephritis

HEMATURIA GLOMERULAR AISLADA PERSISTENTE:

Definición Presencia de hematuria de origen glomerular (con acantocitos, y ocasionalmente cilindros hemáticos), sin otro elemento de inflamación glomerular, es decir, sin HTA, ni edema, con proteinuria menor a 1 gm /24 hs, y función renal normal, estable. En caso de hematuria persistente, aislada, las posibilidades principales es que se trate de Glomerulonefritis por IgA (lo más frecuente), enfermedad de membrana basal fina, Sd. de Alport, (raro) o el inicio de una glomerulonefritis crónica (< probable) (15) .También puede tratarse de una GN Post-Streptocócica, con persistencia de hematuria. A continucación se describirán brevemente:

Nefropatía por IgA: Enfermedad de Berger y síndrome de Shönlein-Henoch, se considerán por algunos como espectro de una misma enfermedad, siendo Berger la forma limitada al riñón. Es la Glomerulonefritis aguda más frecuente. Se da entre los 15 y 35 años, y es 3 veces más frecuente en hombres, tiene tres patrones clínicos de presentación:

1. Como hematuria macroscópica, 24 – 48 hs. posterior a cuadro respiratorio alto, asociado a dolor lumbar. Hematuria dura 2 – 6 días, rara vez presenta HTA o deterioro de función renal, y tiene proteinuria mínima. Recurre hasta en un 50%. De esta forma se presenta un 40-50%. 2. Como hematuria microscópica, con proteinuria leve, detectada solo en Sed. Orina, por rutina. 30 – 40 % se presenta así. 3. Como Sd. Nefrítico (HTA, Edema, Hematuria), pudiendo evolucionar algunos de estos casos como GNRP. Se presentan así un 10 %. Generalmente cursan con proteinuria menor a 1-2 gm/24 hs,. Pocos desarrollan Sd. nefrótico (10 %) Existe un grupo de pacientes (20 – 40 %), que evoluciona con falla renal progresiva, llegando a

Page 114: Examen final 20460

IRT en 5 – 25 años. Se ha detectado como factor de riesgo de esta evolución la presencia de Sd. nefrótico o proteinuria > 1gm/24h, , edad, HTA, deterioro de la función renal, ausencia de hematuria macroscópica. Si no tiene ninguno de estos factores, tiene poca probabilidad de desarrollar falla renal, y no se ha demostrado que el tratamiento, en este grupo de bajo riesgo, altere la evolución de la enfermedad. Al laboratorio, el complemento es generalmente normal, rara vez elevado. Existe aumento de IgA circulante entre un 30 – 50 %, sin ser específico de esta enfermedad. Para el diagnóstico, se requiere de biopsia renal, donde se identifican por inmunohistoquímica, la presencia de depósitos de IgA. Biopsia de piel carece de especificidad y de sensibilidad para diagnóstico de enfermedad de Berger. BIBLIOGRAFÍA 1.Jennette C, Falk R: Small Vesel Vasculitides. N Engl J Med 337:1512, 1997 2.Kashtan, CE Alport Síndrome and thin glomerular basement Membrane disease. JAM Soc Nephrol 1998;9:1736. 3.Falk R et al: Primary glomerular disease, en “Breneer & Rector´s, The Kidney, 6a ed, BM Brenner (editor). Phyladelfia, Saunders, 2000. pp 1263-1349. 4.Hricik, DE et al: Glomerulonephritis. N Eng J Med 339:889,1998 5.Antony BF. Attack rates of acute nephritis after type 49 streptoccocal infection of the skin and of the respiratory tract. J Clin Inv, 1969;48:1697. 6.Oliviera DBG. Poststreptococcal glomerulonephritis:getting to know an old enemy. Clin Rxp Immunol 1997;107:8-10 7.LangeK, et all. Evidence for the in situ origin of poststreptococcal glomerulonephritis: glomerular localization of endpstreptosin and the clinical significance of the subsequent antibody response. Clin Nephrol 1983;19:3-10 8.Rodriguez-Iturbe, B. Epidemic poststreptococcal glomerulonephritis, Kidney int 1984, 25:129. 9.Tejani A. Poststreptococcal glomerulonephritis: current clinical and pathologic concepts. Nephron 1990;55:15. 10. Potter EV. Twelve to seventeen year follow up of patients with poststreptococcal acute glomerulonephritis in Trinidad. N Engl J Med 1982; 307:725-9. 99.- Se trata de gestante de 27 semanas de evolución, presenta desde hace 6 días malestar

general, astenia, náuseas, cefalea, edemas y ligero dolor en hipocondrio derecho. Se realizan pruebas de lab con los siguientes resultados: Hb 8.5 g/dL, bilirrubina 1,5 mg/dL, LDH 670 UI/L, AST 182 UI/L, plaquetas 80.000/mm3. Lo más probable es que se trate de un embarazo complicado por:

a) Muerte fetal con paso de sustancias tromboplásticas a la circulación materna. b) Desprendimiento prematuro de placenta superior al 50%. c) Isoinmunización eritrocitaria con transfusión feto-materna. d) Preeclampsia.

Preeclampsia

La preeclampsia es un síndrome clínico caracterizado por hipertensión con disfunción orgánica múltiple, proteinuria, edemas.

Page 115: Examen final 20460

Es definida como un incremento de al menos 140/90 mmHg después de la semana 20 de gestación, un incremento en la presión sanguínea diastólica de al menos 15 mmHg respecto a un nivel previo a la semana 20 combinado con proteinuria (> 300 mg en 24 horas). Las mediciones de la presión arterial citadas deben ser medidas al menos 2 ocasiones con por lo menos 6 horas de separación. La proteinuria puede ser una toma simple de orina al azar que indique al menos 30 mg/dL 3 ó ++ en dos muestras de orina1 según el tipo de prueba. El criterio del incremento de 30 mmHg en la presión sistólica y/o 15 mmHg en la presión diastólica respecto a valores previos a la semana 20 de gestación ha sido eliminado por ser poco específico15

Page 116: Examen final 20460

Myers JE, Baker PN. Hupertensive diseases and eclampsia. Curr Opin Obstet Gynecol 2002; 14: 119-125 2. Tierney, McPhee, Papadakis. Diagnóstico clínico y tratamiento 2003. 38ª ed, México, Manual Moderno, 2003: 770-773 3. Wilson MI, Goodwin TM, Pan VI, Ingles SA. Molecular epidemiology of preeclampsia. Obstet and Gynecol Survey 2003; 58(1):39-66 4. Burrow GM. Complicaciones médicas durante el embarazo. 4ª ed, México, McGraw-Hill panamericana: 1996: 1-25 5. Guyton AC, Hall JE. Embarazo y lactancia en: Tratado de fisiología médica, 10ª ed, México, McGraw-Hill Interamericana 2001: 1135-45 6. Vaticon D. Fisiología de la fecundación, embarazo, parto y lactancia, en: Tresguerres JAF.

Page 117: Examen final 20460

Fisiología Humana. México, Interamericana McGraw-Hill, 1992: 1086-1109 100.- Se trata de femenino de 29 años acude a consulta por una secreción vaginal anormal con mal olor, como a “pescado”, que se hace más intenso después del coito. No refiere prurito genital. En la exploración se observa una abundante secreción blanco-grisácea, que no se adhiere a las pareces vaginales. Al mezclar una muestra de la secreción con una gota de hidróxido potásico al 10% se aprecia claramente el mal olor referido. De los siguientes es el tratamiento de elección para esta paciente es: a) Amoxicilina con Acido Clavulánico por vía oral. b) Clindamicina por vía intravaginal. c) Clotrimazol por vía intravaginal. d) Doxiciclina por vía oral.

Gardnerella vaginalis fue clasificada como una sola especie y fue establecida como agente causal de la vaginosis (antes conocida como vaginitis inespecífica). El cuadro clínico que presenta es caracterizado por una secreción blanca o blanco-grisácea que se percibe generalmente después de la relación sexual con olor fétido aminado (pescado). El diagnóstico certero es la base para evitar posibles complicaciones como la enfermedad inflamatoria pelviana y las complicaciones del embarazo. El tratamiento se basa principalmente en los fármacos como: metronidazol y clindamicina, debido a su efectividad y espectro, pero como todos se deben emplear con adecuada prudencia debido a su toxicidad. Además de que se deben corregir o modificar los factores predisponentes, ya que esta patología va en aumento convirtiéndose por su frecuencia en un problema de salud pública. Bibliografía: 1. Hernández F. Gardnerella vaginalis mobiluncus en la etiología de la vaginosis bacteriana. Rev Costarricense Ciencias Médicas 1998; 19: 57-61. 2. Hansen EA. Gardnerella. Rev Ginecol 2005; 25: 99. 3. Espinosa I, Lorenzo M, Bentancourt A, Riverón Y, Romero M. Caracterización bioquímica y antigénica de diferentes aislamientos de Gardnerella vaginalis. Rev Cubana Invest Biomed 2005; 24: 22-7. 4. Taylor F. Vaginal flora morphotypic profiles and assessment of bacterial vaginosis in women at risk for HIV infection.Infect Dis Obstet Gynecol 2004; 12: 121-6.